Download as doc, pdf, or txt
Download as doc, pdf, or txt
You are on page 1of 121

База Біохімія ЗЛП для іспиту B.

В серцевому м'язі (можливий


Ферменти інфаркт міокарда).
1. Сульфаніламідні препарати C. В скелетних м'язах.
нагадують по структурі D. В нирках.
параамінобензойную кислоту. У чому E. В сполучній тканині.
полягає молекулярна основа їх
фармакологічної дії? 5. У хворого виявлено підвищення
A. * В порушенні синтезу вітаміну. активності ЛДГ1,2, АсАТ,
B. У зв'язуванні з ДНК. креатинфосфокінази. В якому органі
C. В інгібування гліколізу. (органах) ймовірний розвиток
D. В активації ліполізу. патологічного процесу?
E. У руйнуванні клітинної мембрани. A. * В серцевому м'язі (початкова
стадія інфаркту міокарда).
2. Хвора 46 років тривалий час B. У скелетних м'язах (дистрофія,
страждає прогресуючою м'язовою атрофія).
дистрофією (Дюшена). Зміна рівня C. В нирках і наднирниках.
якого ферменту крові є D. В сполучній тканині.
діагностичним тестом в даному E. В печінці та нирках.
випадку?
A. * Креатинфосфокінази. 6. Хворому поставили діагноз інфаркт
B. Лактатдегідрогенази. міокарда. Характерною ознакою для
C. Піруватдегідрогенази. даного захворювання є підвищення в
D. Глутаматдегідрогеназа. крові активності:
E. Аденілаткінази. A. * Кретінфосфокінази.
B. каталази.
3. З сироватки крові людини виділили C. Глюкозо-6фосфатдегідрогенази.
п'ять ізоферментних форм D. A-амілази.
лактатдегідрогенази і вивчили їх E. аргінази.
властивості. Яка властивість
доводить, що виділені ізоферментні 7. При дослідженні крові у хворого
форми одного і того ж ферменту? виявлено значне збільшення
A. * Каталізірованіе однієї і тієї ж активності МВ-форм КФК
реакції. (креатинфосфокінази) та ЛДГ-1. Яка
B. Однакова молекулярна маса. можлива патологія?
C. Однакові фізико-хімічні A. * Інфаркт міокарда.
властивості. B. Гепатит.
D. Тканинна локалізація. C. Ревматизм.
E. Однакова електрофоретична D. Панкреатит.
рухливість. E. Холецистит.

4. В крові хворої виявлено 8. На основі клінічних даних хворому


підвищення активності ЛДГ 4,5, поставлений діагноз гострий
АлАТ, карбамоілорнітінтрансферази. панкреатит. Вкажіть біохімічний тест,
В якому органі можна передбачити який підтверджує цей діагноз:
розвиток патологічного процесу? A. * Активність амілази крові.
A. * У печінці (можливий гепатит). B. Активність кислої фосфатази крові.
C. Активність лужної фосфатази знайдено різке підвищення активності
крові. АсАТ в сироватці крові. Вкажіть
D. Активність амінотрансфераз крові. патологію, для якої це характерно:
E. Рівень креатиніну в крові. A. * Інфаркт міокарда.
B. Вірусний гепатит.
9. Для лікування деяких інфекційних C. Колагенози.
захворювань, що викликаються D. Цукровий діабет.
бактеріями, застосовуються E. Нецукровий діабет.
сульфаніламідні препарати, що
блокують синтез фактора росту 13. У юнака 18 років з ураженням
бактерій. Виберіть механізм дії паренхіми печінки в сироватці крові
сульфаніламідних препаратів: найбільш вірогідно буде виявлена
A. * Є антивітамінами підвищена активність:
парамінобензойной кислоти. A. * Аланінамінотрансфераза.
B. Інгібують всмоктування фолієвої B. ЛДГ1.
кислоти. C. Креатинкінази.
C. Є аллостеричними інгібіторами D. Кислого фосфатази.
ферментів. E. A-амілази.
D. Беруть участь в окисно-відновних
процесах. 14. У сироватці крові хворої знайдена
E. є аллостеричними ферментами. висока активність ізоферменту ЛДГ1.
О патологічному процесі в якому
10. У відділення реанімації поступив органі це свідчить?
чоловік 47 років з діагнозом інфаркт A. * В серці.
міокарда. Яка з фракцій B. У печінки.
лактатдегідрогенази (ЛДГ) буде C. У скелетних м'язах.
переважати в сироватці крові D. У підшлунковій залозі.
протягом перших двох діб? E. У нирках.
A. * ЛДГ1.
B. ЛДГ2. 15. Хворого доставила в стаціонар
C. ЛДГ3. швидка допомога з попереднім
D. ЛДГ4. діагнозом - гострий панкреатит.
E. ЛДГ5. Визначення активності якого
ферменту в крові і в сечі необхідно
11. У відділення інтенсивної терапії провести для підтвердження цього
доставлено жінку 50 років з діагнозом діагнозу?
інфаркт міокарда. Активність якого A. * Aльфа-амілази.
ферменту буде найбільш підвищена B. АлАТ.
на протязі перших двох діб? C. АсАТ.
A. * Аспартатамінотрансфераза. D. Лактатдегідрогенази.
B. Аланінамінотрансфераза. E. холінестеразою.
C. Аланінамінопептідази.
D. ЛДГ4. 16. Діагностичним тестом при
E. ЛДГ5. гострих панкреатитах є визначення в
сечі активності таких ферментів:
12. У хворого через 12 годин після A. * Амілази.
гострого нападу грудного болю B. Лактатдегідрогенази.
C. Креатинкінази.
D. альдолаза. 21. До лікарні поступив хворий з
E. Аланінамінопептідази. підозрою на гострий панкреатит.
Підвищення активності якого
17. Активність яких ферментів слід ферменту слід очікувати при цьому?
визначати з діагностичної та A. * Амілази.
прогностичної метою, якщо в клініку B. Пепсину.
поступив хворий з патологією C. гастріксіна.
серцевого м'яза? D. Креатинкінази.
A. * Креатинкінази, АлАТ, АсАТ. E. Аспартаттрансамінази.
B. аргінази, пептідази, фосфатази.
C. Лізоциму, цітратсінтази, 22. У чоловіка 58 років клінічна
альдолази. картина гострого панкреатиту.
D. нейрамінідази, гексокінази, Підвищення в сечі якого з
піруваткінази. перерахованих нижче речовин буде
підтвердженням діагнозу?
18.У хворого на гострий панкреатит A. * Амілази.
при аналізі крові і сечі різко B. Залишкового азоту.
підвищена активність одного із C. Сечовини.
зазначених ферментів, що D. Альбуміну.
підтверджує діагноз захворювання: E. Сечовиой кислоти
A. * Альфа-амілази.
B. Пепсину.
C. дипептидаза. 23. Літній чоловік звернувся до лікаря
D. Сахарози. після появи болю в грудній клітині. В
E. лактази. сироватці крові виявлено значне
зростання активності ферментів:
19. Назвіть фермент, визначення креатінфосфокінази і ееМВ-ізоформи,
якого в крові є найбільш аспартатамінотрансферази. Про
інформативним в перші години після розвиток патологічного процесу в
виникнення інфаркту міокарда: якій тканині свідчать ці зміни?
A. * Креатинфосфокиназа. A. * В серцевому м'язі.
B. Аспартатамінотрансфераза. B. В тканині легень.
C. Аланінамінотрансфераза. C. В скелетних м'язах.
D. Лактатдегидрогеназа. D. В тканині печінки.
E. глутаматдегідрогеназа. E. В гладких м'язах.

20. У хворого К. в сечі підвищена 24. У хворого гострий панкреатит.


амілазна активність і виявлено При аналізі крові і сечі різко
наявність трипсину, в крові теж підвищена активність одного із
підвищена амілазна активність. Про зазначених ферментів:
патологію якого органу це свідчить? A. * Альфа-амілази.
A. * Підшлункової залози. B. Пепсину.
B. Печінки. C. дипептидаза.
C. Шлунка. D. Сахарози.
D. Нирок. E. лактази..
E. Кишечника.
важливе місце належить їх
25. У слині міститься фермент, який постсинтетичній ковалентній
має сильну бактерицидну дію, модифікації. Яким із зазначених
завдяки здатності руйнувати механізмів здійснюється регуляція
пептидоглікани бактеріальної стінки. активності глікогенфосфорилази і
Вкажіть цей фермент. глікогенсинтетазу?
A. * Лізоцим (мурамідаза) A. * Фосфорилування
. B. α-амілаза. дефосфорілірованіе.
C. Трипсин. B. метилування.
D. Фосфатаза. C. Аденілірованіем.
E. Рибонуклеаза. D. Обмеженим протеолізом.
E. АДФ-рібозілірованіе.
26. При нанесенні стоматологом
пероксиду водню на слизову 30. Захисна функція слини зумовлена
оболонку порожнини рота з'явилася декількома механізмами, в тому числі
інтенсивна піна. Наявність якого наявністю ферменту, що володіє
ферменту розкладає перекис водню? бактерицидною дією. Він викликає
A. * каталази. лізис полісахаридного комплексу
B. холінестеразу. оболонки стафілококів і стрептококів.
C. Ацетілтрасферази. Вкажіть цей фермент.
D. Глюкозо-6-фосфатдегідрогенази. A. * Лізоцим.
E. метгемоглобінредуктази. B. Альфа-амілаза.
C. Олиго-1,6-глюкозидази.
27. Недостатня секреція якого D. колагеназа.
ферменту призводить до неповного E. Бета-глюкуронідаза.
перетравлення жирів в шлунково-
кишковому тракті і поява великої 31. Хворому поставлений попередній
кількості нейтральних жирів в калі? діагноз - інфаркт міокарда.
A. * панкреатичної ліпази. Характерною рисою для даного
B. Фосфоліпази. захворювання є суттєве підвищення в
C. Ентерокінази. крові активності:
D. Амілази. A. * Кретінфосфокінази.
E. Пепсину. B. каталази.
C. Г-6-ФДГ.
28. Ті організми, які в процесі D. a-амілази.
еволюції не створили захисту від E. аргінази.
Н2О2, можуть жити тільки в
анаеробних умовах. Які з 32. Для запобігання нападів гострого
перерахованих ферментів можуть панкреатиту лікар призначив
руйнувати пероксид водню? трасилол (контрикал, гордокс), який є
A. * Пероксидаза і каталаза. інгібітором:
B. оксигенази і гідроксилази. A. * трипсину.
C. Цитохромоксидази, цитохром В5. B. еластази.
D. оксигенази і каталаза. C. Карбоксипептидаза.
E. Флавінзавісімие оксидази. D. хімотрипсину.
E. гастріксіна.
29. У регуляції активності ферментів
B. Порушення синтезу білків
33. Хворому поставлений діагноз: клітинної стінки.
хвороба бері-бері. Активність якого C. Зменшення проникності мембран.
ферменту порушена у пацієнта? D. Пригнічення сульфгідрильних груп
A. * піруватдегідрогенази. тіолових ферментів.
B. Цітратсінтази. E. Коагуляція білка.
C. малатдегідрогеназа.
D. сукцинатдегідрогенази. Вітаміни
E. фумарази. 1. Хворому з гіпоплазією твердих
тканин зуба стоматолог призначив
34. При обробці перекисом водню вітаміни А і Д перорально. На чому
слизової оболонки хворого, який заснована тактика лікування?
страждає запаленням ротової A. * Ці вітаміни регулюють обмін
порожнини, кров пофарбувалася в гетерополісахаридів зуба і сприяють
коричневий колір замість піни. При відкладенню солей кальцію.
зниженні концентрації якого з B. Ці вітаміни сприяють
перерахованих ферментів це перетворенню проколагену в колаген.
можливо? C. Ці вітаміни активують
A. * Каталази енергетичний обмін в тканинах зуба.
B. псевдохолінестерази D. Ці вітаміни сприяють заміні
C. Глюкозо -6- фосфатдегидрогеназа стронцієвого апатиту
D. Ацетілтрансферазой гидроксиапатитом.
E. метгемоглобінредуктази E. Ці вітаміни забезпечують
антиоксидантні властивості тканин
35.У хворого на гострий панкреатит зуба.
виникає загроза некрозу
підшлункової залози, 2. У дитини спостерігається затримка
супроводжується надходженням в прорізування зубів, неправильне їх
кров і тканини активних розташування. Також є скарги на
панкреатичних протеїназ і сухість у роті, поява тріщин в кутах
розщепленням тканинних білків. Які рота з подальшим нагноєнням. З
захисні фактори організму можуть недоліком якого вітаміну це може
пригнічувати ці процеси? бути пов'язано?
A. * Альфа2 - макроглобулін, альфа1 - A. * А.
антитрипсин B. Д.
B. Імуноглобуліни C. Є.
C. Кріоглобуліни, інтерферон D. С.
D. Церулоплазмін, трансферин E. К.
E. гемопексин, гаптоглобін
3. Дівчинка 10 років часто хворіє на
36. Хворому стоматитом призначений гострі респіраторні інфекції, після
препарат з групи сульфаніламідів. яких спостерігаються множинні
Який механізм його точкові крововиливи в місцях тертя
антибактеріальної дії? одягу. Вкажіть, гіповітаміноз якого
A. * Конкурентний антагонізм з вітаміну має місце у дівчинки:
ПАБК. A. * С.
B. В6.
C. В1. B. А.
D. А. C. Є.
E. В2. D. Д.
E. F.
4. У жінки 35 років з хронічним
захворюванням нирок розвинувся 8. У хворого спостерігаються
остеопороз. Вкажіть дефіцит якого з дерматит, діарея, деменція.
нижче перерахованих речовин є При анамнезі виявлено, що основним
основною причиною цього: продуктом харчування хворого є
ускладнення: кукурудза. Дані порушення пов'язані
A. * 1. 25 (OH) 2 D3. з дефіцитом:
B. 25 OH D3. A. * Вітаміну РР.
C. D3. B. Вітаміну В1.
D. D2. C. Вітаміну В2.
E. Холестерину. D. Вітаміну В9.
E. Вітаміну В12.
5. Після епілептичного нападу
педіатром було оглянуто немовля, яке 9. Окуліст виявив у хворого
одержує штучну їжу. У дитини збільшення часу адаптації ока до
виявлено також дерматит. При темряви. Недостатність якого
лабораторному обстеженні вітаміну може бути причиною такого
встановлено зниження аланін- і симптому?
аспартаттрансаміназной активності A. * Вітаміну А.
еритроцитів. Недолік якого вітаміну B. Вітаміну Е.
можна припустити? C. Вітаміну С.
A. * Піридоксину. D. Вітаміну К.
B. Аскорбінової кислоти. E. Вітаміну D.
C. кобаламіна.
D. Рибофлавіну. 10. Хворий скаржився на загальну
E. Кальциферолу. слабкість і кровотеча з ясен.
Недостатність якого вітаміну можна
6. Вагітній жінці, яка мала в анамнезі припустити?
декілька викиднів, призначена A. * Вітаміну С.
терапія, яка містить вітамінні B. Вітаміну Е.
препарати. Вкажіть вітамін, який C. Вітаміну РР.
сприяє виношування вагітності: D. Вітаміну D.
A. * Альфа-токоферол. E. Вітаміну В1.
B. Фолієва кислота.
C. Ціанкобаламін. 11. При обстеженні хворого виявлено
D. Піридоксальфосфат. дерматит, діарея, деменція. Вкажіть,
E. Рутин. відсутність якого вітаміну є
причиною цього стану:
7. У хворих з закупоркою загальної A. * нікотинамід.
жовчної протоки виникають B. Аскорбінової кислоти.
геморагії, що пов'язано з поганим C. Фолієвої кислоти.
засвоєнням вітаміну: D. біотин.
A. * К. E. Рутина.
D. біотин.
12. До лікаря звернувся хворий зі E. Токоферолу.
скаргами на постійну спрагу.
Виявлено гіперглікемія, поліурія і 16. У моркві, гарбузі і деяких
підвищена концентрація 17- червоних овочах містяться каротин.
кетостероїдів в сечі. Яке Нестача якого вітаміну заповнюють ці
захворювання ймовірно? рослинні пігменти?
A. * Стероїдний діабет. A. * Ретинолу.
B. Інсулінозалежний діабет. B. Рутина.
C. Мікседема. C. Рибофлавіну.
D. Глікогеноз I типу. D. Токоферола.
E. Аддисонова хвороба. E. Кальциферолу.

13. У хворого в крові збільшена 17. При різноманітних захворюваннях


концентрація пірувату. Значна його рівень активних форм кисню різко
кількість виводиться з сечею. Який зростає, що призводить до
авітаміноз спостерігається у хворого? руйнування клітинних мембран. Для
A. * Авітаміноз В1. запобігання цьому використовують
B. Авітаміноз Е. антиоксиданти. Найпотужнішим
C. Авітаміноз В3. природним антиоксидантом є:
D. Авітаміноз В6. A. * Альфа-токоферол.
E. Авітаміноз В2. B. Глюкоза.
C. Вітамін Д.
14. У пацієнта захворювання пелагра. D. Жирні кислоти.
При опитуванні стало відомо, що E. Гліцерол.
протягом тривалого часу він
харчувався переважно кукурудзою, 18. У новонародженої дитини
вживаючи мало м'яса. Що стало з'явилися симптоми геморагічної
причиною виникнення пелагри? хвороби в зв'язку з гіповітамінозом К.
A. * Дефіцит триптофану в кукурудзі. Розвиток захворювання обумовлено
B. Дефіцит тирозину в кукурудзі. особливою біологічною роллю
C. Дефіцит проліну в кукурудзі. вітаміну К, який:
D. Дефіцит аланіну в кукурудзі. A. * Є кофактором
E. Дефіцит гістидина в кукурудзі. гаммаглутамілкарбоксілази.
B. Є кофактором
15. На прийом до лікаря звернувся протромбіну.
хворий з симетричним дерматитом C. Є специфічним інгібітором
відкритих ділянок шкіри. З бесіди з антитромбіном.
пацієнтом встановлено, що в D. Впливає на протеолітичну
основному він харчується крупами і активність тромбіну.
їсть мало м'яса, молока і яєць. E. Пригнічує синтез гепарину.
Дефіцит якого з перерахованих
вітамінів є провідним у даного 19. У хворого виявлена болючість по
пацієнта? ходу великих нервових стовбурів і
A. * нікотинамід. підвищена кількість пірувату в крові.
B. Кальциферолу. Недостатність якого вітаміну може
C. Фолієвої кислоти. викликати такі зміни?
A. * В1. кишечника. Призначення якого
B. В2. вітаміну буде стимулювати цей
C. РР. процес?
D. пантотенова кислота. A. * Віт Д3.
E. біотин. B. Віт РР.
C. Віт С.
20. Велика частина учасників D. Віт В12.
експедиції Магелана в Америку E. Віт К.
загинула від захворювання, яке
виявлялося загальною слабкістю, 25. Інститут геронтології людям
підшкірними крововиливами, похилого віку радить вживати
випадінням зубів, кровотечею ясен. комплекс вітамінів, який містить
Вкажіть назву цього авітамінозу: вітамін Е. Яку головну функцію він
A. * Скорбут (цинга). виконує?
B. Пелагра. A. * Антиоксидантну.
C. Рахіт. B. Антигеморагічну.
D. Поліневрит (бері-бері). C. антискорбутную.
E. Анемія Бірмера. D. Антиневрологічну.
E. Антидерматичну.
21. Під час патронажу лікар виявив у
дитини симетричну шорсткість щік, 24. У хворого після видалення
діарею, порушення нервової жовчного міхура утруднений процес
діяльності. Недолік будь харчових всмоктування Са через стінку
факторів є причиною такого стану? кишечника. Призначення якого
A. * Нікотинової кислоти, вітаміну буде стимулювати цей
триптофану. процес?
B. Лізину, аскорбінової кислоти. A. * Віт Д3.
C. треонін, пантотенової кислоти. B. Віт РР.
D. Метіоніну, ліпоєвої кислоти. C. Віт С.
E. фенілаланіну, пангамовой кислоти. D. Віт В12.
E. Віт К.
23. У жінки, що тривалий час
знаходиться на дієті з використанням 25. Інститут геронтології людям
очищеного рису, виявлений похилого віку радить вживати
поліневрит (хвороба бері-бері). комплекс вітамінів, який містить
Відсутність якого вітаміну в їжі вітамін Е. Яку головну функцію він
призводить до розвитку цього виконує?
захворювання? A. * Антиоксидантну.
A. * Тіаміну. B. Антигеморагічну.
B. Аскорбінової к-ти. C. антискорбутную.
C. Піридоксаль. D. Антиневрологічну.
D. Фолієвої к-ти. E. Антидерматичну
E. Рибофлавіну.
26. Після курсу терапії хворому з
24. У хворого після видалення виразкою дванадцятипалої кишки
жовчного міхура утруднений процес лікар пропонує вживання соків з
всмоктування Са через стінку капусти і картоплі. Зміст якої
речовини в цих продуктах сприятливо C. Трансамінази.
діє для профілактики і загоєння D. альдолаза.
виразок? E. гексокіназа.
A. * Вітамін U.
B. Пантотеновакислота. 30. Лікування дитини, хворої на рахіт,
C. Вітамін С. за допомогою вітаміну Д3 не дало
D. Вітамін В1. позитивного результату. Яка
E. Вітамін К. найбільш імовірна причина
неефективності лікування?
27. Лікар-дієтолог радить хворому під A. * Порушення гідроксилювання
час лікування перніциозної анемії вітаміну Д3.
вживати в раціоні харчування B. Недостатність ліпідів в їжі.
напівсиру печінку. Наявність якого C. Порушення включення вітаміну Д3
вітаміну в цьому продукті сприяє в фермент.
лікуванню анемії? D. Підвищене використання вітаміну
A. * Вітаміну В12. Д3 мікрофлорою кишечника.
B. Вітаміну В1. E. Порушення транспорту вітаміну
C. Вітаміну В2. Д3 білками крові.
D. Вітаміну С.
E. Вітаміну Н. 31. Після лікування хворого
антибіотиками внаслідок придушення
28. У чоловіка, який тривалий час не мікрофлори кишечника можливий
вживав з їжею жирів, але отримував гіповітаміноз вітамінів:
достатню кількість вуглеводів і A. * В12.
білків, виявлено дерматит, погане B. С.
загоєння ран, погіршення зору. Яка C. А.
можлива причина порушення обміну D. Р.
речовин? E. Д.
A. * Нестача лінолевої кислоти,
вітамінів А, Д, Е, К. 32. Вітамін А в комплексі зі
B. Нестача пальмітинової кислоти. специфічними циторецепторів
C. Нестача вітамінів РР, Н. проникає через ядерні мембрани,
D. Низька калорійність дієти. індукує процеси транскрипції,
E. Нестача олеїнової кислоти. стимулює ріст і диференціювання
клітин. Ця біологічна функція
29. Хворі на алкоголізм отримують реалізується наступною формою
основну масу калорій зі спиртними вітаміну А:
напоями. У них може виникнути A. * Трансретіноевая кислота.
характерна недостатність тіаміну B. Трансретіналь.
(синдром Верніке-Корсакова), при C. Цісретіналь.
якій спостерігаються порушення D. Ретинол.
функцій нервової системи, психози, E. Каротин.
втрата пам'яті. Зі зниженням
активності якого ферменту це може 33. При недостатності тіаміну -
бути пов'язано? вітаміну В1 виникає хвороба бері-бері
A. * піруватдегідрогенази. (поліневрит). Який метаболіт при
B. Алкогольдегідрогенази. цьому накопичується в крові?
A. * Піруват. E. Вітаміну В1.
B. Лактат.
C. Сукцинат. 38. Хворому, що страждає
D. Цитрат. тромбоемболічної хворобою,
E. Малат. призначений штучний антикоагулянт
пелентан. Антагоністом якого
34. Злоякісна гіперхромні анемія - вітаміну є це з'єднання?
хвороба Бірмера - виникає внаслідок A. * Вітаміну К.
нестачі вітаміну В12. Який B. Вітаміну Е.
біоелемент входить до складу цього C. Вітаміну А.
вітаміну? D. Вітаміну Д.
A. * Кобальт. E. Вітаміну С.
B. Молібден.
C. Цинк. 39. У хворого спостерігається
D. Залізо. гемералопія (куряча сліпота). Яке з
E. Магній. перерахованих речовин буде мати
лікувальну дію?
35. У хворого відзначені такі зміни: A. * Каротин.
порушення зору в сутінках, сухість B. Кератин.
кон'юнктиви і рогівки. Такі C. Креатин.
порушення можуть бути при нестачі: D. Карнітин.
A. * Вітаміну А. E. Карнозин.
B. Вітаміну В.
C. Вітаміну С. 40. Хворий скаржиться на загальну
D. Вітаміну D. слабкість і кровоточивість ясен.
E. Вітаміну В12. Недодостаток якого вітаміну може
бути причиною такого стану?
36. Після видалення 2/3 шлунка в A. * Вітаміну С.
крові зменшилася кількість B. Вітаміну Е.
еритроцитів, збільшився їх обсяг, C. Вітаміну А.
знизився рівень гемоглобіну. Дефіцит D. Вітаміну Н.
якого вітаміну призводить до таких E. Вітаміну Д.
змін картини крові?
A. * В12. 41. У хворого 36 років, який страждає
B. С. на хронічний алкоголізм, в крові має
C. Р. місце накопичення пірувату, в
D. В6. еритроцитах - зниження активності
E. РР. транскетолази. Назвіть
коферментную форму вітаміну,
37. При якому гіповітамінозі недостатністю якого обумовлені
спостерігається одночасне порушення зазначені зміни:
репродуктивної функції і дистрофія A. * Тіаміндифосфат.
скелетної мускулатури? B. Карбоксібіотін.
A. * Вітаміну Е. C. Метилкобаламін.
B. Вітаміну А. D. фосфопірідоксаль.
C. Вітаміну К. E. тетрагідрофолат.
D. Вітаміну Д.
42. У хворого 43 років B. Фосфорилювання радикалів
спостерігається хронічний серину.
атрофічний гастрит, мегалобластна C. Частковий протеоліз.
гіпохромна анемія. Підвищується D. Полімеризація.
виділення метилмалоновой кислоти з E. Глікозилювання.
сечею. Недостатністю якого вітаміну
обумовлено виникнення зазначеного 46. Після лікування запального
порушення? процесу антибіотиками у хворого
A. * Вітаміну В12. порушилося згортання крові
B. Вітаміну В2. внаслідок ураження мікрофлори
C. Вітаміну В3. кишечника. Недолік якого вітаміну
D. Вітаміну В5. спостерігається у хворого?
E. Вітаміну В1. A. * К.
B. В1.
43. Для діагностики ряду захворювань C. Д.
визначають активність трансаміназ D. Р.
крові. Який вітамін входить до складу E. С.
кофакторів цих ферментів?
A. * В6. 47. У хворого в результаті
B. В2. неповноцінного харчування з'явилася
C. В1. діарея, деменція і дерматит.
D. В8. Недодостатком якого вітаміну
E. В5. викликано дане стан?
A. Вітаміну РР.
44. У дитини 2 років після тривалої B. Вітаміну В1.
антибіотикотерапії розвинувся C. Вітаміну В2.
дисбактеріоз: майже повна D. Вітаміну С.
відсутність кишкової палички. E. Вітаміну В12.
Недостатність вітамінів якої групи
може виникнути в зв'язку з цим? 48. У пацієнта при повноцінному
A. * В. харчуванні розвинулася
B. А. гиперхромная (мегалобластична)
C. С. анемія. Напередодні він переніс
D. Е. операцію з резекції шлунка. Яка
E. D. причина анемії?
A. * Недодостаток фактора Касла.
45. Для запобігання післяопераційної B. Недодостаток вітаміну С в їжі.
кровотечі 6-річній дитині C. Недодостаток вітаміну РР в їжі.
рекомендовано приймати вікасол, D. Недодостаток білка в їжі.
який є синтетичним аналогом E. Недодостаток фолієвої кислоти в
вітаміну К. Вкажіть, які їжі.
Посттрансляційні зміни факторів
згортання крові активуються під 49. У 2-річну дитину кишковий
впливом вікасолу: дисбактеріоз, на фоні якого з'явився
A. * Карбоксилювання геморагічний синдром. Найбільш
глутамінової кислоти. можливою причиною геморагії у цієї
дитини є:
A. * Нестача вітаміну К. Вкажіть, яке з наведених природних
B. Активація тромбопластину тканин. сполук використовується в якості
C. Гіповітаміноз РР. антиоксидантного засобу?
D. Дефіцит фібриногену. A. * Токоферол.
E. Гіпокальціємія. B. Тіамін.
C. Глюконат.
50. У дитини при черговому D. Пиридоксин.
обстеженні виявлено припинення E. Холін.
мінералізації кісток. Недодостаток
якого вітаміну є причиною цього? 55. Для профілактики пелагри
A. * Кальциферолу. (авітамінозу РР) в організм людини
B. Рибофлавіну. має надходити така амінокислота, яка
C. Токоферолу. є попередником вітаміну РР, як:
D. Фолієвої кислоти. A. * Триптофан.
E. кобаламина. B. Глутамат.
C. Аспартат.
51. У хворого різко підвищилася D. Метіонін.
кровоточивість ясен. Які вітаміни слід E. Гліцин.
призначити цьому пацієнту? 56. Який вітамін завдяки гідрофобним
A. С, К. боковому радикалу вбудовується в
B. В1, В2. фосфоліпідний матрикс біомембрани,
C. А, Е. стабілізуючи його, і виконує функцію
D. РР, В12. потужного біоантиоксидант?
E. Біотин, пантотенову кислоту. A. * Токоферол.
B. Вітамін D.
52. Хворому поставлений діагноз C. Вітамін В6.
бери-бери. Активність якого D. Никотинамид.
ферменту порушена у пацієнта? E. Ціанкобаламін.
A. * піруватдегідрогенази.
B. Цітратсінтази. 57. Прояви аліментарної
C. малатдегідрогеназа. недостатності ніацину (пелагри)
D. сукцинатдегідрогенази. можуть бути пом'якшені збільшенням
E. фумарази. в дієті амінокислоти:
A. * Триптофану.
53. Інститут геронтології людям B. Пролін.
похилого віку радить вживати C. Тирозину.
комплекс вітамінів, містить вітамін Е. D. треонін.
Яку головну функцію він виконує? E. лейцин.
A. * Антиоксидантну.
B. Антігеморагіческую. 58. У хворого діагностовано
C. Антіскорбную. мегалобластна анемія. Вкажіть
D. Антіневріческую. з'єднання, недостатня кількість якого
E. Антідерматіческую. може призводити до розвитку цієї
хвороби.
54. У процесі лікування пародонтозу A. * Ціанокобаламін.
застосовують антиоксидант B. Гліцин.
природного та штучного походження. C. Мідь.
D. Холекальциферол. 63. Відомо, що молекула колагену
E. Магній. містить амінокислоти (оксипролина,
оксилізину). Які з перерахованих
59. У пацієнта після вживання сирих речовин беруть участь в
яєць з'явилися дерматити. Який гидроксилировании пролина при
розвинувся авітаміноз? синтезі колагену?
A. * Авітаміноз біотину. A. * Аскорбінова кислота.
B. Авітаміноз фолієвої кислоти. B. Фолієва кислота.
C. Авітаміноз пантотенової кислоти. C. Пантотеновакислота.
D. Авітаміноз D. Глутаминовая кислота.
параамінобензойноїкислоти. E. Аспарагінова кислота.
E. Авітаміноз інозиту.
64. У юнака 20 років діагностували
60. У моркві, гарбузі і інших макроцитарну анемію, в сечі
корисних овочах містяться каротин. підвищений рівень метилмалоновой
Недолік якого вітаміну заповнюють кислоти. В першу чергу це
ці рослинні пігменти? обумовлено дефіцитом:
A. * Ретинолу. A. * Ціанкобаламін.
B. нафтохінона. B. Нікотинової кислоти.
C. Рибофлавіну. C. пантотенова кислота.
D. Токоферола. D. Аскорбінової кислоти.
E. Кальциферолу. E. біотин.

61. При різних захворюваннях рівень 65. Юнак 20 років звернувся до лікаря
активних форм кисню різко зростає, зі скаргами на загальну слабкість,
що призводить до руйнування швидку стомлюваність, дратівливість,
клітинних мембран. Для запобігання зниження працездатності,
цьому використовують кровоточивість ясен, петехії на шкірі.
антиоксиданти. Потужним Недостатність якого вітаміну може
природним антиоксидантом є: мати місце в даному випадку?
A. * Альфа-токоферол. A. * Аскорбінової кислоти.
B. глюколов. B. Рибофлавіну.
C. Вітамін Д. C. Тіаміну.
D. Жирні кислоти. D. Ретинолу.
E. Гліцерол. E. Фолієвої кислоти

62. У хворого виявлена болючість по 66. При обстеженні хворого


ходу великих нервових стовбурів і атрофічним гастритом виявлена
підвищений вміст пірувату в крові. мегалобластна анемія. Дефіцит якої
Недостатність якого вітаміну може речовини є причиною виникнення
викликати такі зміни? анемії у цього хворого?
A. * В1. A. * гастромукопротеид.
B. В2. B. Вітамін B6.
C. РР. C. Вітамін B1.
D. Пантотенова кислота. D. Еритропоетин.
E. Біотин. E. Залізо.
67. У хворого діагностовано 71.Прі недостатності тіаміну -
себорейний дерматит, пов'язаний з вітаміну В1 виникає захворювання
дефіцитом вітаміну H (біотину). бери-бери (поліневрит) і порушується
Порушення активності якого з вуглецевий обмін. Який метаболіт
перерахованих ферментів при цьому накопичується в крові?
спостерігається у хворого? A. * Піруват.
A. * Ацетил-КоА-карбоксилаза B.Діоксіацетонфосфат.
B. Алкольдегідрогеназа. C.Сукцінат.
C. Амінотрансферази. D.Ізоцітрат.
D. Карбомоілфосфатсінтетаза. E.Оксалоацетат.
E. піруватдекарбоксилази.
72.У пацієнта при повноцінному
68. У дитини 2-х років кишковий харчуванні розвинулася
дисбактеріоз, на фоні якого виник гиперхромная (мегалобластічна)
геморагічний синдром. Найбільш анемія. Напередодні ВІН переніс
імовірною причиною геморагій у цієї операцію з резекції шлунка. Яка
дитини є: причина анемії?
A. * Нестача вітаміну К. A. * Недолік фактора Касла.
B. Дефіцит фібриногену. B. Нестача вітаміну Е в їжі.
C. Гіповітаміноз РР. C. Нестача вітаміну В2 в їжі.
D. Гіпокальціємія. D. Недолік Білка в їжі.
E. Активація тромбопластину тканин. E. Нестача вітаміну В1 в їжі.

69. Через рік після субтотальної 73. У хворого спостерігаються


резекції шлунка з приводу виразки дерматит, діарея, деменція. Основним
малої кривизни виявлено зміни в продуктом харчування хворого є
крові - анемія, лейко- і кукурудза. З недоліком якого вітаміну
тромбоцитопенія, ЦП- 1,3, наявність пов'язані дані порушення?
мегалобластів і мегалоцитів. Дефіцит A. * Вітамін PP.
якого фактора зумовив розвиток цієї B. Вітамін B12.
патології? C. Вітамін B6.
A. * Фактор Касла. D. Вітамін B1.
B. Хлористоводнева кислота. E. Вітамін B2.
C. Пепсин.
D. Гастрін. 74. У хворого діагностовано
E. Муцин. себорейний дерматит, пов'язаний з
дефіцитом вітаміну H (біотину).
70.У хворого діагностували Порушення активності якого з
мегалобластична анемія. Вкажіть перерахованих ферментів
з'єднання, недостатня кількість якого спостерігається у хворого?
може призводити до розвитку цієї A. * Ацетил-КоА-карбоксилаза
хвороби? B. піруватдегідрогенази.
A. * Ціанокобаламін. C. Сукцинатдегідрогеназа.
B.Гліцін. D. Карбомоілфосфатсінтетаза.
C.Медь. E . малатдегідрогеназа.
D.Холекальціферол.
E.Магній.
Біологічне окислення D. аллостеріческого регулятором.
E. Продуктом.
1. Цикл Кребса відіграє важливу роль
в реалізації глюкопластічного ефекту 5. Знешкодження ксенобіотиків і
амінокислот. Це обумовлено активних ендогенних метаболітів
обов'язковим перетворенням часто відбувається за рахунок
безазотистих залишків амінокислот в: введення в молекулу субстрату атома
A. * оксалоацетата. кисню. Вкажіть, які з перерахованих
B. Малат. процесів причетні до цього?
C. Сукцинат. A. * гідроксилювання.
D. Фумарат. B. декарбоксилюванню.
E. Цитрат. C. Переамінування.
D. Фосфорилювання.
2. Посилення перекисного окислення E. Дезамінування.
ліпідів і біополімерів є одним з
основних механізмів пошкодження 6. У пацієнта, що перебуває в зоні
структури і функції клітинних радіаційного ураження, в крові
мембран і загибелі клітини. збільшилася концентрація
Причиною цього є: малонового діальдегіду та
A. * Посилене утворення вільних гідропероксидів. Причиною цих змін
радикалів кисню та пригнічення могло послужити:
антиоксидантних систем. A. * Збільшення в організмі кисневих
B. Гіповітаміноз В1. радикалів і активація
C. Гіпервітаміноз В1. ПІДЛОГА.
D. Гіповітаміноз В12. B. Збільшення кетонових тел.
E. Гіпервітаміноз В12. C. Збільшення молочної кислоти.
D. Збільшення холестерину.
3. При отруєнні ціанідом настає E. Зменшення білків в організмі.
миттєва смерть. В чому полягає
механізм дії ціаніду на 7. Ціанід калію, що потрапив в
молекулярному рівні? організм пацієнта Б., викликав смерть
A. * Пригнічує цитохромоксидазу. через кілька хвилин на тлі явища
B. Пов'язує субстрати ЦТК. гіпоксії. Найбільш імовірною
C. Блокує сукцинатдегідрогеназу. причиною токсичної дії ціаніду було
D. Інактивує кисень. гальмування активності:
E. Пригнічує цитохром в. A. * цитохромоксидазу.
B. НАДН-дегідрогенази.
4. У експериментальних тварин з C. АТФ-синтетази.
раціону харчування виключили D. НАДФН-дегідрогенази.
ліпоєвої кислоти. При цьому у них E. АТФ-ази.
спостерігалося пригнічення
піруватдегідрогеназного комплексу. 8. При малярії призначають
Чим є ліпоєва кислота для цього препарати - структурні аналоги
ферменту? вітаміну В2 (рибофлавіну).
A. * Коферментом. Порушення синтезу яких ферментів у
B. Субстратом. плазмодія викликають ці препарати?
C. Інгібітором. A. * ФАД-залежних дегідрогеназ.
B. цитохромоксидазу. A. * Креатинфосфат.
C. пептідази. B. Креатин.
D. НАД залежних дегідрогеназ. C. Креатинін.
E. амінотрансфераз. D. Глюкозо-6-фосфат.
E. АМФ.
9. При розтині трупа 40 річний жінки
судовий експерт встановив, що 13. Ціанистий калій є отрутою, смерть
смерть настала в результаті отруєння організму настає миттєво. Назвіть, на
ціанідом. Блокування якого процесу які ферменти в мітохондріях діє
ціанідом є найбільш вірогідною ціаністий калій:
причиною смерті? A. * цитохромоксидазу [Аа3].
A. * Тканинного дихання. B. Флавіновие ферменти.
B. Розпаду глікогену. C. Цитохром В5.
C. гліколіз. D. НАД + - залежні
D. Циклу трикарбонових кислот. дегідрогенази.
E. глюконеогенезі. E. Цитохром Р-450.

10. На судово-медичну експертизу 14. Мікросомальне окислення є


надійшла кров дитини і універсальною біологічною системою
передбачуваного батька для окислення неполярних з'єднань
встановлення батьківства. Вкажіть, [багатьох лікарських засобів,
ідентифікацію яких хімічних токсичних з'єднань], стероїдних
компонентів необхідно здійснити в гормонів, холестерину. Назвіть, який
дослідній крові? цитохром входить до складу
A. * ДНК. оксігеназной ланцюга мікросом?
B. т-РНК. A. * Цитохром Р 450.
C. р-РНК. B. Цитохром а3.
D. м-РНК. C. Цитохром в.
E. ма-РНК. D. Цитохром c.
E. Цитохром а.
11. Процес синтезу АТФ, що йде
зв'язано з реакціями окислення за 15. Встановлено, що до складу
участю системи дихальних ферментів пестициду входить арсенат натрію,
мітохондрій, називається: який блокує ліпоєвої кислоти.
A. * окисного Вкажіть, активність яких ферментів
фосфорилюванням. порушується?
B. Субстратні A. * ПВК - дегідрогеназну комплексу.
фосфорилюванням. B. мікросомального окислення.
C. вільне окислення. C. метгемоглобінредуктази.
D. фотосинтетичної D. глутатіонпероксидази.
фосфорилюванням. E. глутатіонредуктаза.
E. Перекісноим окисленням.
16. До лікарні поступила робітниця
12.Для нормального метаболізму хімічного підприємства з ознаками
клітин необхідні макроергічні отруєння. У волоссі цієї жінки
сполуки. знайдено підвищена концентрація
Що відноситься до макроергів? арсенату, який блокує ліпоєвої
кислоти. Вкажіть, порушення якого 20. Хворі на алкоголізм отримують
процесу є найбільш імовірною основну масу калорій зі спиртними
причиною отруєння? напоями. У них може виникнути
A. * Окисного декарбоксилирования характерна недостатність тіаміну
ПВК. (синдром Верніке-Корсакова), при
B. мікросомального окислення. якій спостерігається порушення
C. Відновлення метгемоглобіну. функцій нервової системи, психози,
D. Відновлення органічних перекисів. втрата пам'яті. Зі зниженням
E. Знешкодження супероксидних активності якого ферменту пов'язаний
іонів. цей процес?
A. * піруватдегідрогенази.
17. Судмедексперт при розтині трупа B. Алкогольдегідрогенази.
20 - річної дівчини встановив, що C. Трасамінази.
смерть настала в результаті отруєння D. альдолаза.
ціанідом. Який фермент в E. гексокіназа.
найбільшою мірою гальмує ціанід?
A. * цитохромоксидазу. 21.Ціаніди є надзвичайно потужними
B. малатдегідрогеназа. клітинними отрутами, які при
C. гемсінтетази. надходженні в організм людини
D. Аспартатамінотрансфераза. можуть призвести до смерті.
E. карбамоілфосфатсінтетази. Блокування якого ферменту
тканинного дихання лежить в основі
18. Яка кількість молекул АТФ може такого їх дії?
синтезуватися при повному окисленні A. * цитохромоксидазу.
ацетил КоА в циклі трикарбонових B. Ферохелатази.
кислот? C. каталази.
A. * 12. D. Гемоглобінредуктази.
B. 1. E. Глюкозо-6-фосфатдегідрогенази.
C. 5.
D. 8. 22. Внаслідок дефіциту вітаміну В1
E. 3. порушується окисне
декарбоксилювання альфа-
19. Судово-медичний експерт при кетоглутаровой кислоти. Синтез
розтині трупа дівчини 20 років якого з наведених коферментів
встановив, що смерть настала порушується при цьому?
внаслідок отруєння ціанідами. A. * тіамінпірофосфат (ТПФ).
Порушення якого процесу найбільш B.Нікотінамідаденіндінуклеотіда
ймовірно було причиною смерті (НАД).
дівчини? C.Флавінаденіндінуклеотіда.
А. * Тканинного дихання. D.Ліпоевой кислоти (ЛК).
В. Синтезу гемоглобіну. E.Коензіма А.
C. Транспорту кисню гемоглобіном.
D. Синтезу сечовини. 23. При обстеженні пацієнта виявлено
E. Транспорту водню за допомогою збільшення кількості пірувату в крові
малатаспартатного механізму. і зниження активності транскетолази
в еритроцитах. Про недостатності
якого вітаміну можна судити за 27. Яка карбонова кислота,
даними біохімічних показників? проміжний продукт циклу
А. * Тіаміну. трикарбонових кислот, бере участь у
B. Ретинолу. зв'язуванні кальцію в кістковій
C. Токоферолу. тканині?
D. біотин. A. * Лимонна кислота (цитрат).
E. піродоксіна. B. Оцтова кислота (ацетат).
C .Яблочная кислота (малат).
24. При обстеженні перетворення D .Янтарная кислота (сукцинат).
харчового барвника було E .Альфа-кетоглутарова кислота
встановлено, що знищення цього (Альфа - кетоглутарат).
ксенобиотика відбувається тільки в
одну фазу - мікросомального 28. Дослідженнями останніх
окислення. Назвіть компонент цієї десятиліть встановлено, що
фази. безпосередніми "виконавцями"
A. * Цитохром Р-450. апоптозу в клітині є особливі
B. Цитохром в. ферменти - каспаз. В освіті одного з
C. Цитохром с. них бере участь цитохром С. Вкажіть
D. Цитохром а. його функцію в нормальній клітині.
E. Цитохромоксидази. A. * Фермент дихального ланцюга
перенесення електронів.
25. До лікарні доставлено хворого з B .ферментів ЦТК.
отруєнням інсектицидом - ротеноном. C .ферментів бета - окислення
Яка ділянка мітохондріальної жирних кислот.
ланцюга перенесення електронів D .Компонент Н + АТФ - азной
блокується цією речовиною? системи.
A. * НАДН-коензим Q редуктаза. E. Компонент піруватдегідрогеназноі
B. Сукцинат-коензим Q редуктаза. системи.
C. Коензим Q-цитохром с редуктаза.
D. Цитохром з оксидаза. 29. При обстеженні пацієнта виявлено
E. АТФ-синтетаза. ожиріння II ступеня. З'ясувалося, що
він любить вживати багато
26. Фармакологічні ефекти солодощів, жирну їжу, веде
антидепресантів пов'язані з малорухливий спосіб життя, тому в
блокуванням (інгібуванням) ними його організмі пріоритетним є
ферменту, який каталізує розпад анаболічний тип обміну. Які з
таких біогенних амінів, як наступних шляхів можна вважати
норадреналін, серотонін в амфіболічнімі?
мітохондріях нейронів головного A. * Цикл трикарбонових кислот.
мозку. Який фермент бере участь у B. Глюконеогенез.
цьому процесі? C. Липолиз.
A. * моноамінооксидазу. D. Гліколіз.
B. трансаміназ. E. Окислення жирних кислот.
C. декарбоксилаз.
D. пептідази. 30. При мікроскопії клітин серцевого
E. ЛіАЗ. м'яза людини знайдені органели
овальної форми, оболонка яких
утворена двома мембранами: D. Цитохром в.
зовнішня - гладка, а внутрішня E. Цитохром з1.
утворює кристи. Біохімічно
встановлено наявність ферменту 34. Центральним проміжним
АТФ-синтетази. Які органели продуктом усіх обмінів (білків,
досліджувались? ліпідів, вуглеводів) є:
A. * Мітохондрії. A. * Ацетил-КоА.
B. Центросоми. B. сукцініл-КоА.
C. Ендоплазматичний ретікулюм. C. щавлевої-оцтова кислота.
D. Лізосоми. D. Лактат.
E. Рибосоми. E. Цитрат.

31. При тиреотоксикозі підвищується 35. У хворих на алкоголізм часто


продукція тиреоїдних гормонів Т3 і спостерігається гіповітаміноз B1,
Т4, розвивається схуднення, який є наслідком порушення
тахікардія, психічна збудженість та харчування. Симптомами
інше. Як саме впливають тиреоїдні гіповітамінозу В1 є порушення
гормони на енергетичний обмін в нервової системи, психози, втрата
мітохондріях клітин? пам'яті. Чому до дефіциту вітаміну В1
A. * роз'єднує окислювання й особливо чутливі клітини нервової
фосфорилювання. тканини?
B. Активують субстратне A. * Посилюється аеробний розпад
фосфорилювання. глюкози.
C. Блокують субстратне B. Посилюється ліполіз жирової
фосфорилювання. тканини.
D. Блокують дихальний ланцюг. C. Посилюється окислення жирних
E. Активують окисне кислот.
фосфорилювання. D. Підвищується інтенсивність
гліколізу.
32. Біологічне окислення і E. Знижується інтенсивність
знешкодження ксенобіотиків гліколізу.
відбувається за рахунок
гемосодержащіхся ферментів. Який 36. Як тироксин впливає на процеси
метал є обов'язковою складовою цих тканинного дихання і окисного
ферментів? фосфорилювання у хворої
A. * Fe. тиреотоксикозом?
B. Zn. A. * Роз'єднує процес тканинного
C. Co. дихання і окисного фосфорилювання.
D. Mg. B. Блокує транспорт електронів по
E. Mn. ланцюгу цитохромів.
33. У печінці хворого порушена C. Викликає гідроліз АТФ.
детоксикація природних метаболітів і D. Пригнічує активність Фад-
ксенобіотиків. Назвіть цитохром, дегідрогеназ.
активність якого може бути знижена. E. Пригнічує активність Над-
A. * Цитохром Р-450. дегідрогеназ.
B. Цитохромоксидази ,.
C. Гемоглобін.
37. При ентеробіозі призначають C. оксалоацетата ----- Глюкоза.
акрихін - структурний аналог вітаміну D. Глюкоза ----- Рибоза-5-фосфат.
В2. Порушення синтезу яких E. Лактат ----- Піруват.
ферментів у мікроорганізмів викликає
цей препарат? 3. У жінки 45 років відсутні симптоми
A. * ФАД-залежних дегідрогеназ. діабету, але визначається натщесерце
B. цитохромоксидазу. підвищений вміст глюкози в крові
C. пептідази. (7,5 ммоль / л). Який наступний тест
D. НАД залежних дегідрогеназ ,. необхідно провести?
E. амінотрансфераз. A. * Визначення толерантності до
глюкози.
38. У дитини 6 років знижена B. Визначення ацетонових тіл в сечі.
активність, є ознаки порушення C. Визначення залишкового азоту в
координації рухів. При обстеженні крові.
встановлено генетичний дефект D. Визначення глюкози крові натще.
одного з ферментів ПВК- E. Визначення гліколізованого
дегідрогеназну комплексу. Вкажіть, гемоглобіну.
який з перерахованих нижче
лабораторних показників був 4. У хворого, виснаженого
вирішальним в обґрунтуванні голодуванням, в печінці та нирках
діагнозу? посилюється процес:
A. * Піруват вище норми. A. * глюконеогенезі.
B. Аланін нижче норми. B. Синтезу сечовини.
C. Піруват нижче норми. C. Синтезу білірубіну.
D. Аланин вище норми. D. Утворення гиппуровой кислоти.
E. Лактат нижче норми. E. Синтезу сечової кислоти.

Вуглеводи 5. Анаеробне розщеплення глюкози


до молочної кислоти регулюється
1. що надійшов з їжею глікоген відповідними ферментами. Вкажіть,
Гідролізований в шлунково- який фермент є головним
кишковому тракті. Який кінцевий регулятором цього процесу?
продукт утворився в результаті цього A. * фосфофруктокинази.
процесу? B. Глюкозо-6-фосфат ізомерази.
A. * Глюкоза. C. альдолаза.
B. Лактат. D. енолаза.
C. Лактоза. E. Лактатдегидрогеназа
D. Галактоза. 6. Еритроцити людини не містять
E. Фруктоза. мітохондрій. Який основний шлях
утворення АТФ в цих клітинах?
2. Авидин є сильним специфічним A. * Анаеробний гліколіз.
інгібітором біотіновіх ферментів. Яка B. Аеробний гліколіз.
з наведених нижче реакцій буде C. Окислювальне
блокуватися при додаванні авідіна до фосфорилирование.
клітинного гомогенату? D. Креатінкіназная реакція.
A. * Піруват ----- оксалоацетата. E. Аденілаткіназная реакція.
B. Глюкоза ----- Піруват.
7. При інтенсивній роботі в м'язах 11. Концентрація глюкози в плазмі
утворюється значна кількість аміаку. крові здорової людини знаходиться в
Яка амінокислота грає основну роль в наступних межах:
транспортуванні його в печінку і A. * 3,5-5,5 ммоль / л.
використовується в реакціях B. 2-4 ммоль / л.
глюконеогенезу? C. 10-25 ммоль / л.
A. * Аланин. D. 6-9,5 ммоль / л.
B. Аргінін. E. 1-2 ммоль / л.
C. Лізин.
D. Орнитин. 12. При дослідженні крові у хворого
E. Аспартат. виявлена виражена гіпоглікемія
натще. У біоптатах печінки знижена
8. Хвороба Гірке - це захворювання, кількість глікогену. Недостатність
при якому спостерігається якого ферменту є причиною
накопичення глікогену в печінці та захворювання?
нирках. Дефіцит якого ферменту є A. * глікогенсинтетазу.
причиною цього захворювання? B. фосфорилазу.
A. * Глюкозо-6-фосфатази. C. Фруктозодіфосфатази.
B. глікогенфосфорилази. D. піруваткарбоксілази.
C. Кінази фосфорілази. E. альдолаза.
D. фосфоглюкомутази.
E. глюкокіназа. 13. Після виконання важкої м'язової
роботи хронічний алкоголік
9. Біоенергетика мозку знепритомнів. Назвіть можливу
характеризується значною причину втрати свідомості:
залежністю від постачання киснем. A. * Гіпоглікемія.
Який субстрат окислення має B. Гіперглікемія.
найбільше значення для забезпечення C. Кетонемія.
енергією мозку? D. Азотемія.
A. * Глюкоза. E. Гіперамоніємія.
B. Жирні кислоти.
C. Кетонові тіла. 14. Яка речовина є основним
D. Гліцерол-3-Ф. джерелом енергії для мозкової
E. фосфоенолпіруват. тканини?
A. * Глюкоза.
10. При перетворенні глюкози в B. Жирні кислоти.
пентозном циклі утворюються C. Гліцерин.
фосфати різних моносахаридів. Які з D. Амінокислоти.
цих речовин можуть бути використані E. Молочна кислота.
для синтезу нуклеїнових кислот?
A. * Рібоза- 5-фосфат. 15. Активація якого процесу в
B. Рибулоза-5-фосфат. клітинах пухлини шлунка є найбільш
C. еритроза-4-фосфат. вірогідною причиною появи в
D. седогептулозо-7-фосфат. шлунковому соку молочної кислоти?
E. ксилулозо-5-фосфат. A. * Анаеробного гліколізу.
B. пентозофосфатного шляху.
C. Бета-окислення жирних кислот.
D. аеробних розщеплення глюкози. D. Хвороба Паркінсона.
E. глюконеогенезі. E. Хвороба Марк - Ардла.

16. При глікогенозі - хворобі Гірке - 20. Під час бігу на коротку дистанцію
порушується перетворення глюкозо- у нетренованих людей
6-фосфату в глюкозу, що призводить спостерігається м'язова крепатура
до накопичення глікогену в печінці та внаслідок накопичення лактату.
нирках. Дефіцит якого ферменту є Вкажіть, з посиленням якого
причиною захворювання? біохімічного процесу це може бути
A. * Глюкозо-6-фосфатази. пов'язано.
B. глікогенсинтетазу. A. * гліколізу.
C. фосфорилазу. B. глюконеогенез.
D. гексокінази. C. пентозофосфатного шляху.
E. альдолаза. D. Ліпогенез.
E. глікогенез.
17. Для синтезу полісахаридних
ланцюгів глікогену використовується 21. У хворого, який проходить курс
попередник - активна форма глюкози. лікувального голодування,
Безпосереднім донором залишків нормальний рівень глюкози в крові
глюкози в процесі синтезу глікогену підтримується, головним чином,
є: глюконеогенезом. З якої
A. * УДФ-глюкоза. амінокислоти в печінці людини
B. Глюкозо-1-Фостат. найбільш активно синтезується
C. АДФ-глюкоза. глюкоза?
D. Глюкозо-6-фосфат. A. * аланін.
E. Глюкозо-3-фосфат. B. Лізину.
C. Валина.
18. Характерною ознакою глікогенозу D. глутамінової кислоти.
є біль в м'язах під час фізичної E. лейцин.
роботи. У крові реєструється
гіпоглікемія. Вроджена недостатність 22. Дитина млявий, апатичний.
якого ферменту обумовлює цю Печінка збільшена і при її біопсії
патологію? виявлено значний надлишок
A. * глікогенфосфорилази. глікогену. Концентрація глюкози в
B. Глюкозо-6-фосфатдегідрогенази. крові нижче норми. У чому причина
C. Альфа-амілази. зниженій концентрації глюкози в
D. Гамма-амілази. крові цього хворого?
E. лізосомальної глікозідази. A. * Знижено (відсутній) активність
глікогенфосфорилази в печінці.
19. У дитини з точковою мутацією B. Знижена (відсутній) активність
генів виявлено відсутність глюкозо-6- гексокінази.
фосфатази, гіпоглікемію і C. Підвищена активність
гепатомегалию. Визначте патологію, глікогенсинтетазу.
для якої характерні ці ознаки? D. Знижена (відсутній) активність
A. * Хвороба Гірке. глюкозо-6-фосфатази.
B. Хвороба Корі. E. Дефіцит гена, який відповідає за
C. Хвороба Аддісона. синтез глюкозо-1
фосфатурідінтрансферази. 27. Під час бігу на короткі дистанції у
нетренованого людини виникає
23. У людей, які тривалий час м'язова гіпоксія. До накопичення
перебували в стадії гіподинамії, після якого метаболіту це призводить?
фізичного навантаження виникають A. * Лактату.
інтенсивні болі в м'язах. Яка B. Кетонових тел.
найбільш ймовірна причина цього? C. Ацетил-КоА.
A. * Накопичення в м'язах молочної D. Глюкозо-6-фосфату.
кислоти. E. оксалоацетата.
B. Посилений розпад м'язових білків.
C. Накопичення креатиніну в м'язах. 28. У цитоплазмі міоцитів розчинена
D. Зменшення складу ліпідів в м'язах. велика кількість метаболітів
E. Підвищення складу АДФ в м'язах. окислення глюкози. Назвіть один з
них, який безпосередньо
24. Внаслідок тривалого голодування перетворюється в лактат.
в організмі людини швидко A. * Піруват.
пропадають резерви вуглеводів. Який B. оксалоацетата.
процес метаболізму оновлює склад C. Гліцерофосфат.
глюкози в крові? D. Глюкозо-6-фосфат.
A. * Глюконеогенез. E. Фруктозо-6-фосфат.
B. Анаеробний гліколіз.
C. Аеробний гліколіз. 29. При хронічному передозуванні
D. Глікогеноліз. глюкокортикоїдів у хворого
E. Пентозофосфатний шлях. розвивається гіперглікемія. Назвіть
процес вуглецевого обміну, за
25. Під час харчування рахунок якого підвищується
новонародженої дитини молоком концентрація глюкози?
з'явилися пронос, блювота, A * Глюконеогенез.
метеоризм. Про B. Глікогеноліз.
спадковунедостатність якого C. Аеробний гліколіз.
ферменту слід думати? D. Пентозофосатний цикл.
A. * лактази. E. глікогенез.
B. мальтази.
C. Ізомерази. 30. Під час бігу на довгі дистанції
D. Олиго-1,6-глюкозидази. скелетна мускулатура тренованої
E. Пепсину. людини використовують глюкозу з
метою отримання енергії АТФ для
26. У новонародженого фізіологічна м'язового скорочення. Вкажіть
желтуха.Уровень вільного білірубіну основний процес утилізації глюкози в
значно перевищує норму. Дефіцитом цих умовах?
якого ферменту це обумовлено? A. * Аеробний гліколіз.
A. * УДФ-глюкуронілтранферази. B. Анаеробний гліколіз.
B. Трансамінази. C. Глікогеноліз.
C. ксантиноксидазу. D. Гліконеогенз.
D. аденозіндезамінази. E. глікогенез.
E. гемоксигенази.
31. У людей після тривалого D. Глюконеогенза.
фізичного навантаження виникають E. Біосинтез глікогену.
інтенсивні болі в м'язах. Що може
бути найбільш імовірною причиною 35. Під час голодування м'язові білки
цього? розпадаються до вільних амінокислот.
A. * Накопичення в м'язах молочної В який процес найбільш ймовірно
кислоти. будуть втягуватися ці сполуки в цих
B. Посилений розпад м'язових білків. умовах?
C. Накопичення креатиніну в м'язах. A. * Глюконеогенез в печінці.
D. Підвищена збудливість в м'язах. B. Глюконеогенез в м'язах.
E. Підвищення кількість АДФ в C. Синтез вищих жирних кислот.
м'язах. D. Глікогеноліз.
E. Декарбоксилирование.
32. У крові дитини виявлено високу
кількість галактози, концентрація 36. Після тривалого фізичного
глюкози знижена. Спостерігається навантаження під час заняття
катаракта, розумова відсталість, фізичною культурою у студентів
розвивається жирове переродження розвинулася м'язова крепатура.
печінки. Яке захворювання має Причиною її виникнення стало
місце? накопичення в скелетних м'язах
A. * Галактоземия. молочної кислоти. Вона утворилася в
B. Цукровий діабет. організмі студентів в результаті:
C. лактоземія. A. * гліколізу.
D. Стероїдний діабет. B. глюконеогенез.
E. Фруктоземія. C. пентозофосфатний циклу.
D. липолизу.
33. У хворого, який тривалий час E. глікогенез.
страждає на хронічний ентероколіт,
після вживання молока з'явилися 37. У новонародженого
метеоризм, діарея, коліки. З спостерігається диспепсія після
дефіцитом якого ферменту це годування молоком. При заміні
пов'язано? молока розчином глюкози симптоми
A. * лактази. диспепсії пропадають. Недостатня
B. сахараза. активність якого ферменту
C. мальтази. спостерігається у новонародженого
D. Амілази. швидше за все?
E. глікогенсінтази. A. * лактази.
B. сахараза.
34. Після відновлення кровообігу в C. мальтази.
пошкодженої тканини зупиняється D. Амілази.
накопичення лактату і зменшується E. ізомальтаза.
швидкість вживання глюкози.
Активація якого процесу обумовлені 38. У хворої дитини виявлено
ці метаболічні зрушення? затримка розумового розвитку,
A. * аеробного гліколізу. збільшення печінки, погіршення зору.
B. Анаеробного гліколізу. Доктор пов'язує ці симптоми з
C. липолизу. дефіцитом в організмі дитини
галактозо-1- 42. Після перекладу на змішане
фосфатуріділтрансферази. Який харчування у новонародженої дитини
патологічний процес спостерігається? виникла диспепсія з діареєю,
A. * Галактоземия. метеоризм, відставання в розвитку.
B. Фруктоземія. Недолік яких речовин є основою
C. Гіперглікемія. біохімічної даної патології?
D. Гіпоглікемія. A. * сахараза і ізомальтази.
E. Гіперлактатацідемія. B. лактази і целобіази.
C. трипсин і хімотрипсину.
39. При перетворенні глюкози в D. Ліпази і креатинкінази.
пентозном циклі утворюються E. целюлази і фосфофруктокинази.
фосфати різних моносахаридів. Яке з
цих речовин може бути використано 43. У регуляції активності ферментів
для синтезу нуклеїнових кислот? важливе місце належить їх
A. * Рибоза-5-фосфат. постсинтетическом ковалентного
B. рибулозо-5-фосфат. модифікації. Яким із зазначених
C. еритроза-4-фосфат. механізмів здійснюється регуляція
D. седогептулозо-7-фосфат. активності глікогенфосфорилази і
E. ксилулозо-5-фосфат. глікогенсинтетазу?
A. * фосфорилювання і
40. Хвора Л., 46 років скаржиться на дефосфорілірованіем.
сухість у роті, спрагу, часте B. Метилирование.
сечовипускання, загальну слабкість. C. ацетилювання.
При біохімічному дослідженні крові D. Обмеженим протеолізом
виявлено гіперглікемія, E. АДФ-рібозілірованіе.
гіперкетонемія. У сечі є глюкоза,
кетонові тіла. На електрокардіограмі 44. Хворий хворіє на цукровий діабет,
дифузні зміни в міокарді. У хворої: який супроводжується гіперглікемією
A. * Цукровий діабет. натще понад 7,2 ммоль / л. Рівень
B. Аліментарна гіперглікемія. якого білка плазми крові дозволяє
C. Гострий панкреатит. ретроспективно (за попередні 4-8
D. Нецукровий діабет. тижні до обстеження) оцінити рівень
E. Ішемічна хвороба серця. глікемії:
A. * Глікозильованого гемоглобіну.
41. Одним з факторів, що призводять B. Альбуміну.
до карієсу, є органічні кислоти, що C. Фібриногену.
виробляються мікроорганізмами D. С-реактивного білка.
зубного нальоту. Який з ферментів E. Церулоплазміну.
призводить до утворення молочної
кислоти? 45. Через деякий час після
A. * Лактатдегидрогеназа. інтенсивного фізичного тренування у
B. Гексокіназа. спортсмена активується
C. альдолаза. глюконеогенез, основним субстратом
D. фосфофруктокинази. якого є:
E. Алкогольдегідрогеназа. A. * Лактат.
B. Аспарагінова кислота.
C. Глутаминовая кислота.
D. Альфа-кетоглутарат. який тип модифікації білків має місце
E. Серін. при діабетичної катаракти?
A. * Гліколізірованіе.
46. Швидка допомога доставила в B. Фосфорилювання.
клініку хвору, яка втратила свідомість C. АДФ-рібозілірованіе.
на вулиці. При біохімічному аналізі D. Метилирование.
крові виявлено: концентрація E. Обмежений протеоліз.
кетонових тіл 6,8 ммоль / л,
концентрація сечовини 7,5 ммоль / л, 50. У 8-місячної дитини
збільшена концентрація С-пептиду, спостерігається блювота і діарея після
вміст кальцію 2,2 ммоль / л, глюкоза прийому фруктових соків.
крові 2,5 ммоль / л. Що привело до Навантаження фруктозою привела до
втрати свідомості пацієнтки? гіпоглікемії. Вкажіть, спадкова
A. * Гіпоглікемія. недостатність якого ферменту є
B. Порушення знешкодження аміаку. причиною стану дитини?
C. Гіпокальціємія. A. * Фруктозо -1- фосфатальдолази
D. Ацетонемія. B. фруктокінази.
E. Гіпертиреоїдизм. C. гексокіназа.
D. фосфофруктокинази.
47. При нестачі кровообігу в період E. Фруктоза-1,6-дифосфатази.
інтенсивної м'язової роботи в м'язі в
результаті анаеробного гліколізу 51. У хворого 34 років має місце
накопичується молочна кислота. Яка знижена витривалість до фізичних
її подальша доля? навантажень в той час, як в скелетних
A. * Включається в глюконеогенез в м'язах вміст глікогену підвищений.
печінці. Зниженням активності якого
B. Видаляється через нирки з сечею. ферменту це обумовлено?
C. Використовується в м'язі для A. * глікогенфосфорилази.
синтезу глюкози. B. Глюкозо-6-фосфатдегідрогенази.
D. Використовується тканинами для C. фосфофруктокинази.
синтезу кетонових тіл. D. глікогенсінтази.
E. Використовується в тканинах для E. Глюкозо-6-фосфатази.
синтезу жирних кислот.
52. При дослідженні крові у хворого
48. У слині міститься альфа-амілаза, виявлена виражена гіпоглюкоземія
здатна розщеплювати поживні натщесерце. При дослідженні
речовини. На які субстрати може біоптату печінки виявилося, що в
діяти цей фермент? клітинах печінки не відбувається
A. * Вуглеводи. синтез глікогену. Недостатність якого
B. Ліпіди. ферменту є причиною захворювання?
C. Прості білки. A. * глікогенсинтетазу.
D. Нуклеопротеїни. B. фосфорилазу.
E. хромопротеїни. C. Фруктозодіфосфатази.
D. піруваткарбоксілази.
49. У жінки 62 років розвинулася E. альдолаза.
катаракта (помутніння кришталика)
на тлі цукрового діабету. Вкажіть,
53. Хворого доставили в медичний D. Мальтози.
заклад в коматозному стані. За E. Сахарози.
словами супроводжуючих вдалося
з'ясувати, що хворий втратив 57. У хворої 38 років після прийому
свідомість під час тренування на аспірину і сульфаніламідів
завершальному етапі марафонської спостерігається посилений гемоліз
дистанції. Яку кому найімовірніше еритроцитів, який викликаний
запідозрити: недостатністю глюкозо-6-
A. * Гіпоглікемічну. фосфатдегідрогенази. Порушенням
B. гипергликемической. утворення якого коферменту
C. ацидотический. обумовлена ця патологія?
D. гіпотиреоїдного. A. * НАДФН • Н +.
E. Печінкову. B. ФАДН2.
C. пиридоксальфосфатом.
54. При якому захворюванні в сечі D. ФМНН2.
хворого з'являються кетонові тіла? E. убіхінон.
A. * При цукровому діабеті.
B. При гострому гломерулонефриті. 58. У хворого 57 років, який страждає
C. При сечокам'яної хвороби. на цукровий діабет, розвинувся
D. При туберкульозі нирки. кетоацидоз. Біохімічної основою
E. Пр інфаркті нирки. цього стану є зменшення ступеня
утилізації ацетил-КоА через брак:
55. У хворого, що проходить курс A. * оксалоацетата.
лікувального голодування, B. Альфа-кетоглутарата.
нормальний рівень глюкози в крові C. глутамат.
підтримується головним чином за D. аспартату.
рахунок глюконеогенезу. З якої E. сукцинату.
амінокислоти в печінці людини
найбільш активно синтезується 58. У хворого 57 років, який страждає
глюкоза? на цукровий діабет, розвинувся
A. * аланін. кетоацидоз. Біохімічної основою
B. Лізину. цього стану є зменшення ступеня
C. Валина. утилізації ацетил-КоА через брак:
D. глутамінової кислоти. A. * оксалоацетата.
E. лейцин. B. Альфа-кетоглутарата.
C. глутамат.
56. У хлопчика 2 років D. аспартату.
спостерігається збільшення в E. сукцинату.
розмірах печінки та селезінки,
катаракта. У крові підвищена 59. У пацієнтки з постійною
концентрація глюкози, проте тест гіпоглікемією аналіз крові після
толерантності до глюкози в нормі. введення адреналіну суттєво не
Вкажіть, спадкове порушення обміну змінився. Лікар припустив порушення
якої речовини є причиною цього? в печінці. Про зміну якої функції
A. * галактози. печінки може йти мова?
B. Фруктози. A. * Глікогендепонірующей.
C. Глюкози. B. Холестерінообразующей.
C. Кетогенной. запалі очі, ціаноз особи. Результати
D. гликолитическому. аналізів: глюкоза крові 20,1 ммоль / л
E. Екскреторної. (N = 3,3-5,5 ммоль / л), в сечі 3,5% (в
N = 0). Який можливий діагноз можна
60. Однорічна дитина відстає в поставити?
розумовому розвитку від своїх A. * Гипергликемическая кома.
однолітків. Вранці: блювота, судоми, B. Гіпоглікемічна кома.
втрата свідомості. У крові - C. Гостра серцева недостатність.
гіпоглікемія натще. З дефектом якого D. Гостре алкогольне отруєння.
ферменту це пов'язано? E. Анафілактичний шок.
A. * глікогенсінтази.
B. фосфорилазу. 64. Дитина 10 років постійно
C. аргінази. скаржиться на біль і судоми в м'язах
D. сахараза. після фізичного навантаження. При
E. лактази. обстеженні крові вміст глюкози,
лактату і креатину відповідає
61. У результаті виснажливої м'язової фізіологічній нормі. У сечі
роботи у робочого значно визначається міоглобін. У біопсійного
зменшилася буферна ємність крові. матеріалі м'язів виявлено дефіцит
Надходженням якого кислого фосфорилазу глікогену - ферменту,
речовини в кров можна пояснити це який каталізує перетворення:
явище? A. * Глікогену в глюкозо-1-фосфат.
A. * Лактату. B. Глюкозо-6-фосфату в глюкозу.
B. пірувату. C. Глюкозо-6-фосфату в глікоген.
C. 1, 3-бісфосфогліцерата. D. Глюкози в глюкозо-6-фосфат.
D. Альфа-кетоглутарата. E. "Затравки" глікогену в глікоген.
E. 3-фосфогліцерата.
65. Активація якого процесу в
62. У клініку доставлений пацієнт з пухлинних клітинах є найбільш
ознаками гострого алкогольного можливою причиною появи в шлунку
отруєння. Які зміни вуглеводного молочної кислоти?
обміну характерні для цього стану A. * Анаеробного розщеплення
A. * У печінці знижується швидкість глюкози.
глюконеогенезу. B. пентозофосфатного шляху.
B. У печінці посилюється розпад C. Бета-окислення жирних кислот.
глікогену. D. аеробних розщеплення глюкози.
C. У м'язах переважає анаеробний E. глюконеогенезі.
розпад глюкози.
D. У печінці посилюється 66. У дівчини 24 років, виснаженою
глюконеогенез. голодуванням, в печінці найбільш
E. У м'язах посилюється аеробний вірогідно посилюється:
розпад глюкози. A. * Глюконеогенез.
B. Синтез тригліцеридів.
63. Хвору Х. привезла швидка C. Освіта креатину.
допомога. Стан важкий, свідомість D. Синтез гіпуровой кислоти.
відсутня, адинамія, тахікардія, запах E. Освіта коньюгированного
ацетону з рота. Шкірні покриви сухі, білірубіну.
глюкозо-6-фосфатдегідрогенази в
67. При глікогенозі - хворобі Гірке - еритроцитах. Освіта якої речовини
порушується перетворення глюкозо- пентозофосфатного шляху буде
6-фосфату в глюкозу, що призводить порушено при цьому в найбільшій
до накопичення глікогену в печінці та мірі?
нирках. Дефіцит якого ферменту є A. * НАДФН • Н +.
причиною захворювання? B. фосфоенолпіруват.
A. * Глюкозо-6-фосфатази. C. ФАДН2.
B. глікогенсинтетазу. D. Глюкозо-6-фосфату.
C. фосфорилазу. E. діоксиацетонфосфат.
D. гексокінази.
E. альдолаза. 71. Новонароджена дитина
відмовляється від їжі, у нього
68. При обстеженні хворого в крові блювота, пронос, а згодом виникло
виявлено: глюкоза 16 мМ / Л, помутніння кришталика. При
ацетонових тіл -0,52мМ / л; в сечі: обстеженні: цукор в крові - 8,5
діурез 10л на добу, глюкоза 2%, ммоль / л, а в сечі - 1%. Який діагноз
ацетон тіла +++. Про яке найбільш вірогідний?
захворювання можна думати? A. * Галактоземия.
A. * Цукровий діабет. B. Алкаптонурия.
B. Нецукровий діабет. C. Тирозиноз.
C. Стероїдний діабет. D. Фенілкетонурія.
D. Нирковий діабет. E. Цистинурия.
E. Хвороба Іценко-Кушинга.
72. В реанимационное отделение
69. Дитина кволий, апатичний. было доставлен грудной ребенок с
Печінка збільшена і при її біопсії такими признаками: рвота, диарея с
виявлено значний надлишок нарушением роста и развития,
глікогену. Концентрація глюкози в катаракта, умственная отсталость.
крові нижче норми. У чому причина Был установлен диагноз
зниженій концентрації глюкози в галактоземия. Дефицит какого
крові цієї хворої? фермента имеет место?
A. * Знижена (відсутня) активність A. Гексоза-1-фосфатуридилилтран-
глікоген-фосфорилази в печінці. сферазы,
B. Знижена (відсутня) активність B. Глюкокиназиы.
гексокінази. C. УДФ глюкозо-4-епимеразы,
C. Підвищена активність D. УДФ-глюкозо-пирофосфори-лазы,
глікогенсинтетазу. E. Глюкозо-6-фосфатдегидрогеназы.
D. Знижена (відсутня) активність
глюкозо-6-фосфатази. 73. У людей, которые
E. Дефіцит гена, який відповідає за продолжительное время находились в
синтез глюкозо-1- состоянии гиподинамии, после
фосфатурідінтрансферази. физической нагрузки возникают
интенсивные боли в мышцах. Какая
70. У жінки 40 років виявлена наиболее возможная причина этого?
гемолітична анемія, яка обумовлена A. *Накопление в мышцах молочной
генетичним дефектом ферменту кислоты.
B. Усиленный распад мышечных печени. Какое заболевание имеет
белков. место?
C. Накопление креатинина в A. *Галактоземия.
мышцах. B. Сахарный диабет.
D. Уменьшение содержания липидов C. Лактоземия.
в мышцах. D. Стероидный диабет.
E. Повышение содержания АДФ в E. Фруктоземия.
мышцах.
78. В моче больного Б. выявлен
74. У ребенка выявили галактоземию. сахар, кетоновые тела, содержание
Концентрация глюкозы в крови глюкозы в крови составляет 10,1
существенно не изменяется. ммоль/л. Какое заболевание Вы
Дефицитом какого фермента можете предположить у больного ?
обусловленно это заболевание? A. *Сахарный диабет.
A. *Галактозо-1-фосфат- B. Атеросклероз.
уридилтрансферазы. C. Токсический гепатит.
B. Амило-1, 6-глюкозидазы. D. Панкреатит.
C. Фосфоглюкомутазы. E. Инфаркт миокарда.
D. Галактокиназы.
E. Гексокиназы. 79. Биосинтез пуринового кольца
происходит на рибозо-5-фосфате
75. У больного, изможденного путем постепенного наращивания
голоданием, в печени и почках атомов азота и углерода и замыкания
усиливается процесс: колец. Источником рибозофосфата
A. *Глюконеогенеза. служит:
B. Синтеза мочевины. A. *Пентозофосфатний цикл.
C. Синтеза билирубина. B. Гликолиз.
D. Образования гиппуровой кислоты. C. Цикл трикарбоновых кислот.
E. Синтеза мочевой кислоты. D. Глюконеогенез.
E. Гликогенолиз.
76. Анаэробное расщепление
глюкозы в молочную кислоту 80. В реанимационное отделение
регулируется соответствующими каретой скорой помощи доставлена
ферментами. Укажите, какой женщина без сознания. При
фермент является главным клиническом исследовании уро-вень
регулятором этого процесса? глюкозы в крови - 1,98 ммоль/л, Нв-
A. *Фосфофруктокиназа. 82 г/л, эритроциты - 2,1*1012 г/л, CОЭ
B. Глюкозо-6-фосфат изомераза. - 18 мм/ч, лейкоциты - 4,3*109 г/л. У
C. Альдолаза. больной вероятно:
D. Энолаза. A. *Гипогликемия.
E. Лактатдегидрогеназа. B. Сахарный диабет.
C. Галактоземия.
77. В крови выявлено высокое D. Недостаток соматотропного
содержание галактозы, концентрация гормона.
глюкозы снижена. Наблюдается E. Почечный диабет.
катаракта, жировое перерождение
81. У больного сахарным диабетом 85. У пациента 40 лет
после инъекции инсулина наступила непереносимость молочных
потеря сознания, судороги. Какой продуктов. Недостаточностью,
результат может дать биохимический какого фермента пищеварения можно
анализ крови на содержание сахара? объяснить это явление?
A. *1,5 ммоль/л. A. *Лактазы.
B. 8,0 ммоль/л. B. Лактатдегидрогеназы.
C. 10,0 ммоль/л. C. Мальтазы.
D. 3,3 ммоль/л. D. Липазы.
E. 5,5 ммоль/л. E. Амилазы.

86. При беге на короткие дистанции у


82. Вследствие продолжительного нетренированного человека возникает
голодания в организме человека мышечная гипоксия. К накоплению
быстро исчезают резервы углеводов. какого метаболита в мышцах это
Какой из процессов метаболизма приводит?
возобновляет содержание глюкозы в A. *Лактата.
крови? B. Кетоновых тел.
A. *Глюконеогенез. C. Ацетил(КoA).
B. Анаэробний гликолиз. D. Глюкозо-6-фосфата.
C. Аэробный гликолиз. E. Оксалоацетата.
D. Гликогенолиз.
E. Пентозофосфатний путь. 87. При хронической передозировке
глюкокортикоидов у больного
83. Во время питания развивается гипергликемия. Назовите
новорожденного ребенка молоком процесс углеводного обмена, за счет
матери у него появились рвота, которого увеличивается
метеоризм, понос. О наследственной концентрация глюкозы:
недостаточности какого фермента A. *Глюконеогенез.
следует думать? B. Гликогенолиз.
A. *Лактазы. C. Аэробный гликолиз.
B. Мальтазы. D. Пентозофосфатный цикл.
C. Изомеразы. E. Гликогенез.
D. Олиго-1, 6-глюкозидазы.
E. Пепсина. 88. При беге на длинные дистанции
скелетная мускулатура
84. У больного в печени наблюдается тренированного человека использует
накопление чрезмерного количества глюкозу с целью получения энергии
гликогена. Клинически это АТФ для мышечного сокращения.
проявляется: Укажите основной процесс
A. *Гипогликемией. утилизации глюкозы в этих условиях
B. Гипергликемией. A. *Аэробный гликолиз.
C. Кетонурией. B. Анаэробный гликолиз.
D. Галактоземией. C. Гликогенолиз.
E. Фруктозурией. D. Глюконеогенез.
E. Гликогенез.
89. У хворої дитини з дефіцитом освіти якого коферменту обумовлена
галактозо-1-фосфатуріділтрансферази ця патологія?
виявлена катаракта, затримка A. * НАДФН.
розумового розвитку, збільшення B .ФАДН2.
печінки. Що є причиною цього C. пиридоксальфосфатом
патологічного стану? D. ФМНН2.
A. * Галактоземия. E. убіхінон.
B. Фруктоземія.
C. Гіперглікемія. 93. У 8-ми місячної дитини
D. Гіпоглікемія. спостерігається блювота і діарея після
E. Гіперлактатацідемія. прийому фруктовихсоков.
Навантаження фруктозою привело до
90. У крові дитини виявлено високий гіпоглікемії. Вкажіть, спадкова
вміст галактози, концентрація недостатність якого ферменту є
глюкози знижена. Спостерігається причиною стану дитини?
катаракта, розумова відсталість, A. * Фруктозо -1 - фосфатальдолази.
розвивається жирове переродження B. фруктокінази.
печінки. Яке захворювання має C. Гексокіназа.
місце? D. фосфофруктокинази.
A. * Галактоземия. E. Фруктозо-1.6.-діфосфататзи.
B. Цукровий діабет.
C. лактоземія. 94.Сніженіе співвідношення
D. Стероїдний діабет. аденілових нуклеотидів АТФ / АДФ
E. Фруктоземія. призводить до посилення гліколізу в
тканинах пародонта в умовах гіпоксії.
91. У хлопчика 2 років Яка реакція при цьому активується?
спостерігається збільшення в A. * Фосфофруктокіназная.
розмірах печінки та селезінки, B. Альдолазная.
катаракта. У крові підвищена C .Тріозофосфатізомеразная.
концентрація цукру, однак тест D. Енолазная.
толерантності до глюкози в нормі. E. Лактатдегідрогеназная.
Вкажіть, спадкове порушення обміну
якої речовини є причиною цього 95.Ізвестно, що в адипоцитах жирової
стану? тканини пентозо-фосфатний шлях має
A. * Галактози. характер цікла.Какая головна функція
B. Фруктози. цього циклу в жировій тканині?
C. Глюкози. A. * Генерація НАДФН2.
D .Мальтози. B. Продукція рибоза - фосфатів.
E .Сахарози. C .Обезврежіваніе ксенобіотиків.
D. Генерація енергії.
92. У хворої 38 років після прийому E. Окислення глюкози до кінцевих
аспірину і сульфаніламідів продуктів.
спостерігається посилений гемоліз
еритроцитів, викликаний 96.Ерітроціти людини не містять
недостатністю глюкозо-6- мітохондрій. Який основний шлях
фосфатдегідрогенази.Нарушеніем утворення АТФ в цих клітинах?
A. * Анаеробний гліколіз.
B. Аеробний гліколіз. печінки. Яке з наведених речовин
C .Окіслітельно фосфорилювання. можна віднести до ліпотропних?
D .Креатінкіназна реакція. A. * Холін.
E. Аденілаткіназна реакція. B. Холестерин.
C. триацилгліцеридів.
97. У 22-річної жінки внаслідок D. Жирні кислоти.
тривалого вживання E. Рибофлавін.
сульфаніламідних препаратів
з'явилися ознаки гемолітичної анемії, 4. При жировій інфільтрації печінки
що обумовлено спадковим порушується синтез фосфоліпідів.
порушенням синтезу ферменту Вкажіть, яка з перерахованих речовин
пентозофосфатного циклу глюкозо-6- може посилювати процеси
фосфатдегідрогенази, який забезпечує метилювання в синтезі фосфоліпідів?
утворення в організмі: A. * Метіонін.
A. * НАДФ-Н2. B. Аскорбінова кислота.
B. ФМН. C. Глюкоза.
C. АТФ. D. Гліцерин.
D. НАД. E. Цитрат.
E. ФАД.
5. У крові хворих на цукровий діабет
Ліпіди спостерігається підвищення вмісту
вільних жирних кислот (НЕЖК).
1. Наявністю яких ліпідів зумовлена Причиною цього може бути:
мутність сироватки крові: A. * Підвищення активності
A. * хиломикрон. трігліцерідліпази адипоцитів.
B. Холестерину. B. Накопичення в цитозолі
C. Жирних кислот. пальмитоил-КоА.
D. Тригліцеридів. C. Активація утилізації кетонових тіл.
E. Гліцерину. D. Активація синтезу
аполипопротеинов А-1, А-2, А-4.
2. Для підвищення результатів E. Зниження активності
спортсмену рекомендували фосфатидилхолін-холестеін-
застосовувати препарат, який містить ацілтрансферази плазми крові.
карнітин. Який процес в найбільшій
мірі активується карнітином? 6. Хворому 65 років з ознаками
A. * Транспорт жирних кислот в загального ожиріння, небезпекою
мітохондрії. жирової дистрофії печінки
B. Синтез кетонових тіл. рекомендована дієта, збагачена
C. Синтез ліпідів. ліпотропні речовинами, серед яких
D. Тканинне дихання. важливе значення має вміст у
E. Синтез стероїдних гормонів. продуктах:
A. * Метіоніну.
3. У разі ненадходження або B. Холестерину.
недостатнім утворенням в організмі C. Глюкози.
людини ліпотропних факторів у неї D. Вітаміну С.
розвивається жирове переродження E. Гліцину.
7. Жирні кислоти як висококалорійні A. * Жирні кислоти.
сполуки зазнають перетворення в B. Амінокислоти.
мітохондріях, в результаті яких C. Молочна кислота.
утворюється велика кількість енергії. D. Піровиноградна кислота.
Якими шляхами проходять ці E. α-кетоглутаровую кислота.
процеси?
A. * Бета-окислення. 12. В основі ліполізу (мобілізації
B. декарбоксилюванню. жирних кислот з жирових депо)
C. Трансамінування. лежить ферментативний процес
D. Дезамінування. гідролізу жиру до жирних кислот і
E. Відновлення. гліцерину. Утворені жирні кислоти
надходять в кров і транспортуються в
8. При аналізі крові виявлено високий складі:
вміст холестерину в ліпопротеїнів A. * альбуміну.
низької щільності (ЛПНЩ). Які B. глобулін.
можливі наслідки для організму цього C. ЛПВЩ (ліпопротеїнів високої
явища? щільності).
A. * Виникнення атеросклерозу. D. ЛПНЩ (ліпопротеїнів низької
B. Цукровий діабет. щільності).
C. Ожиріння. E. хиломикрон.
D. Гіпертонія.
E. Жовтуха. 13. Хворий страждає на гіпертонію,
атеросклеротичним ураженням судин.
9. При цукровому діабеті Вкажіть, вживання якого ліпіду йому
збільшується вміст кетонових тіл в необхідно знизити в добовому
крові, що призводить до раціоні.
метаболічного ацидозу. З якої A. * Холестерину.
речовини синтезуються кетонові тіла? B. Олеїнової кислоти.
A. * Ацетил-КоА. C. Лецитину.
B. сукцініл-КоА. D. Моноолеатгліцеріда.
C. пропіоніл-КоА. E. Фосфатидилсерин.
D. Малоні-КоА.
E.-КоА. 14. У чоловіка 35 років
феохромоцитома. У крові виявлено
10. У хворого спостерігається підвищений рівень адреналіну і
кетонурія. При якому захворюванні в норадреналіну, концентрація вільних
сечі з'являються кетонові тіла? жирних кислот зросла в 11 разів.
A. * При цукровому діабеті. Вкажіть, активація якого ферменту
B. При гострому гломелуронефріте. під впливом адреналіну підвищує
C. При сечокам'яної хвороби. ліполіз?
D. При туберкульозі нирки. A. * ТАГ-ліпази.
E. При інфаркті нирки. B. ліпопротеідліпази.
C. Фосфоліпази А2.
11. Для роботи серцевого м'яза D. Фосфоліпази С.
необхідна енергія. Вкажіть, який E. Холестеролестерази.
субстрат є основним джерелом енергії
в працюючому м'язі?
15. Для серцевого м'яза характерний 19. При обстеженні хворого виявлено
аеробний характер окислення підвищений вміст в сироватці крові
субстратів. Основним з них є: ліпопротеїнів низької щільності. Яке
A. * Жирні кислоти. захворювання можна припустити у
B. триацилгліцеролів. цього хворого?
C. Гліцерол. A. * Атеросклероз.
D. Глюкоза. B. Ураження нирок.
E. Амінокислоти. C. Гострий панкреатит.
D. Гастрит.
16. При обстеженні хворого виявили E. Запалення легенів.
застій жовчі в печінці і жовчні камені
в жовчному міхурі. Вкажіть основний 20. У працівника хімчистки виявлена
компонент жовчних каменів, що жирова дистрофія печінки.
утворюються в цьому стані. Порушення синтезу якої речовини в
A. * Холестерин. печінці може привести до даної
B. Тригліцериди. патології?
C. білірубіната кальцію. A. * Фосфатидилхолін.
D. Білок. B. тристеарин.
E. Мінеральні солі. C. Сечовини.
D. фосфатидними кислоти.
17. Підвищення рівня ЛПВЩ веде до E. холевой кислоти.
зниження ризику захворювання
атеросклерозом. Який механізм 21. У чоловіка 58 років є ознаки
антиатерогенного дії ЛПВЩ? атеросклеротичного ураження
A. * Мають холестерин з тканин. серцево-судинної системи.
B. Поставляють тканин холестерин. Збільшення якого з перерахованих
C. Беруть участь у розпаді нижче показників біохімічного
холестерину. аналізу крові найбільш характерно
D. Активують перетворення для цього стану?
холестерину в жовчні кислоти. A. * Рівня ЛПНЩ.
E. Сприяють всмоктуванню B. Ерікопротеінов.
холестерину в кишечнику. C. Рівня ЛПВЩ.
D. Активності
18. В організмі людини основним аланінамінотрансферази.
місцем депонування E. Активності сукцинатдегідрогенази.
триацилгліцеролів (ТАГ) є жирова
тканина. Разом з тим їх синтез 22. Людину вкусила змія. Він починає
відбувається в гепатоцитах. У вигляді задихатися, в сечі з'являється
чого відбувається транспорт ТАГ з гемоглобін. У крові проходить
печінки в жирову тканину? гемоліз еритроцитів. Дія токсичного
A. * ЛПДНЩ. зміїної отрути призводить до:
B. хіломікронів. A. * Освіті лізолецітіна.
C. ЛПНЩ. B. ацидоз.
D. ЛПВЩ. C. Поліурія.
E. Комплексу з альбуміном. D. Розвитку алкалоза.
E. Освіті тригліцеридів.
23. У хворого хлопчика 12-ти років A. * Холін.
вміст холестерину в сироватці крові B. ДОФА.
до 25 ммоль / л. В анамнезі - спадкова C. Холестерин.
сімейна гіперхолестеринемія, D. Ацетоацетат.
причиною якої є порушення синтезу E. линолієва кислота.
білків-рецепторів:
A. * ліпопротеїнів низької щільності. 27. При обстеженні підлітка, який
B. ліпопротеїнів високої щільності. страждає ксантомаозом, виявлена
C. хиломикрон. сімейна гіперхолестеринемія.
D. ліпопротеїнів дуже низької Концентрація яких ліпопротеїнів
щільності. значно підвищена при цій патології?
E. ліпопротеїнів проміжної щільності. A. * ЛПНЩ.
B. хіломікронів.
24. Після прийому жирної їжі у C. ЛПДНЩ.
хворого з'являються нудота і печія, D. ЛПВЩ.
спостерігається стеаторея. Причиною E. НЕЖК.
цього стану може бути.
A. * Недостатність жирних кислот. 28. Пацієнт відзначає часті проноси,
B. Підвищене виділення ліпази. особливо після вживання жирної їжі,
C. Порушення синтезу трипсину. схуднення. Лабораторні дослідження
D. Недостатність амілази. показали присутність стеатореи, кал
E. Порушення синтезу фосфоліпази. гіпохоліческій. Можлива причина
такого стану:
25. Пацієнтці з високим ступенем A. * Обтурація жовчних шляхів.
ожиріння в якості харчової добавки B. Запалення слизової тонкого
рекомендований карнітин для кишечника.
поліпшення «згортання» жиру. Яку C. Недостатність ліпази.
основну участь приймає карнітин в D. Порушення активності
процесі окислення жирів? фосфоліпази.
A. * Транспорт ВШК з цитозолю в E. Незбалансована дієта.
мітохондрії.
B. Транспорт ВШК з жирових депо в 29. Чоловік 40 років пробіг 10 км за
тканини. 60 хв. Як зміниться енергетичний
C. Беруть участь в одній з реакцій обмін в м'язовій тканині?
бета-окислення ВЖК. A. * Підвищиться швидкість
D. Беруть участь в активації ВШК. окислення жирних кислот.
E. Беруть участь в активації В. Посилиться гліколіз.
внутрішньоклітинного ліполізу. C. Посилиться глюконеогенез.
D. Посилиться глікогеноліз.
26. У експериментального E. Посилиться протеоліз.
тваринного, що знаходилося на
безбілковому раціоні, розвинулася 30. Пацієнт голодує 48 годин. Які
жирова інфільтрація печінки, перш за речовини використовуються м'язовою
все, внаслідок дефіциту тканиною в якості джерела енергії в
метилирующих агентів. Назвіть цих умовах?
метаболіт, утворення якого порушено A. * Кетонові тіла.
у піддослідної тварини. B. Гліцерин.
C. Піруват. E. Всмоктування амінокислот.
D. Лактат.
E. Амінокислоти. 35. При утилізації арахідонової
кислоти по ціклоксігеназной шляху
31. Хворий після прийому жирної їжі утворюються біологічно активні
відчуває нудоту, млявість, з часом речовини. Вкажіть їх.
з'явилися ознаки стеатореї. У крові A. * Простогландин.
холестерин - 9,2 ммоль / л. Причиною B. Тироксин.
такого стану є недостатність: C. Біогенні аміни
A. * Жовчних кислот. D. Соматомідіни.
B. Тригліцеридів. E. Інсуліноподібний фактори росту
C. Жирних кислот.
D. Фосфоліпідів. 36. При емоційному стресі
E. хиломикрон. активується гормончувствітельной
трігліцерідліпази. Який вторинний
32. експерементальний тварині посередник бере участь у цьому
давали надмірна кількість глюкози з процесі?
міченим вуглецем протягом тижня. В A. * цАМФ.
якому з'єднанні можна виявити цю B. цГМФ.
мітку? C. АМФ.
A. * пальмітинової кислоти. D. Диацилглицерол.
B. метіонін. E. Іони Са2 +.
C. Вітаміні А.
D. Холін. 37. У хворої дитини при аналізі крові
E. арахідонової кислоти. встановлено гіперліпопротеїнемія, що
передалася у спадок. Генетичний
33. Для поліпшення результатів дефект синтезу якого ферменту
спортсмену рекомендували приймати обумовлює це явище?
препарат, який містить карнітин. A. * ліпопротеінліпази.
Який процес в більшій мірі B. глікозідази.
активується карнітином? C. Протеиназа.
A. * Транспорт жирних кислот в D. гемсінтетази.
мітохондрії. E. фенілаланінгідроксилази.
B. Синтез стероїдних гормонів.
C. Синтез кетонових тіл. 38. Хворому з метою попередження
D. Синтез ліпідів. жирової дистрофії печінки доктор
E. Тканинне дихання. призначив ліпотропний препарат
донор метильних груп. Це ймовірно:
34. У хворого на хронічний гепатит A. * S-Аденозілметіонін.
виявлено значне зниження синтезу і B. Холестерин.
секреції жовчних кислот. Який C. Білірубін.
процес найбільш порушений в D. Валін.
кишечнику цього хворого? E. Глюкоза.
A. * Емульгування жирів.
B. Переварювання білків. 39. Надмірне вживання вуглеводів
C. Переварювання вуглеводів. (600 г на добу), що перевищує
D. Всмоктування гліцерину. енергетичні потреби у людини 28
років буде супроводжуватися що дитина не може фіксувати погляд,
активацією: не стежить за іграшками, на очному
A. * Ліпогенез. дні відзначається симптом «вишневої
B. липолизу. кісточки». Лабораторні аналізи
C. гліколіз. показали, що в мозку, печінці і
D. глюконеогенезі. селезінці підвищений рівень
E. Бета-окислення жирних кислот. гангліозиду глікометіда. Яке
порушення спостерігається у дитини?
40. Спортсмену необхідно підвищити A. * Хвороба Тея-Сакса.
результати. Для цього йому B. Хвороба Вільсона-Коновалова.
рекомендовано вживати препарат, C. Синдром Шерешевського-Тернера.
який містить карнітин. Який процес в D. Хвороба Німана-Піка.
найбільшій мірі активується цим E. Хвороба Мак-Ардля.
з'єднанням?
A. * Транспорт жирних кислот. 44. Серед антиатеросклеротичні
B. Транспорт амінокислот. препаратів, що застосовуються з
C. Транспорт іонів Кальцію. метою профілактики і лікування
D. Транспорт глюкози. атеросклерозу є левостатін. Він діє
E. Тракспорт вітаміну К. шляхом:
A. * Гальмування біосинтезу
41. У клініку потрапила дитина 1-го холистерина.
року з ознаками ураження м'язів B. Пригнічення всмоктування
кінцівок і тулуба. Після огляду холестерину в кишечнику.
виявлено дефіцит карніціна в м'язах. C. Активація метаболізму
Біохімічної основою цієї патології є холестерину.
порушення процесу: D. Стимуляція екскреції холестерину
A. * Транспорт жирних кислот в з організму.
мітохондріях. E. Всіма проведеними шляхами.
B. Регуляція рівня Са2 + в
мітохондріях.
C. субстратне фосфорилювання. 45. У пацієнта, який перебував в зоні
D. Утилізація молочної кислоти. радіаційного ураження, в крові
E. Окисного фосфорилювання. збільшилася концентрація
малонового діальдегіду,
42. У хворих, які страждають важкою гідропероксидів. Причиною цих змін
формою діабету і не отримують може бути:
інсулін спостерігається метаболічний A. * Збільшення в організмі кисневих
ацидоз. Підвищенням концентрацій радикалів і активація перекисного
яких метаболітів це обумовлює? окислення ліпідів.
A. * Кетонових тел. B. Збільшення кетонових тел.
B. Жирних кислот. C. Збільшення молочної кислоти.
C. Ненасичених жирних кислот. D. Збільшення холестерину.
D. триацилгліцеролів. E. Зменшення білків крові.
E. Холістерола.
46. Після прийому жирної їжі хворий
43. До лікарні потрапила дитина 6 відчуває дискомфорт, в калі є
років. При обстеженні було виявлено, неперетравлені краплі жиру. Реакція
сечі на жовчні кислоти позитивна. A. * Атеросклероз.
Причиною такого стану є нестача: B. Ураження нирок.
A. * Жовчних кислот. C. Гострий панкреатит.
B. Жирних кислот. D. Гастрит.
C. хиломикрон. E. Запалення легенів.
D. Тригліцеридів.
E. Фосфоліпідів. 52. У чоловіка 58 років з'явилися
ознаки атеросклеротичного ураження
47. Для серцевого м'яза характерним є серцево-судинної системи.
аеробний характер окислення Збільшення якого з перерахованих
субстратів. Основним з них є: нижче показників біохімічного
A. * Жирні кислоти. аналізу крові найбільш характерно
B. триацилгліцеролів. для цього стану?
C. Гліцерол. A. * Рівня ЛПНЩ.
D. Глюкоза. B. Рівня ЛПДНЩ.
E. Амінокислоти. C. Рівня ЛПВЩ.
D. Активності
49. При обстеженні хворого виявили аланінамінотрансферази.
застій жовчі в печінці і жовчні камені E. Активності сукцинатдегідрогенази.
в жовчному міхурі. Вкажіть основний
компонент жовчних каменів, які 53. У юнака 20 років з
утворюються в цьому стані макроцитарною анемією в сечі
A. * Холестерин. підвищений кількість
B. Тригліцериди. метилмалоновой кислоти, що в першу
C. білірубіната кальцію. чергу обумовлено дефіцитом:
D. Білок. A. * Ціанкобаламін.
E. Мінеральні солі. B. Нікотинової кислоти.
C. пантотенова кислота.
50. У клініку потрапила дитина, 1 рік, D. Аскорбінової кислоти.
з ознаками ураження м'язів. Після E. біотин.
обстеження виявлено дефіцит
карнітину в м'язах. Біохімічної 54. При цукровому діабеті глюкоза
основою цієї патології є порушення погано засвоюється тканинами, в
процесу: крові значно збільшується рівень
A. * Транспорту жирних кислот в інших низькомолекулярних речовин,
мітохондріях. які використовуються деякими
B. Регулювання рівня Ca2 + в органами і тканинами як
мітохондріях. енергетичний матеріал. Це в першу
C. субстратне фосфорилювання. чергу:
D. утилізації молочної кислоти. A. * Кетонові тіла.
E. Синтезу актину і міозину. B. Молочна кислота.
C. Піровиноградна кислота.
51. При обстеженні хворий виявлено D. Альфа-кетоглутарова кислота.
підвищений вміст в сироватці крові E. Оксалат.
ліпопротеїнів низької щільності. Яке
захворювання можна передбачити у 55. Кал хворого містить багато
цього хворого? нерасщепленного жиру і має
сірувато-білий колір. Вкажіть субстрат є основним джерелом енергії
причину цього явища в працюючому м'язі?
A. * Обтурація жовчної протоки. A. * Жирні кислоти.
B. Недостатня активація пепсину B. Амінокислоти.
соляною кислотою. C. Молочна кислота.
C. Гіповітаміноз Е. D. Піровиноградна кислота.
D. Ентерит. E. А-кетоглутарова кислота.
E. Роздратування епітелію.
60. Недостатня секреція якого
56. Скарги і об'єктивні дані дають ферменту зумовлює неповне
можливість припустити у хворого перетравлювання жирів в шлунково-
запальний процес в жовчному міхурі, кишковому тракті і поява великої
порушення колоїдних властивостей кількості нейтральних жирів в
жовчі, ймовірність утворення калових масах?
жовчних каменів. Що може головним A. * панкреатичної ліпази.
чином викликати їх освіту? B. Фосфоліпази.
A. * Холестерин. C. Ентерокінази.
B. Урати. D. Амілази.
C. Оксалати. E. Пепсину.
D. Хлориди.
E. Фосфати. 61. При обстеженні в клініці у жінки
встановлена недостатність активності
57. При дослідженні плазми крові ліпопротеінліпази, яка гідролізує
пацієнта через 4 години після тригліцериди хіломікронів на
прийому ним жирної їжі встановлено, поверхні ендотелію капілярів жирової
що вона є каламутною. Найбільш тканини. Які біохімічні порушення
імовірною причиною цього стану є слід очікувати?
підвищення концентрації в плазмі: A. * Гіперліпопротеїнемія - I типу.
A. * хиломикрон. B .Гіперліпопротеінемія - ПА типу.
B. ЛПВЩ. C. Гіперліпопротеїнемія - III типу.
C. ЛПНЩ. D. Гіперліпопротеїнемія - IV типу.
D. Холестерину. E. Гіперліпопротеїнемія - ІІБ типу.
E. Фосфоліпідів.
62. Стеатоз виникає внаслідок
58. Під час аналізу крові виявлено накопичення триацилгліцеролів в
високий вміст холестерину в бета- гепатоцитах. одним з механізмів
ліпопро-теіновой фракції. Які розвитку цього захворювання є
можливі наслідки для організму цього зменшення утилізації нейтрального
явища? жиру ЛПДНЩ. Які ліпотропні
A. * Виникнення атеросклерозу. речовини запобігають розвитку
B. Цукрового діабету. стеатозу?
C. Ожиріння. A. * Метіонін, В6, В12.
D. Гіпертонія. B .Аргінін, В2, В3.
E. Жовтуха. C. Аланин, В1, РР.
D. Валін, В3, В2.
59. Для роботи серцевого м'яза E. Ізолейцин, В1, В2.
необхідна енергія. Вкажіть, який
63. У діабетичного хворого 60 років в його знешкодження в нервовій
крові виявлено кетоацидоз. тканині.
біохімічним наслідком кетоацидозу A. * Синтез глутаміну.
при цукровому діабеті є зменшення B. Синтез солей амонію.
рівня використання ацетил-ко - А C. Синтез сечовини.
внаслідок дефіциту: D. Трансамінірованніе.
A. * Оксалоацетата. E. Освіта парних з'єднань.
B .Сукціната.
C. аспартату. 4. Кухар в результаті необачності
D. Ацетату. обпалив руку парою. Підвищення
E .Аланіна. концентрації якої речовини
викликало почервоніння, набряклість
64. Пацієнту з метою підтримки і болючість ураженої ділянки шкіри?
роботи печінки був презначен A. * Гістаміну.
препарат, який містить метіонін. B. Тіаміну.
Завдяки цьому може бути забезпечено C. Глутаміну.
синтез: D. Лізину.
A. * Фосфатидилхолін. E. галактозамін.
B .Фосфатіділсеріна.
C .Цітрата. 5. Хворому з прогресуючою м'язовою
D. Лактат. дистрофією було проведено
E .Пірувата. біохімічне дослідження сечі. Поява
якої речовини у великій кількості в
сечі може підтвердити захворювання
амінокислоти м'язів у даного хворого?
A. * Креатину.
1. Вкажіть, яке із з'єднань є B. Порфірини.
акцептором аміногруп в реакціях C. Сечовини.
трансамінування амінокислот: D. гиппуровую кислоти.
A. * α -кетоглутарат. E. Креатиніну.
B. аргініносукцінат.
C. Лактат. 6. При декарбоксилюванні
D. цітрулін. амінокислоти гістидину утворюється
E. Орнитин. надзвичайно активний амін - медіатор
запалення і алергії, а саме:
2. Які компоненти фракції A. * Гістамін.
залишкового азоту переважають в B. Серотонін.
крові при продукційних Азотемія: C. Дофамин.
A. * Амінокислоти, сечовина. D. γ-аміномасляна кислота.
B. Ліпіди, вуглеводи. E. триптаміни.
C. Кетонові тіла, білки.
D. Порфірини, білірубін. 7. Одна з форм вродженої патології
E. Сечова кислота, холін. супроводжується гальмуванням
перетворення фенілаланіну в тирозин.
3. Аміак особливо токсичний для Біохімічної ознакою хвороби є
ЦНС людини. Вкажіть головний шлях накопичення в організмі деяких
органічних кислот, в тому числі для діагностики пошкоджень
кислоти: внутрішніх органів. Кофактором цих
A. * фенілпіровиноградна. ферментів є активна форма вітаміну:
B. Лимонної. A. * В6.
C. Піровиноградна. B. В1.
D. Молочної. C. В12.
E. глутамінової. D. В2.
E. РР.
8. Карбомоілфосфатсінтетаза
каталізує реакцію утворення 12. Під дією опромінення
карбомоілфосфата з вільного аміаку. ультрафіолетовими променями у
Цей фермент постачає людини темніє шкіра, що є захисною
карбомоілфосфат для синтезу: реакцією організму. Яке захисна
A. * Сечовини. речовина - похідне амінокислот -
B. Пуринів. синтезується в клітинах під впливом
C. Креатину. ультрафіолету?
D. Ліпідів. A. * Меланин.
E. Аминокислот. B. Аргінін.
C. Метіонін.
9. У організмі людини є пептид, в D. Фенілаланін.
утворенні якого бере участь гамма- E. Тироксин.
карбоксильна група глутамінової
кислоти. Цей пептид називається: 13. При декарбоксилюванні
A. * глутатіону. амінокислот утворюється ряд
B. карнозином. біологічно активних речовин. Вкажіть
C. Ансеріном. одне з них.
D. Окситоцин. A. * ГАМК.
E. вазопресин. B. оксалоацетата.
C. Глутамин.
10. Центральну роль в обміні D. Глутатіон.
амінокислот в нервової тканини E. α-кетоглутаратом.
відіграє глутамінова кислота. Це
пов'язано з тим, що дана 14. У вогнищі запалення утворюється
амінокислота: біогенний амін, що володіє
A. * Зв'язує аміак з утворенням судинорозширювальну дію. Назвіть
глутаміну. його.
B. Використовується для синтезу A. * Гістамін.
ліків. B. Серотонін.
C. Використовується для синтезу C. ДОФА.
глюкози. D. триптаміни.
D. Використовується для синтезу E. ГАМК.
нейроспецифических білків.
E. Використовується для синтезу 15. До лікарні потрапила дитина 7
ацетонових тіл. років в стані алергічного шоку, який
розвинувся після укусу оси. У крові
11. Визначення активності підвищена концентрація гістаміну. В
трансаміназ широко застосовується
результаті якої реакції утворюється E. Дофамін.
цей амін?
A. * декарбоксилюванню. 20. У новонародженого на 6 день
B. дегідрування. життя в сечі виявлено надлишок
C. Дезамінування. фенілпірувата і фенілацетат. Обмін
D. Гідроокісленія. якої амінокислоти порушено в
E. Відновлення. організмі дитини?
A. * Феніаланіна.
16. У хворого виявлена серповидно- B. Триптофану.
клітинна анемія. Заміна якої C. Метіоніну.
амінокислоти в поліпептидному D. Гістидину.
ланцюгу Hb на валін призводить до E. Аргініну.
цього захворювання?
A. * глутамінової кислоти. 21. Альбіноси погано переносять
B. аспарагінова кислоти. сонячну засмагу, з'являються опіки.
C. Лецину. Порушення метаболізму якої
D. Аргініну. амінокислоти лежить в основі цього
E. треоніну. явища?
A. * фенілаланіну.
17. Катіонні глікопротеїни є B. Метіоніну.
основними компонентами слини C. Триптофану.
привушних залоз. Які амінокислоти D. глутамінової кислоти.
обумовлюють їх позитивний заряд? E. Гістидину.
A. * Лізин, аргінін, гістидин.
B. Аспартат, глутамат, гліцин. 22. У хворого явна прогресуюча
C. Аспартат, аргінін, глутамат. м'язова дистрофія. Назвіть показник
D. Глутамат, валін, лейцин. обміну азоту сечі, характерний для
E. Цистеин, гліцин, пролін. такого стану?
A. * Креатин.
18. В організмі людини амінокислоти B. амонійного солі.
дезамінуются шляхом C. Креатинін.
трансамінування, в результаті чого D. Сечова кислота.
аминогруппа переноситься на: E. Сечовина.
A. * α -кетоглутарат.
B. Сукцинат. 23. При алкаптонуріі в сечі хворого
C. Цитрат. знайдено велику кількість
D. Фумарат. гомогентизиновой кислоти (сеча
E. Малат. темніє на повітрі). Природжений
дефект якого ферменту має місце?
19. При декарбоксилюванні A. * Оксидази гомогентизиновой
амінокислот утворюються біогенні кислоти.
аміни. Назвіть, який з них відноситься B. Аланінамінотрансфераза.
до медіаторів запалення? C. тирозинази.
A. * Гістамін. D. Фенілаланін-4-монооксигенази.
B. Адреналін. E. Тірозінамінотрансферази.
C. ГАМК.
D. Таурин.
24. У новонародженого 28. У дитини в крові підвищена
спостерігаються епілептиформні кількість фенилпировиноградной
судоми, викликані дефіцитом кислоти. Який вид лікування потрібно
вітаміну В6. Це викликано при фенілкетонеміі?
зменшенням в нервовій тканині A. * Дієтотерапія.
гальмівного медіатора - гамма- B. Вітамінотерапія.
аміномасляної кислоти. Активність C. Ферментотерапія.
якого ферменту знижена? D. Антибактеріальна терапія.
A. * глутаматдекарбоксилази. E. Гормонотерапія.
B. Аланінамінотрансфераза.
C. глутаматдегідрогеназа. 29. Мати помітила дуже темну сечу в
D. пірідоксалькінази. її 5-річну дитину. Дитина скарг
E. Глутаматсінтетази. ніяких не висловлює. Жовчних
пігментів в сечі не виявлено.
25. У психіатрії для лікування ряду Поставлено діагноз алкаптонурія.
захворювань ЦНС використовують Дефіцит якого ферменту має місце?
біогенні аміни. Вкажіть препарат тієї A. * Оксидази гомогентизиновой
групи, який є медіатором кислоти.
гальмування. B. фенілаланінгідроксилази.
A. * Гамма-аміномасляна кислота. C. тирозинази.
B. Гістамін. D. Оксидази оксіфенілпірувата.
C. Серотонін. E. декарбоксилаз фенілпірувата.
D. Дофамін.
E. Таурин. 30. Біогенні аміни: гістамін,
серотонін, дофамін і інші - дуже
26. Наявність білка в розчині можна активні речовини, впливають на різні
виявити за допомогою кольорових фізіологічні функції організму. В
реакцій. Яка з нижчеперелічених результаті якого процесу
реакцій дасть негативний результат утворюються біогенні аміни в
при повному гідролізі білка? тканинах організму?
A. * Біуретова. A. * декарбоксилюванню
B. Нінгідринова. амінокислот.
C. Ксантопротеиновая. B. Дезамінування амінокислот.
D. Фоля. C. Трансамінування амінокислот.
E. Сакагучи. D. Окислення амінокислот.
E. Відновного реамінірованія.
27. У дитини грудного віку виділена
сеча темніє на повітрі. У крові і сечі 31. У новонародженого
виявили гомогентизинової кислоту. спостерігалися судоми, які проходили
Який найбільш ймовірний симптом? після призначення вітаміну B6. Цей
A. * Алкаптонурия. ефект найбільш ймовірно викликаний
B. Альбінізм. тим, що вітамін B6 бере участь в
C. Цистинурия. утворенні:
D. Порфирія. A. * Гамма-аміномасляної кислоти
E. Гемолітична анемія. (ГАМК).
B. Замінімих амінокислот.
C. Гема.
D. Гістаміну. D. Аспартатамінотрансфераза.
E. нікотинамід. E. УДФ-глюкуронілтрансферази.

32. метильних груп (- СН 3) 36. У хворого з черепно-мозковою


використовуються в організмі для травмою спостерігаються
синтезу таких важливих сполук, як епілептиформні сдорожние напади,
креатин, холін, адреналін. Джерелом які регулярно повторюються. Освіта
цих груп є одна з незамінних якого біогенного аміну порушено при
амінокислот, а саме: цьому стані?
A. * Метіонін. A. * ГАМК.
B. Валін. B. Гістаміну.
C. Лейцин. C. Адреналіну.
D. Изолейцин. D. Серотоніну.
E. Триптофан. E. Дофаміну.

33. При декарбоксилюванні глутамату 37. Аміак є дуже отруйною


в ЦНС утворюється медіатор речовиною, особливо для нервової
гальмування. Назвіть його. системи. Яка речовина бере особливо
A. * ГАМК. активну участь у знешкодженні
B. Глутатіон. аміаку в тканинах мозку?
C. Гістамін. A. * Глутаминовая кислота.
D. Серотонін. B. Лізин.
E. Аспарагин. C. Пролин.
D. Гистидин.
34. У крові хворого на рак сечового E. Алонін.
міхура знайдено високий рівень
серотоніну і оксіантраніловой 38. Хворий 13 років скаржиться на
кислоти. З надлишком надходження в загальну слабкість, запаморочення,
організм якої амінокислоти це стомлюваність. Спостерігається
пов'язано? відсталість в розумовому розвитку.
A. * Триптофану. При огляді виявлено високу
B. Аланін. концентрацію валіну, ізолейцину,
C. Гістидину. лейцину в крові та сечі. Сеча
D. Метіоніну. специфічного запаху. Що може бути
E. Тирозину. причиною такого стану?
A. * Хвороба клинового сиропу.
35. Немовля відмовляється від B. Хвороба Аддісона.
грудного харчування, його дихання C. Тирозиноз.
не ритмічне, сеча має спечіфіческій D. Гістидинемія.
запах «пивний закваски» або E. Базедова хвороба.
«клинового сиропу». Природжений
дефект якого ферменту викликав дану 39. У хворого з діагнозом «злоякісний
патологію? карціноід» різко підвищений вміст
A. * Дегідрогенази розгалужених серотоніну в крові. Виберіть
альфа-кетокислот. амінокислоту з яких може утворитися
B. Глюкозо-6-фосфатдегідрогенази. цей біогенний амін.
C. Гліцеролкінази. A. * Триптофан.
B. Аланін. трихлороцтового заліза з'являється
C. Лейцин. оливково-зелене забарвлення. Для
D. Тіреонін. якої патології обміну амінокислот
E. Метіонін. характерні ці зміни?
A. * Фенілкетонурії.
40. У хворої дитини з уповільненим B. Алкаптонурія.
розумовим і фізичним розвитком, C. Тирозиноз.
який страждає частими рвотами після D. Альбінізм.
прийому їжі, в сечі виявлено E. ксантинурія.
фенилпировиноградная кислота.
Наслідком порушення якого обміну є 44. У 12-річного хлопчика в сечі
ця патологія? виявлено високий вміст усіх
A. * Обміну амінокислот амінокислот аліфатичного ряду. При
B. Ліпідного обмілити. цьому відзначено найбільш висока
C. Вуглеводного обмілити. екскреція цистину і цистеїну. Крім
D. Водно-сольового обмілити. того, УЗД нирок показало присутність
E. Фосфорно-кальцієвого обміну. каменів в них. Виберіть можливу
патологію.
41. У новонародженої A. * Цистинурия.
спостерігаються судоми, які B. Алкаптонурия.
проходили після призначення C. Цистит.
вітаміну В6. Цей ефект найбільш D. Фенілкетонурія.
ймовірно викликаний тим, що вітамін E. Хвороба Хартнупа.
В6 входить до складу ферменту:
A. * Глютаматдекарбоксілази. 45. До лікаря звернувся пацієнт зі
B. Піруватдегідратази. скаргами на непереносимість
C. Альфа-кетоглутаратдегидрогенази. сонячної радіації. Спостерігаються
D. Амінолевулінатсінтази. опіки шкіри і порушення зору.
E. глікогенфосфорилази. Попередній діагноз альбінізм.
Порушення обміну якої амінокислоти
42. У новонародженого на пелюшках відзначається у цього пацієнта?
виявлені темні плями, що свідчить A. * Тирозину.
про утворення гомогентінзіновой B. Пролін.
кислоти. З порушенням обміну якої C. Лізину.
речовини це пов'язано? D. аланін.
A. * Тирозину. E. Триптофан.
B.Галактози.
C.Метіоніна. 46. До лікарні швидкої допомоги
D.Холестеріна. доставили дитину 7 років в стані
E. Триптофану. алергічного шоку, який розвинувся
після укусу оси. У крові підвищена
43. У дитини 1,5 років спостерігається концентрація гістаміну. В результаті
відставання розумового і фізичного якої реакції утворюється цей амін?
розвитку, посветление шкіри і A. * декарбоксилюванню.
волосся, зниження вмісту в крові B. Гідрооксілірованія.
катехоломінов. При додаванні до C. дегидрированием.
свіжої сечі декількох крапель 5% D. Дезамінування.
E. Відновлення. 50. При укусі отруйної змії у людини
може розвинутися гемолітична
47. Основна маса азоту в організмі жовтяниця. Вкажіть показник плазми
виводиться у вигляді сечовини. крові, який зростає у потерпілого в
Зниження активності якого ферменту першу чергу
в печінці призводить до гальмування A. * Білірубін непрямий
синтезу сечовини і нагромадження (некон'югований).
аміаку в крові і тканинах? B. Білірубін прямий (кон'югований).
A. * Карбамоілфосфатсінтази. C. Сечова кислота.
B. Аспартатамінотрансфераза. D. Сечовина.
C. Уреаза. E. Вільні амінокислоти.
D. Амілази.
E. Пепсину. 51. У чоловіка 32 років діагностовано
гостра променева хвороба.
48. У дитини 6 міс. спостерігається Лабораторно встановлено різке
різке відставання в психомоторному зниження рівня серотоніну в
розвитку, бліда шкіра з тромбоцитах. Найімовірнішою
екзематозними змінами, біле волосся, причиною зниження
блакитні очі, напади судом. Який з тромбоцитарного серотоніну є
наступних лабораторних аналізів порушення процесу
крові і сечі найімовірніше дозволить декарбоксилювання:
встановити діагноз? A. * 5-оксітріптофана.
B. Серіна.
A. * Визначення концентрації C. Тирозину.
фенілпірувата. D. піровиноградної кислоти.
B. Визначення концентрації E. Гістидину.
трептофана.
C. Визначення концентрації 52. У дитини 3 років після
гістидину. перенесеної важкої вірусної інфекції
D. Визначення концентрації лейцину. відзначається блювота, яка
E. Визначення концентрації валіну. повторюється, втрата свідомості,
судоми. При обстеженні крові
49. Дитина 10-місячного віку, батьки виявлено гіперамоніємія. З чим
якого брюнети, має світле волосся, можуть бути пов'язані зміни
дуже світлу шкіру і блакитні очі. біохімічних показників крові у цієї
Зовні при народженні виглядав дитини?
нормально, але протягом останніх 3-х
місяців спостерігалися порушення A. * З порушенням знешкодження
мозкового кровообігу, відсталість в аміаку в арнітіновом циклі.
розумовому розвитку. Причиною B. З активацією процесів
такого стану може бути: декарбоксилювання амінокислот.
A. * Фенилкетонурия. C. З порушенням знешкодження
B. Галактоземия. біогенних амінів.
C. Глікогеноз. D. З посиленням гниття білків в
D. Гостра порфірія. кишечнику.
E. Гістидинемія. E. З пригніченням активності
ферментів трансамінування.
53. У дитини, яка народилася 2 дні 57. У хлопчика 4 років після
тому, спостерігається жовте перенесеного важкого вірусного
забарвлення шкіри і слизових гепатиту спостерігається блювота,
оболонок. Причиною такого стану є втрата свідомості судоми. У крові -
тимчасовий недолік ферменту: гипераммониемия. Порушення якого
A. УДФ- глюкуронілтрансферази. біохімічного процесу викликало
B. Сульфотрансферази. подібний стан хворого?
C. гемсінтетази. A. * Порушення знешкодження
D. гемоксигенази. аміаку в печінці.
E. Білівердінредуктази. B. Порушення знешкодження
бігенних амінів.
54. У пацієнта цироз печінки. C. Посилення гниття білків в
Дослідження якого з перерахованих кишечнику.
речовин, що виводяться з сечею, D. Активація декарбоксилирования
може характеризувати стан амінокіслот.
антитоксичної функції печінки? E. Пригнічення ферментів
A. * Гіпуровой кислоти. трасамінірованія.
B. амонійного солей.
C. Креатиніну. 58. Травма мозку викликана
D. Січовий кислоти. підвищенням утворенням аміаку. Яка
E. Аминокислот. амінокислота бере участь у видаленні
аміаку з цієї тканини?
55. Вітамін А в комплексі з A. * Глутаминовая кислота.
спецефічними циторецепторів B. Тирозин.
проникають через ядерну мембрану, C. Валін.
індукує процеси транскрипції, що D. Триптофан.
стимулює ріст і диференціювання E. Лізин.
клітин. Ця біологічна функція
реалізується наступною формою 59. Педіатр при огляді дитини
вітаміну А: зазначив відставання у фізичному і
A. * Транс-ретиноевая кислота. розумовому розвитку. Аналіз сечі
B. Транс-ретиналь. показав, що різке підвищення вмісту
C. Цис-ретиналь. кетокислот, дає кольорове
D. Ретинол. забарвлення з хлорним залізом. Яке
E. Каротин. порушення обміну речовин виявлено?
A. * Фенилкетонурия.
56. У юнака 19 років явні ознаки B. Алкаптонурия.
депігментації шкіри, обумовлені C. Тирозинемия.
порушенням синтезу меланіну. D. Цистинурия.
Вкажіть, порушенням обміну якої E. Альбінізм.
амінокислоти це викликано?
A. * Тирозину. 60. У ході катаболізму гістидину
B. Триптофану. утворюється біогенний аміак, який
C. Гістидину. володіє потужним
D. пролін. судинорозширювальну дію. Назвіть
E. Гліцину. його.
A. * Гістамін. 64. При аналізі сечі 3-х міс. дитини
B. Серотонін. виявлено підвищену кількість
C. ДОФА. гомогентизиновой кислоти, сеча при
D. Норадреналін стоянні на повітрі набуває темне
E. Дофамин. забарвлення. Для якого з нижче
перерахованих захворювань
61. У хворого діагностовано характерні описані зміни?
алкаптонурія. Вкажіть фермент, A. * Алкаптонурія.
дефект якого є причиною цієї B. Фенілкетонурії.
патології. C. альбінізм.
A. * оксидазой гомогентінзіновой D. аміноацидурія.
кислоти. E. цістінуріей.
B. Фінілаланінгідроксілаза.
C. глутаматдегідрогеназа. 65. За клінічними показниками
D. піруватдегідрогенази. хворому призначений
E. ДОФА-декарбоксилази. пиридоксальфосфат. Для корекції
яких процесів рекомендується цей
62. У клініку госпіталізовано хворого препарат?
з діагнозом карциноїд кишечника. A. * Трансамінування і
Аналіз виявив підвищене вироблення декарбоксилювання амінокислот.
серотоніну. Відомо, що ця речовина B. Окисного декарбокслірованія
утворюється з амінокислоти амінокислот.
триптофану. Який біохімічний C. Дезамінування пуринових
механізм лежить в основі цього нуклеотидів.
процесу? D. Синтез пуринових і піримідинових
A. * Декарбоксилирование. основ.
B. Дезамінування. E. Синтезу білка.
C. Мікросомальне окислення
D. Трансамінування. 66. У пацієнта з тривалим
E. Освіта парних з'єднань. епілептичним нападом у вогнищі
збудження внаслідок розпаду
63. При повторному дії біогенних амінів постійно
ультрафіолетових променів шкіра утворюється аміак. Знешкодження
темніє внаслідок синтезу в ній цієї речовини в головному мозку
меланіну, що захищає клітину від здійснюється за участю:
пошкоджень. Основним механізмом A. * глутамінової кислоти.
включення цього захисту є: B. Січовий кислоти.
A. * Активація тирозинази. C. Аміномасленной кислоти.
B. Пригнічення тирозинази. D. Серіна.
C. Активація оксидази E. ліпоєвоїкислоти.
гомогентизиновой кислоти.
D. Пригнічення оксидази 67. У людини відчуття страху
гомогентизиновой кислоти. викликається синтезом в лімбічної
E. Пригнічення системи мозку диоксифенилаланина
фенілаланінгідроксилази. (ДОФА). З якої речовини йде його
синтез?
A. * Тирозину.
B. глутамінової кислоти. декарбоксилювання глутамінової
C. Триптофану. кислоти. Назвіть його.
D. Лізину. A. * ГАМК
E. 5-оксітріптофана. B. Палфі.
C. ТДФ.
68. Депресії, емоційні розлади є D. АТФ.
наслідком нестачі в головному мозку E. ТГФК.
норадреналіну, серотоніну та інших
біогенних амінів. Збільшення їх 72. При лабораторному дослідженні у
вмісту в синапсах можна досягти за дитини виявлено підвищений вміст в
рахунок антидепресантів, які крові і сечі лейцину, валіну,
гальмують фермент: ізолейцину і їх кетопроізводних. Сеча
A. * моноамінооксидазу. мала характерний запах клинового
B. діамінооксідази. сиропу. Недостатність якого
C. оксидази L-амінокислот. ферменту характерна для цього
D. оксидазой Д-амінокислот. захворювання?
E. Фенілаланін-4-монооксигенази. A. * Дегідрогенази розгалужених
амінокислот.
69. При тестуванні на гіперчутливість B. Амінотрансферази.
пацієнту під шкіру ввели алерген, C. Глюкоза-6-фосфатази.
після чого спостерігалося D. Фосфофруктакінази.
почервоніння, набряк, біль внаслідок E. Фосфофруктомутази.
дії гістаміну. В результаті якого
перетворення амінокислоти гістидину 73. Дитина 9-ти міс. харчується
утворюється цей біогенний амін? синтетичними сумішами, що не
A. * декарбоксилюванню. збалансованими за вмістом вітаміну
B. Метилювання. В6. У дитини спостерігається
C. Фосфорилювання. пілагроподобний дерматит, судом,
D. ізомеризації. анемія. Розвиток судом може бути
E. Дезамінування. пов'язано з дефіцитом освіти:
A. * ГАМК.
70. Хворому з передбачуваним B. Гістаміну.
діагнозом «прогресуюча м'язова C. Серотоніну.
дистрофія» був сделат аналіз сечі. Яке D. ДОФА.
з'єднання в сечі підтверджує діагноз E. Дофаміну.
пацієнта?
A. * Креатин. 74. У новонародженої дитини
B. Колаген. спостерігаються зниження
C. Порфірини. інтенсивності смоктання, часта
D. Міоглобін. блювота, гіпотонія. У сечі і крові
E. кальмодуліном. значно підвищена концентрація
цитрулина. Який метаболічний
71. До лікаря звернувся пацієнт зі процес порушений?
скаргами на запаморочення, A. * Орнітіновий цикл.
погіршення пам'яті, періодичні B. ЦТК.
судоми. Встановлено, що причиною C. Гліколіз.
таких змін є продукт
D. Глюконеогенез. D. Спостерігається підвищене
E. Цикл Корі. використання вітаміну РР.
E. Серотонін сприяє зменшенню
75. При обстеженні в поліклініці у вітаміну РР в організмі.
чоловіка діагностували гостру
променеву хворобу. Лабораторно
встановлено різке зниження 79. До лікаря звернувся чоловік 33
серотоніну в тромбоцитах. року зі скаргами на болі в суглобах.
Порушення метаболізму якої При огляді пацієнта виявлено
речовини є можливою причиною пігментація склери і вушних раковин.
зниження тромбоцитарного При аналізі сечі встановлено, що в
серотоніну? лужному середовищі і на повітрі вона
A. * 5-оксітріптофана. чорніє. Найбільш вірогідним
B. Тирозину. діагнозом є:
C. Гістидину. A. * Алкаптонурия.
D. Фінілаланіна. B. Цистиноз.
E. Серіна. C. Альбінізм.
D. Фенілкетонурія.
76. Хворому, у якого підвищена E. Тирозиноз.
кислотність шлункового соку, лікар
порекомендував їсти варене, а не 80. Немовля відмовляється від
смажене м'ясо тому, що екстрактивні годування грудьми, порушимо,
речовини, пептиди і жири смаженого дихання неритмічне, сеча має
м'яса діють переважно за наступним специфічний запах "пивної закваски"
механізмом: або "кленового сиропу".
A. * Стимулюють вироблення Природжений дефект якого ферменту
гастрину G клітинами. викликав дану патологію?
B. Дратують смакові рецептори. A. * Дегідрогенази розгалужених
C. Дратують механорецептори альфа-кетокислот.
ротової порожнини. B. Глюкозо-6-фосфатдегідрогенази.
D. Дратують механорецептори C. Гліцеролкінази.
шлунка. D. Аспартатамінотрансфераза.
E. Стимулюють вироблення E. УДФ-глюкуронілтрансферази.
секретину в 12-палої кишці.
81. У немовляти на 6 день життя в
77. У хворого з карціномной сечі виявлено надлишок
пухлиною спостерігається авітаміноз фенілпірувата і фенілацетат. Обмін
PP, в крові різко підвищена якої амінокислоти порушено в
концентрація серотоніну. Що є організмі дитини?
причиною такого стану? A. * фенілаланіну.
A. * Триптофан переважно B. Триптофану.
перетворюється в серотонін. C. Метіоніну.
B. Порушено використання D. Гістидину.
серотоніну в організмі. E. Аргініну.
C. Спостерігається аліментарній
недостатності вітаміну РР. 82. При визначенні залишкового
азоту знайшли, що азот сечовини
значно знижений. Для захворювання 90. До лікарні поступив 9-річний
якого органу це характерно? хлопчик, розумово і фізично
A. * Печінки відсталий. При біохімічному аналізі
B. Мозку крові виявлено підвищену кількість
C. Серця. фенілаланіну. Блокування якого
D. Кишечника. ферменту може призвести до такого
E. Шлунка. стану?
A. * Фенілаланін-4-монооксигенази.
83. У хворої 63 років внаслідок B. Оксидази гомогентизиновой
крововиливу в шлунково-кишковий кислоти.
тракт білки крові виявилися C. Глутамінтрансамінази.
доступними для дії мікроорганізмів D. Аспартатамінотрансфераза.
кишечника - зазнали гниття. Виберіть E. глутаматдекарбоксилази.
з нижчеперелічених речовин продукт,
концентрація якого збільшилася у 92. За клінічними показаннями
даної хворої хворому призначено прийом
A. * Індол. пиридоксальфосфата. Для корекції
B. Креатин. яких процесів рекомендований цей
C. Ціанкобаламін. препарат?
D. Тіамін. A. * Трансамінування і
E. Триптофан. декарбоксилювання амінокислот
B. Окисного декарбоксилирования
86. У немовляти спостерігаються кетокислот
епілептіморфние судоми, викликані C. Дезамірованія амінокислот
дефіцитом вітаміну В6. Це викликано D. Синтезу пуринових і
зменшенням в нервовій тканині піримідинових основ
гальмівного медіатора - гамма- E. Синтезу білка.
аміномасляної кислоти. Активність
якого ферменту знижена: 95. У сечі новонародженого
A. * глутаматдекарбоксилази. визначається цитрулін і високий
B. Аланінамінотрансфераза. рівень аміаку. Вкажіть, утворення
C. глутаматдегідрогеназа. якої речовини найімовірніше
D. пірідоксалькінази. порушено у цього малюка?
E. Глутаматсінтетази. A. * Сечовини.
B. Січовий кислоти.
87. Внаслідок попадання окропу на C. аміаку.
руку уражену ділянку шкіри D. Креатиніну.
почервонів, набряк, став болючим. E. Креатину.
Яка речовина може привести до такої
реакції? 96. Хворому 24 років для лікування
A. * Гістамін. епілепсії ввели глутамінової кислоти.
B. Лізин. Лікувальний ефект при даному
C. Тіамін. захворюванні обумовлений не самим
D. Глутамин. глутамат, а його продуктом:
E. Аспарагин. A. Гамма-аміномасляної кислотою.
B. Гістамін.
C. серотонін.
D. дофаміну. тим, що вітамін B6 входить до складу
E. таурин. ферменту:
A. * глутаматдекарбоксилази.
97. Хворий 13 років, скаржиться на B. піруватдегідрогенази.
загальну слабкість, втома. C. піруваткарбоксілази.
Спостерігається відставання в D. Амінолевулінатсінтази.
розумовому розвитку. При E. глікогенфосфорилази.
обстеженні виявлено високу
концентрацію валіну, ізолейцину, 104. При якому гіповітамінозі може
лейцину в крові та сечі. Сеча має істотно знижуватися активність
специфічний запах. Що може бути трансаміназ сироватки?
причиною такого стану: A. * В6- піридоксин.
A. * Хвороба кленового сиропу. B. В1- тіамін.
B. Хвороба Аддісона. C. В2- рибофлавін.
C. Тирозиноз. D. В5 -нікотінамід,
D. Гістидинемія. E. Нд -фоліевая кислота.
E. Базедова хвороба.
105. У дитини 2 років спостерігається
99. До лікарні доставлено дворічну відставання в розумовому і фізичному
дитину з уповільненим розумовим і розвитку, посветление шкіри і
фізичним розвитком, що страждає волосся, зниження вмісту в крові
частими рвотами після прийому їжі. катехоламінів. При додаванні до
У сечі визначена свіжої сечі декількох крапель 5%
фенилпировиноградная кислота. трихлороцтового заліза з'являється
Наслідком порушення якого обміну є оливково-зелене забарвлення. Для
дана патологія? якої патології обміну амінокислот
A. * Обміну амінокислот. характерні дані зміни?
B. Ліпідного обміну. A. * Фенілкетонурії.
C. Вуглеводного обміну. B. Алкаптонурія.
D. Водно-сольового обміну. C. Тирозиноз.
E. Фосфорно-утримуючі D. Альбінізм.
E. ксантинурія.
100. Еозинофільні гранулоцити
знижують реакцію гіперчутливості 106. У дванадцятирічного хлопчика в
негайного типу завдяки виділенню сечі виявлено високий вміст усіх
ними цілого ряду инактивирующих амінокислот аліфатичного ряду. При
ферментів. Вкажіть один з них: цьому відзначено найбільш висока
A. * гістамінази. екскреція цистину і цистеїну. Крім
B. аргінази. того, УЗД нирок показало наявність
C. глутамінази. каменів в них. Виберіть можливу
D. Аспарагиназа. патологію
E. трансаміназ. A. * Цистинурия.
B. Алкаптонурия.
102. У новонародженого C. Цистит.
спостерігалися судоми, які проходили D. Фенілкетонурія.
після призначення вітаміну B6. Цей E. Хвороба Хартнупа.
ефект найбільш ймовірно викликаний
107. При аналізі сечі 3-місячної аналізів крові і сечі дозволить
дитини виявлено підвищену кількість встановити діагноз?
гомогентизиновой кислоти, сеча при A. * Визначення концентрації
стоянні на повітрі набуває темного фенілпірувата.
забарвлення. Для якого з B. Визначення концентрації
нижчеперелічених захворювань триптофану.
характерні описані зміни? C. Визначення концентрації
A. * Алкаптонурія. гістидину.
B. Фенілкетонурії. D. Визначення концентрації лейцину.
C. альбінізм. E. Визначення концентрації валіну.
D. аміноацидурія.
E. цістінуріей. 112. Дівчинка 10-тимісячної віку,
батьки якої брюнети, має світле
108. До лікарні поступив дворічний волосся, дуже світлу шкіру і блакитні
хлопчик. У нього часті блювоти, очі. Зовні при народженні виглядала
особливо після прийому їжі. Дитина нормально, але протягом останніх 3
відстає у вазі і фізичному розвитку. місяців спостерігалися порушення
Волосся темне, але трапляються сиві мозкового кровообігу, відставання в
пасма. Запропонуйте лікування для розумовому розвитку. Причиною
даного випадку: такого стану є:
A. * Дієта зі зниженим вмістом A. * Фенилкетонурия.
фенілаланіну. B. Галактоземия.
B. Ферментативна терапія. C. Глікогеноз.
C. Введення специфічних D. Гостра порфірія.
амінокислотних сумішей. E. Гістидинемія.
D. Дієта з підвищеним вмістом
вуглеводів (або жирів) і зниженим
вмістом білків. 115. В організмі людини є пептид, в
E. безбілкову дієта. утворенні якого бере участь гамма-
карбоксильна група глутамінової
110. Основна маса азоту з організму кислоти. Цей пептид називається:
виводиться у вигляді сечовини. A. * глутатіону.
Зниження активності якого ферменту B. карнозином.
в печінці призводить до гальмування C. Ансеріном.
синтезу сечовини і нагромадження D. Окситоцин.
аміаку в крові і тканинах? E. вазопресин.
A. * Карбамоілфосфатсінтази.
B. Аспартатамінотрансфераза. 116. У новонародженої дитини в сечі
C. Уреаза. виявлено фенилпировиноградная
D. Амілази. кислота. Вкажіть патологію, з якою
E. Пепсину. це пов'язано:
A. * Фенилкетонурия.
111. У дитини 6 місяців B. Алкаптонурия.
спостерігається різке відставання в C. Альбінізм.
психомоторному розвитку, бліда D. Галактоземия.
шкіра, біляве волосся, блакитні очі, E. Подагра.
напади судом. Який з лабораторних
117. При опроміненні 122.Ізвестно, що накопичення аміаку
ультрафіолетовими променями у є провідною причиною мозкової коми
людини темніє шкіра, що є захисною при печінковій недостатності. Яка
реакцією організму. Яке захисна вільна амінокислота грає першорядну
речовина - похідне амінокислот - роль в утилізації аміаку в мозку:
синтезується в клітинах під впливом A. * Глютаминовая кислота.
ультрафіолетового опромінення? B .Аланін.
A. * Меланин. C .Тріптофан.
B. Аргінін. D. Гистидин.
C. Метіонін. E .Цістеін.
D. Фенілаланін.
E. Тироксин. 123.Под впливом опромінення
ультрафіолетовими променями у
118. Травма мозку викликала людини темніє шкіра, що є захисною
підвищене утворення аміаку. Яка реакцією організму. Яке захисна
амінокислота бере участь у речовина - похідне амінокислот -
знешкодженні аміаку в цій тканини? синтезується в клітинах під впливом
A. * Глутаминовая. ультрафіолету?
B. Тирозин. A. * Меланин.
C. Валін. B.Аргінін.
D. Триптофан. C.Метіонін.
E. Лізин. D.Фенілаланін.
E.Тіроксін.
120. У вогнищі запалення 124. Особливістю обміну тирозину є
утворюється біогенний амін, що включення його в процес синтезу
володіє судинорозширювальну дію. гормонів. Вкажіть один з них, що
Назвіть його. утворюється в мозковому шарі
A. * Гістамін. надниркових залоз.
B. Серотонін. A. * Адреналін.
C. ДОФА. B.Глюкагон.
D. триптаміни. C.Тіроксін.
E. ГАМК. D.Гістамін.
E.Серотонін.
121. В клініку госпіталізована хвора з
діагнозом карциноида кишечника. Гормони
Аналіз показав підвищення кількості
серотоніну. Відомо, що ця речовина 1. Як тироксин впливає на процеси
утворюється з амінокислоти тканинного дихання і окисного
триптофану. Який біохімічний фосфорилювання у хворої
механізм лежить в основі утворення тиреотоксикозом?
серотоніну? A. * Роз'єднує процес тканинного
A. * Декарбоксилирование. дихання і окисного фосфорилювання.
B. Дезамінування. B. Блокує транспорт електронів по
C. Мікросомальне окислення. ланцюгу цитохромів.
D. Трансамінування. C. Викликає гідроліз АТФ.
E. Освіта парних з'єднань. D. Знищує активність ФАД-
дегідрогенази.
E. Знищує активність НАД- B. Глюкокортикоїдів.
дегідрогеназ. C. Кортикотропіну.
D. Інсуліну.
2. При операції на щитовидній залозі E. соматотропіну.
помилково були видалені
паращитовидні залози. Розвинулося 6. У дитячу лікарню поступила
захворювання тетания. Обмін якого дитина з ознаками рахіту (деформація
біоелемента був порушений? кісток, пізніше заростання тім'я і ін.).
A. * Кальцію. При біохімічному аналізі крові
B. Магнію. відзначені такі зміни:
C. Калію. A. * Зниження рівня Са ++.
D. Натрію. B. Зниження рівня К +.
E. Заліза. C. Підвищення рівня фосфатів.
D. Зниження рівня Mg +.
3. На земній кулі існують території E. Підвищення рівня Na +.
(біохімічні провінції), у частини
населення яких виражений 7. У хворого з набряками вміст
ендемічний зоб. Дефіцит якого натрію в плазмі крові становить 160
біоелемента в грунті, воді та харчових ммоль / л. Зміна змісту якого гормону
продуктах викликає це може призвести до такого стану?
захворювання? A. * Збільшення альдостерону.
A. * Йода. B. Зменшення альдостерону.
B. Цинку. C. Збільшення глюкокортикоїдів.
C. Меди. D. Збільшення тиреоїдних гормонів.
D. Заліза. E. Збільшення Nа-диуретического
E. кобальту. гормону.

4. У хворого сеча в кількості 8 л на 8. Хворий 23 років скаржиться на


добу має питому вагу 1,006. При головний біль, зміна зовнішнього
недостатності функції якого гормону вигляду (збільшення розмірів ніг,
виникає це захворювання? кистей, рис обличчя), огрубіння
A. * вазопресин. голосу, погіршення пам'яті.
B. Інсуліну. Захворювання почалося приблизно 3
C. йодтироніни. роки тому без видимих причин. При
D. Глюкокортикоїдів. огляді - збільшення надбрівних дуг,
E. Соматотропіну. носа, мови. Аналіз сечі без особливих
змін. Причиною такого стану може
5. В ендокринологічний диспансер бути:
звернулася жінка 40 років зі скаргами A. * Гиперсекреция соматотропіну.
на тремтіння рук, серцебиття, B. Нестача глюкагону.
постійну гіпертермію (37-38оС), C. Нестача тироксину.
схуднення. При аналізі крові D. Недолік альдостерону.
виявлено підвищення рівня цукру, E. Гиперсекреция кортикостероїдів.
жирних кислот і амінокислот.
Гіперпродукція яких гормонів 9. Тривале вживання великих доз
викликає ці симптоми? аспірину викликає пригнічення
A. * йодтироніни (тироксин та інші).
синтезу простагландинів у результаті A. * Роз'єднують окислення і
зниження активності ферменту: окислювальне фосфорилювання.
A. * Циклооксигенази. B. Активують субстратне
B. Пероксидази. фосфорилювання.
C. 5-ліпоксигенази. C. Блокують субстратне
D. Фосфоліпази А2. фосфорилювання.
E. фосфодіестерази. D. Блокують дихальний ланцюг.
E. Активують окисне
10. Використання глюкози фосфорилювання.
відбувається шляхом її транспорту з
екстрацелюлярного простору через 14. У сечі хворого Б. виявлено цукор,
плазматичну мембрану всередину кетонові тіла, вміст глюкози в крові
клітини. Цей процес стимулюється становить 10,1 ммоль / л. Наявність
гормоном: якого захворювання можна
A. * інсуліну. припустити у хворого Б?
B. глюкагону. A. * Цукровий діабет.
C. Тироксином. B. Атеросклероз.
D. альдостерону. C. Токсичний гепатит.
E. Адреналіном. D. Панкреатит.
E. Інфаркт міокарда.
11. Для стимуляції родової діяльності
породіллі лікар призначив 15. У хворого встановлено зниження
простагландин Е2. З чого синтезу вазопресину, що призводить
синтезується це з'єднання? до поліурії і, як наслідок, до
A. * арахідонової кислоти. вираженої дегідратації організму. Що
B. фосфатидними кислоти. з перерахованого є найбільш
C. пальмітинової кислоти. імовірним механізмом поліурії?
D. стеаринової кислоти. A. * Зниження канальцевої
E. глютамінової кислоти. реабсорбції води.
B. Порушення канальцевої
12. З метою ранньої діагностики реабсорбції іонів Nа +.
вагітності досліджується сеча жінки. C. Зниження канальцевої реабсорбції
Поява яких гормонів в сечі білка.
достовірно свідчить про вагітність? D. Порушення реабсорбції глюкози.
A. * Хоріонічного гонадотропіну. E. Підвищення гідростатичного
B. Естріол. тиску.
C. 17-бета-естрадіолу.
D. Тестостерону. 16. Арахидоновая кислота як
E. Прогестерона. незамінний компонент їжі є
попередником біологічно активних
13. При тиреотоксикозі підвищується речовин. Вкажіть, які сполуки
продукція тиреоїдних гормонів Т3 і синтезуються з неї?
Т4, розвивається схуднення, A. * Простагландин Е1.
тахікардія, психічна збудливість та B. Холін.
інше. Як саме впливають тиреоїдні C. Норадреналін.
гормони на енергетичний обмін в D. Етаноламін.
мітохондріях клітин? E. Трийодтиронин.
21. Іони Са2 + - один з еволюційно
17. У дитини є порушення древніх вторинних месенджерів в
формування емалі та дентину зубів клітинах. Вони є активаторами
через пониженого вмісту іонів глікогенолізу, якщо взаємодіють з:
кальцію в крові. Дефіцит якого A. * кальмодуліном.
гормону може викликати такі зміни? B. кальцитоніну.
A. * Тиреокальцитонін. C. кальциферол.
B. Соматотропного гормону. D. кінази легких ланцюгів міозину.
C. Тироксину. E. фосфорилазу С.
D. Інсуліну.
E. трийодтироніну. 22. Зростання дорослого чоловіка
становить 112 см при пропорційній
18. У хворого на цукровий діабет статурі і нормальному розумовому
після ін'єкції інсуліну настала втрата розвитку. Недостатність вироблення
свідомості, судоми. Який результат якого гормону викликала такі
може дати біохімічний аналіз крові на симптоми?
вміст цукру? A. * Соматотропного гормону.
A. * 1,5 ммоль / л. B. гонадотропних гормонів.
B. 8,0 ммоль / л. C. антидіуретичного гормону.
C. 10,0 ммоль / л. D. Тиреотропного гормону.
D. 3,3 ммоль / л. E. Тироксину.
E. 5,5 ммоль / л.
23. Після крововиливу в мозок з
19. У хлопчика 9 років, що пошкодженням ядер гіпоталамуса у
знаходиться на стаціонарному хворої 67-річної жінки виник
лікуванні, виявлено ураження нирок і нецукровий діабет. Що стало
підвищений артеріальний тиск. З причиною поліурії в даному випадку?
підвищенням якого біологічно A. * Зменшення реабсорбції води.
активного пептиду пов'язаний цей B. Зменшення реабсорбції іонів
стан? калію.
A. * ангіотензину ІІ. C. Прискорення клубочкової
B. антидіуретичного гормону. фільтрації.
C. Глюкагон. D. Гіперглікемія.
D. Калідіна. E. Гіпоглікемія.
E. Інсуліну.
24. Простагландини
20. Лікар призначив аспірин хворому використовуються в клініці як
ревматизмом як протизапальний терапевтичні засоби. Що є основою
засіб. Синтез яких речовин, для їх синтезу?
пов'язаних із запаленням, блокує A. * Арахидоновая кислота.
аспірин? B. фосфатидними кислота.
A. * простагландинів. C. Пальмітинова кислота.
B. Гліцерину. D. Стеаринова кислота.
C. треоніну. E. Глютаминовая кислота.
D. Глюкагону.
E. Дофаміну. 25. При надлишкової секреції
гормону у хворого розвинувся
екзофтальм, виникла тахікардія, A. * вазопресин.
дратівливість і схуднення. Про який B. естрогенів.
гормон йдеться? C. Альдостерону.
A. * тироксину. D. Кортизола.
B. Адреналіні. E. Тироксину.
C. Дезоксикортикостерону.
D. Тестостерон. 30. На основі тестостерону
E. естрадіолу. синтезовані препарати, що
застосовуються при захворюваннях,
26. Особливістю обміну тирозину є що супроводжуються з виснаженням,
включення його в процес синтезу при переломах і т. Д. Який ефект в дії
гормонів. Вкажіть один з них, що цих препаратів максимально
утворюється в мозковому шарі посилений порівняно з вихідними
надниркових залоз. структурами?
A. * Адреналін. A. * Анаболический.
B. Глюкагон. B. Андрогенний.
C. Тироксин. C. естрогенів.
D. Гістамін. D. Катаболічний.
E. Серотонін. E. Амфіболіческій.

27. Тирозин використовується в 31. Хлопчик 10-ти років надійшов до


якості субстрату в процесі синтезу лікарні для обстеження з приводу
тироксину. Вкажіть хімічний елемент, маленького росту. За два останні роки
який бере участь в цьому процесі. він зріс лише на 3 см. Недостатністю
A. * Йод. якого гормону обумовлений такий
B. Кальцій. стан?
C. Залізо. A. * Соматотропного.
D. Мідь. B. Адренокортикотропного.
E. Цинк. C. Гонадотропні.
D. Тиреотропного.
28. У хворого відмічається схуднення, E. Паратгормону.
підвищення основного обміну при
підвищеному апетиті і хорошому 32. Чоловік 50 років пережив сильний
харчуванні. Надлишок якого гормону стрес. У крові різко збільшилася
викликає такі зміни? концентрація адреналіну і
A. * Тироксину. норадреналіну. Які ферменти
B. Глюкагону. каталізують процес інактивації
C. АКТГ. останніх?
D. Інсуліну. A. * моноамінооксидазу.
E. Адреналіну. B. глікозідази.
C. пептідази.
29. У хворої 50 років спостерігається D. карбоксилаза.
спрага. Добовий діурез 4-5 літрів. E. тирозинази.
Рівень глюкози в крові 4,6 ммоль / л,
в сечі глюкоза не виявлена. В даному 33. До біорегулятори клітинних
випадку доцільно перевірити вміст в функцій ліпідної природи належать
крові:
тромбоксани. Джерелом для синтезу 37. Тестовим показником на розвиток
цих сполук є: пухлини мозкової частини
A. * Арахидоновая кислота. надниркових залоз є рівень гормонів:
B. Стеаринова кислота. A. * Катехоламінів.
C. Пальмітинова кислота. B. Мінералокортикоїди.
D. фосфатидними кислота. C. Глюкокортикоїдів.
E. пальмітоолеїнової кислота. D. Полових гармонов.
E. кортіколіберіна.
34. Тривалий негативний емоційний
стрес, що супроводжується викидом 38. Яке з'єднання є попередником в
катехоламінів, може викликати синтезі простагландинів в організмі
помітне схуднення. Це пов'язано з: A. людини?
* Посиленням ліполізу. A. * Арахидоновая кислота.
B. Порушенням травлення. B. Пальмітинова кислота.
C. Посиленням окисного C. Ліноленова кислота.
фосфорилювання. D. Олеїнова кислота.
D. Порушенням синтезу ліпідів. E. Ліноленова кислота.
E. Посиленням розпаду білків.
39. Аспірин має протизапальну дію,
35. Хворий 45 скаржиться на так як пригнічує активність
невгамовну спрагу, споживання циклооксигенази. Рівень будь
великої кількості рідини (до 5 л), біологічно активних речовин буде
виділення значної кількості сечі (до 6 знижуватися?
л на добу). Концентрація глюкози A. * простагландинів.
крові становить 4,4 ммоль / л, рівень B. лейкотрієнів.
кетонових тіл не підвищений. Сеча C. Катехоламінів.
неокрашенная, питома вага 1,002; D. біогенні аміни.
цукор в сечі не визначається. Дефіцит E. Іодтіронінов.
якого гормону може призводити до
таких змін? 40. У 4-хмесячного дитини яскраво
A. * вазопресин. виражено явище рахіту. Розладів
B. Альдостерону. травлення не відзначається. Дитина
C. Інсуліну. багато знаходиться на сонці.
D. Глюкагону. Протягом 2-х місяців дитина
E. АКТГ. отримувала вітамін Д3, однак прояви
рахіту не зменшилися. Чим можна
36. Хворий знаходиться в стані пояснити розвиток рахіту у цієї
гіпоглікемічної коми. Вкажіть, дитини?
передозування якого гормону може A. * Порушенням синтезу
призвести до такої ситуації? кальцитріолу.
A. * Інсуліну. B. Порушенням синтезу
B. Прогестерона. кальцитоніну.
C. Кортизола. C. Порушенням синтезу
D. Соматотропіну. паратгормону.
E. Кортикотропіну. D. Порушенням синтезу тироксину.
E. Порушенням синтезу інсуліну.
41. При цукровому діабеті і тіла, екзофтальм, гіперглікемія,
голодуванні в крові збільшується азотемії. Яке це захворювання?
вміст ацетонових тіл, які A. * Базедова хвороба.
використовуються в якості B. Бронзова хвороба.
енергетичного матеріалу. Назвіть C. Невроз.
речовину, з якого вони синтезуються: D. Туберкульоз надниркових залоз.
A. * Ацетил-КоА. E. Мікседема.
B. сукцініл-КоА.
C. Цитрат. 46. Який з перерахованих гормонів
D. Малат. знижує швидкість ліполізу в жировій
E. кетоглутаратом. тканині?
A. * Інсулін.
42. При хронічному панкреатиті B. Адреналін.
спостерігається зменшення синтезу і C. Гидрокортизон.
секреції трипсину. Переварювання D. Соматотропін.
яких речовин порушено? E. Норадреналін.
A. * Розщеплення білків.
B. Розщеплення полісахаридів. 47. У хворого на цукровий діабет
C. Розщеплення ліпідів. після ін'єкції інсуліну настала втрата
D. Розщеплення нуклеїнових кислот. свідомості, судоми. Який результат
E. Розщеплення жиророзчинних дав біохімічний аналіз крові на вміст
вітамінів. глюкози?
A. * 2,5 ммоль / л.
43. Хворий напередодні операції B. 3,3 ммоль / л.
знаходився в стані стресу. Збільшення C. 8,0 ммоль / л.
концентрації якого гормону в крові D. 10 ммоль / л.
супроводжує цей стан? E. 5. 5 ммоль / л.
A. * Адреналіну.
B. Інсуліну. 48. До лікаря звернулися батьки з 5-
C. пролактину. річною дитиною. При обстеженні
D. Прогестерона. виявлено: відставання розумового
E. Глюкагону. розвитку і зростання, дитина
малорухомий. Загальний обмін
44. Аспірин має протизапальну і знижений. Яке захворювання у
аналгетичну дію, так як пригнічує дитини?
біосинтез простагландинів. Який A. * Кретинізм.
фермент інгібується аспірином? B. Синдром Леша-Нихана.
A. * Циклооксигеназа. C. Фенілкетонурія.
B. диоксигеназа. D. Гиперпаратиреоз.
C. гідроксилази. E. Ендемічний зоб.
D. декарбоксилаз.
E. дезамінази. 49. Вторинним посередником в
механізмі дії адреналіну є:
45. У хворого виявлено різке A. * цАМФ.
схуднення, підвищена дратівливість, B. цГМФ.
невелике підвищення температури C. УМФ.
D. ТМФ.
E. ЦМФ. C. Підвищення рівня оксипроліну в
сечі.
50. У пацієнта, що проживає на D. Гіпокальціурія.
специфічній геохимической території, E. Повиеніе змісту в крові сіалових
поставлений діагноз ендемічний зоб. кислот.
Який вид посттрансляционной
модифікації тиреоглобуліну 54. Недостатність в організмі
порушений в організмі хворого? лінолевої і ліноленової кислот
A. * Йодування. призводить до пошкоджень шкіри,
B. Метилирование. випадання волосся, сповільненого
C. Ацетилювання. загоєнню ран, тромбоцитопенії,
D. Фосфорилування. зниження опірності до інфекційних
E. Глікозилювання. захворювань. Порушення синтезу
яких речовин швидше призведе до
51. Який з перерахованих нижче зазначених симптомів?
показників найбільш імовірно A. * ейкозаноїди.
підтверджує діагноз гіпотиреозу? B. інтерлейкін.
A. * Зниження йодтіріонов в крові. C. інтерферон.
B. Зниження пальцітоніта в крові. D. Катехоламінів.
C. Підвищення холестерину в крові. E. кортикостероїдів.
D. Зниження креатиніну в сечі.
E. Зниження кальцію в сечі. 55. Чоловік 42 років страждає на
ревматоїдний артрит. У комплекс
52. При видаленні призначених йому лікувальних
гиперплазированной щитовидної препаратів включений аспірин -
залози у 47-річної жінки було інгібітор. З якої кислоти утворюються
пошкодження паращитовидних залоз. простагландини?
Через місяць після операції у A. * арахідонової.
пацієнтки з'явилися ознаки B. нейрамінової.
гипопаратиреоза: часті судоми, C. ліноленової.
гіперрефлексія, спазм гортані. Що є D. Лінолевої.
найбільш вірогідною причиною стану E. Пропіонова.
жінки?
A. * Гипокальциемия. 56. У хворого цукровий діабет, який
B. Гипонатриемия. супроводжується гіперглікемією
C. Гіперхлоргідрія. більше 7,2 ммоль / л. Рівень білка
D. Гипофосфатемия. плазми крові дозволяє
E. Гіперкаліємія. ретроспективно (за попередні 4-8
тижні до обстеження) оцінити рівень
53. У чоловіка 40 років виявлено глікемії
гіпопаратиреоз. Які результати A. * Глікозильованого гемоглобіну.
лабораторних аналізів є B. Альбуміну.
вирішальними при постановці C. Фібриногену.
діагнозу? D. С-реактивного білка.
A. * Гипокальциемия. E. Церулоплазміну.
B. Гипофосфатемия.
57. У жінки 62-х років розвинулася D. Прогестерон.
катаракта (помутніння кришталика) E. Фолікулін.
на тлі цукрового діабету. Вкажіть,
який тип модифікації білків має місце
при діабетичної катаракти 61. При огляді хворого лікар
A. * Глікозилювання. запідозрив синдром Іценко-Кушинга.
B. Фосфорилювання. Визначення якої речовини в крові
C. АДФ-рібозілірованіе. хворого може підтвердити
D. Метилирование. припущення лікаря?
E. Обмежений протеоліз. A. * Кортизола.
B. Токоферола.
58. Хвора 58 років. Стан важкий, C. Ретинолу.
свідомість затьмарена, шкіра суха, очі D. Адреналіну.
запалі, ціаноз, запах гнилих яблук з E. Холестерину.
рота. Результати аналізів: глюкоза
крові 15,1 ммоль / л, в сечі 3,5%
глюкози. Причиною такого 62. У крові пацієнта вміст глюкози
становища є: натще був 5,65 ммоль / л, через 1
A. * Гипергликемическая кома. годину після цукрового навантаження
B. Гіпоглікемічна кома. становив 8,55 ммоль / л, а через 2
C. Анафілактичний шок години - 4,95 ммоль / л. Такі
D. Уремічна кома. показники характерні для:
E. Гіповалемічна кома. A. * Здорового людини.
B.Больного з прихованим цукровим
59. Підвищену стійкість "моржів" до діабетом.
холодної води пояснюють тим, що у C.Больного з інсулінозалежним
них синтезується у великих цукровим діабетом.
кількостях гормони посилюють D.Больного з інсулінонезалежним на
процеси окислення і утворення тепла цукровий діабет.
в мітохондріях шляхом роз'єднання. E.Больного з тиреотоксикозом.
Які це гормони (гормон)?
A. * Йодовмісні гормони щитовидної 63. Продуктами гідролізу та
залози (йодтироніни). модифікації деяких білків є
B. Адреналін і норадреналін. біологічно активні речовини -
C. Глюкагон. гормони. Вкажіть, з якого з наведених
D. Інсулін. білків в гіпофізі утворюються
E. Кортикостероїди. ліпотропін, кортикотропін,
меланотропін і ендорфіни?
60. У хворої жінки з низьким A. * проопиомеланокортина (ПОМК).
артеріальним тиском після B. Нейроальбуміна.
парентерального введення гормону C. Нейростроміна.
відбулося підвищення артеріального D. Нейроглобуліна.
тиску і рівня глюкози і ліпідів в крові. E. тиреоглобуліну.
Який гормон було введено?
A. * Адреналін. 64. При хворобі Іценко-Кушинга
B. Глюкагон. (гіперфункції кори надниркових залоз
C. Інсулін. з підвищеною продукцією
кортикостероїдів) виникає
гіперглікемія. Який процес при цьому 68. У дитини після чергового
стимулюється? обстеження виявлено зупинка
A. * Глюконеогенез. мінералізації кісток. Недолік якого
B. фосфороліза глікогену. вітаміну могла бути причиною цього?
C. Цикл Кребса. A. * Кальциферолу.
D. Пентозофосфатний шлях B. Рибофлавіну.
окислення глюкози. C. Токоферолу.
E. Гліколіз. D. Фолієвої кислоти.
E. кобаламина.

65. У ендокринологічному відділенні


у жінки 40 років зі скаргами на 69. У крові пацієнта вміст глюкози
спрагу, підвищений апетит був натще 5,6 ммоль / л, через 1 год після
діагностований цукровий діабет. Які цукрового навантаження - 13,8
патологічні компоненти виявлені при ммоль / л, а через 3 години - 9,2
лабораторному дослідженні сечі ммоль / л. Такі показники вірогідні
пацієнтки? для:
A. * Глюкоза, кетонові тіла. A. * Прихованої форми цукрового
B. Білок, амінокислоти. діабету.
C. Білок, креатин. B. Здорової людини.
D. Білірубін, уробилин. C. тиреотоксикозу.
E. Кров. D. Хвороби Іценко-Кушинга.
E. акромегалії.
66. При огляді дитини 11-ти місяців
педіатр виявив викривлення кісток і
затримку мінералізації кісток черепа. 70. У хворого з діагнозом хвороба
Недолік якого вітаміну призводить до Іценко-Кушинга (гіперпродукції кори
цієї патології? надниркових залоз) в крові визначено
A. * холекальциферолу. підвищену концентрацію глюкози,
B. Тіаміну. кетонових тіл, натрію. Який
C. пантотенова кислота. біохімічний механізм є провідним у
D. біофлавоноїдів. виникненні гіперглікемії?
E. Рибофлавіну. A. * Глюконеогенез.
B. глікогенез.
C. Глікогеноліз.
67. У жінки 40 років хвороба Іценко- D. Гліколіз.
Кушинга - стероїдний діабет. При E. Аеробний гліколіз.
біохімічному обстеженні виявлені
гіперглікемія, гіпохлоремія. Який з 71. При цукровому діабеті внаслідок
перерахованих нижче процесів активації процесів окислення жирних
активується в першу чергу? кислот виникає кетоз. До яких
A. * Глюконеогенез. порушень кислотно-лужної рівноваги
B. Глікогеноліз. може привести надмірне накопичення
C. Реабсорбція глюкози. кетонових тіл в крові?
D. Транспорт глюкози в клітку. A. * метаболічного ацидозу.
E. Гліколіз. B. метаболічного алкалозу.
C. Змін не відбудеться. Що з наступного є найбільш
D. дихальний ацидоз. вірогідною причиною розвитку
E. дихальних алкалоу. рахіту:
A. * Порушення синтезу
72. Для профілактики атеросклерозу, кальцитріолу.
ішемічної хвороби серця і порушень B. Підвищена екскреція кальцію з
мозкового кровообігу людина організму.
повинна отримувати 2-6 г незамінних C. Гіперфункція паращитовидних
поліненасичених жирних кислот. Ці залоз.
кислоти необхідні для синтезу: D. Гіпофункція паращитовидних
залоз.
A. * простагладіна. E. Недостатність в їжі кальцію.
B. Жовчних кислот.
C. Стероїдів. 76. Провідними симптоми первинного
D. Вітамінів групи D. гіперпаратиреозу є остеопороз і
E. нейромедіаторів. ураження нирок з розвитком
сечокам'яної хвороби. Які речовини
73. При обстеженні дитини лікар становлять основу каменів при цьому
виявив ознаки рахіту. Брак якого захворюванні?
з'єднання в організмі дитини сприяє A. * Фосфат кальцію.
розвитку цього захворювання? B. Сечова кислота.
A. * 1,25 [ОН] - C. Цистин.
дігідроксіхолекальціферол. D. Білірубін.
B. біотин. E. Холестерин.
C. Токоферолу.
D. Нафтахінона. 77. Хлопчик 5-ти місяців
E. Ретинолу. госпіталізований з приводу
тонічнихсудом. Хворіє з народження.
74. інозітолтріфосфат в тканинах При огляді волосся жорсткі, нігті
організму утворюються в результаті витончені і тендітні, шкірні покриви
гідролізу бліді і сухі, в біохімічному аналізі
фосфатіділінозітоладіфосфата і крові кальцій 1,5 ммоль / л, фосфор -
грають роль вторинних посередників 1,9 ммоль / л. З чим пов'язані ці
(месенджерів) в механізмі дії зміни?
гормонів. Їх дія в клітці направлено A. * Гіпопаратиреоз.
на: B. Гиперпаратиреоз.
A. * Звільнення іонів кальцію з C. Гіперальдостеронізм.
клітинних депо. D. Гіпоальдостеронізм.
B. Активація аденілатциклази. E. Гіпотиреоз.
C. Активація протеїнкінази А.
D. Гальмування фосфодіестерази. 78. Харчовий раціон жінки 30 років,
E. Гальмування протеїнкінази С. яка годує груддю, містить 1000 мг
кальцію, 1300 мг фосфору і 20 мг
75. У 4-річну дитину з спадковим заліза в добу. Як слід відкоригувати
ураженням нирок спостерігаються вміст мінеральних речовин в цьому
ознаки рахіту, концентрація вітаміну харчовому раціоні?
Д в крові знаходиться в межах норми. A. * Збільшити вміст фосфору.
B. Збільшити вміст кальцію. 84. У хворих на тиреотоксикоз
C. Зменшити вміст фтору. спостерігається гіпертермія, булімія,
D. Збільшити вміст заліза. схуднення, що пов'язано з
E. Зменшити вміст заліза. порушенням:
A. * Сполучення окислення і
79. Нормальне будова і функція емалі фосфорилювання.
зуба забезпечується динамічною B. Розпаду АТФ.
рівновагою процесів демінералізації і C. Реакцій синтезу жирів.
ремінералізації. Які гормони D. Реакцій циклу лимонної кислоти.
володіють найбільш вираженою дією E. Реакцій бета-окислення жирних
на баланс цих процесів? кислот.
A. * Тиреокальцитонін і паратгормон.
B. Глюкокортикоїди і 85. До лікаря звернувся хворий зі
мінералокортикоїди. скаргами на постійну спрагу.
C. Інсулін і глюкагон. Виявлено гіперглікемія, поліурія і
D. Мінералокортикоїди і підвищена концентрація 17-
паратгормон. кетостероїдів в сечі. Яке
E. Тиреоїдні гормони і соматотропін. захворювання ймовірно?
A. * Стероїдний діабет.
80. У чоловіка 35 років B. Інсулінозалежний діабет.
феохромоцитома. У крові виявлено C. Мікседема.
підвищений рівень адреналіну і D. Глікогеноз I типу.
норадреналіну, концентрація вільних E. Аддисонова хвороба.
жирних кислот зросла в 11 разів.
Вкажіть, активація якого ферменту 86. У хворого виявлено різке
під впливом адреналіну підвищує схуднення, підвищена подразливість,
ліполіз? невелике підвищення температури
A. * ТАГ-ліпази. тіла, екзофтальм, гіперглікемію,
B. ліпопротеідліпази. азотемію. Яке це захворювання?
C. Фосфоліпази А2. A. * Базедова хвороба.
D. Фосфоліпази С. B. Бронзова хвороба.
E. Холестеролестерази. C. Невроз.
D. Туберкульоз надниркових залоз.
83. Хворий звернувся до лікаря зі E. Мікседема.
скаргами на часте і надмірне
сечовиділення, спрагу. При аналізі 87. У хворого на цукровий діабет
сечі виявлено: добовий діурез -19 після ін'єкції інсуліну сталася втрата
літрів, щільність сечі 1,001. Для якого свідомості, судоми. Який результат
захворювання ці показники є дав біохімічний аналіз крові на вміст
характерними? глюкози?
A. * нецукровий діабет. A. * 2. 5 ммоль / л.
B. стероїдний діабет. B. 3,3 ммоль / л.
C. Цукрового діабету. C. 8,0 ммоль / л.
D. тиреотоксикоз. D. 10 ммоль / л.
E. Хвороби Аддісона. E. 5. 5 ммоль / л.
88. У хворого цирозом печінки інгібітор циклооксигенази. З якої
з'явилися ознаки кислоти утворюються
гіперальдостеронізму. Зниження простагландини?
активності яких ферментів привело до A. * арахідонової.
недостатньої інактивації гормону? B. нейрамінової.
A. * мікросомальне гідроксилази. C. ліноленової.
B. Цитоплазматичні D. Лінолевої.
глутатіонпероксидази. E. Пропіонова.
C. Цитоплазматичні дегидрогеназ.
D. мітохондріальної дегидрогеназ. 93. У чоловіка 62 років діагностували
E. лізосомальних гідролаз. аденому передміхурової залози. Йому
призначили синестрол - синтетичний
89. З метою аналгезії можуть бути естрогенний препарат. Що, в першу
використані речовини, які імітують чергу, обумовлює терапевтичний
ефекти морфіну, але виробляються в ефект цього препарату?
ЦНС. Вкажіть їх. A. * Блокування рецепторів
A. * Бета-ендорфін. тестостерону.
B. Окситоцин. B. Гальмування трансляції.
C. Вазопресин. C. Порушення гліколізу.
D. Кальцитонін. D. Зниження транскрипції.
E. Соматоліберин. E. Гальмування синтезу ЛПДНЩ.

90. У пацієнта, який проживає на 94. У чоловіка 40 років виявлено


специфічній геохимической території, гіпопаратиреоз. Які результати
поставлений діагноз ендемічний зоб. лабораторних аналізів були
Який вид посттрансляционной вирішальними при встановленні
модифікації тиреоглобуліну діагнозу?
порушений в організмі хворого? A. * Гипокальциемия.
A. * Йодування. B. Гипофосфатемия.
B. Метилирование. C. Підвищений рівень оксипроліну в
C. Ацетилювання. сечі.
D. Фосфорилування. D. Гіпокальціурія.
E. Глікозилювання. E. Підвищення вмісту в крові
сіалових кислот.
91. Який з перерахованих нижче
показників найбільш імовірно 95. У хлопчика 15 років з дефіцитом
підтверджує діагноз гіпотиреоз? антидіуретичного гормону
A. * Зниження йодтиронінів в крові. спостерігається:
B. Зниження кальцитонина в крові. A. * Полиурия.
C. Підвищення холестерину в крові. B. Кетонурія.
D. Зниження креатиніну в сечі. C. Глюкозурія.
E. Зниження кальцію в сечі. D. креатинурія.
E. аміноацидурія.
92. Людина 42 років страждає на
ревматоїдний артрит. До комплексу 96. У хворих на тиреотоксикоз
призначених йому лікувальних спостерігається гіпертермія, тремор,
препаратів включений аспірин -
схуднення, що пов'язано з B. Зменшення альдостерону.
порушенням: C. Збільшення глюкокортикоїдів.
A. * Сполучення окислення і D. Збільшення тиреоїдних гормонів.
фосфорилювання. E. Збільшення натрійдіуретічного
B. Розпаду АТФ. гормону.
C. Реакцій окислення тригліцеридів.
D. Реакцій циклу лимонної кислоти. 101. Людина 38 років проходить курс
E. Реакцій бета-окислення жирних лікування в стаціонарі з приводу
кислот. шизофренії. Початкове вміст у крові
глюкози, кетонових тіл, сечовини - в
97. У хворого за добу виділяється 8 л нормі. Шокова терапія регулярними
сечі з питомою вагою 1,006. При ін'єкціями інсуліну призвела до
недостатній функції якого гормону це розвитку інсулінової коми, після чого
виникає? настав поліпшення стану хворого. Що
A. * Вазопресину. було найбільш можливою причиною
B. Інсуліну. інсулінової коми?
C. йодтироніни. A. * Гіпоглікемія.
D. Глюкокортикоїдів. B. Дегідратація тканин.
E. Соматотропіну. C. Метаболічний ацидоз.
D. Кетонемія.
98. Пацієнт скаржиться на м'язову E. Глюкозурія.
слабкість і потемніння шкіри всього
тіла. При обстеженні виявлено такі 102. Характерними ознаками холери є
зміни: артеріальний тиск -100/60 втрата організмом великої кількості
ммртст, рівень глюкози крові 3,0 мм / води та іонів натрію. Основою
л. Яку хворобу можна запідозрити? біохімічного дії холерного токсину є:
A. * Хвороба Аддісона. A. * Активація аденілатциклази
B. мікседему. тонкого кишечника.
C. інсули (з підвищеною продукцією B. Активація синтезу передсердь-ного
інсуліну). натрійуретичного фактора.
D. Синдром Іценко-Кушинга. C. Гальмування синтезу
E. пелагрі. антидіуретичного гормону в
гіпоталамусі.
99. У хворого вміст калію в плазмі D. Посилення секреції реніну
крові становить 7 ммоль / л. Які клітинами юкстагломерулярного
можливі причини такого стану? апарату ниркових артеріол.
A. * Зменшення альдостерону. E. Окислення альдостерону в корі
B. Збільшення альдостерону. надниркових залоз.
C. Зменшення тиреоїдних гормонів.
D. Збільшення тиреоїдних гормонів. 103. Хворий 40 років
E. Збільшення статевих гормонів. госпіталізований зі скаргами на
загальну слабкість, судоми верхніх і
100. У хворого вміст натрію в плазмі нижніх кінцівок, АТ - 160/100 мм рт.
крові становить 160 ммоль / л. Зміна ст. Результати дослідження: глюкоза
змісту якого гормону може призвести крові - 6,5 ммоль / л, холестерин - 6
до такого стану? ммоль / л, кальцій - 2 ммоль / л,
A. * Збільшення альдостерону. фосфор - 1 ммоль / л, натрій - 160
ммоль / л. Сечовиділення - 700 мл за тіла. Які біохімічні процеси при
добу. Яка патологія послужила цьому активуються?
причиною такої стану? A. * Катаболізм.
A. * Гіперальдостеронізм. B. Анаболизм.
B. Гіпоальдостеронізм. C. неоглюкогенезу.
C. гіперпаратиреоїдизмом. D. Ліпогенез.
D. Тиреотоксикоз. E. стероїдогенезу.
E. Рахіт.
108. У хлопчика 9 років, який
104. Тривале вживання великих доз знаходиться на стаціонарному
аспірину викликає пригнічення лікуванні, виявлено ураження нирок і
синтезу простагландинів у результаті підвищений артеріальний тиск. З
зниження активності ферменту: підвищенням вмісту якого біологічно
A. * Циклооксигенази. активного пептиду пов'язано це стан?
B. Пероксидази. A. * ангіотензину II.
C. 5-ліпоксигенази. B. антидіуретичного гормону.
D. Фосфоліпази А2. C. Глюкагону.
E. фосфодіестерази. D. Калідіна.
E. Інсуліну.
105. Використання Гюкоза
відбувається шляхом її транспорту з 109. У хворого на цукровий діабет,
екстрацелюлярного простору через після введення інсуліну настала
плазматичну мембрану всередину втрата свідомості, спостерігаються
клітини. Цей процес стимулюється судоми. Який результат дав
гормоном: біохімічний аналіз крові на вміст
A. * інсуліну. цукру?
B. глюкагону. A. * 1,5 ммоль / л.
C. Тироксином B. 3,3 ммоль / л.
D. альдостерону. C. 8 ммоль / л.
E. Адреналіном. D. 10 ммоль / л.
E. 5,5 ммоль / л.
106. Чоловік 50 років пережив
сильний стрес. У крові різко 110. Простагландини
збільшилася концентрація адреналіну використовуються в клініці як
і норадреналіну. Які ферменти терапевтичні засоби. Що є основою
каталізують процес інактивації для їх синтезу?
останніх? A. * Арахидоновая кислота.
A. * моноамінооксидазу. B. фосфатидними кислота.
B. глікозідази. C. Пальмітинова кислота.
C. пептідази. D. Стеаринова кислота.
D. карбоксилаза. E. Глютаминовая кислота.
E. тирозинази.
111. У хворого в віці 28 років
107. При підвищенні функції внаслідок її пухлини спостерігається
щитовидної залози спостерігається непропорційно інтенсивний ріст рук,
втрата ваги і підвищення температури ніг, підборіддя (акромегалія).
Надлишок якого гормону викликав 116. При обстеженні пацієнта
подібні порушення? встановлено збільшення основного
A. * Соматотропіну. обміну на 50%. Збільшення секреції
B. Тироксину. якого гормону стало причиною цієї
C. Гонадотропіну. зміни?
D. Аденокортік A. * Тироксину
E. Інсуліноподібний чинники B. Інсуліну
зростання. C. Паратгормону
D. Соматотропного гормону
113. При емоційному стресі в E. пролактину
адипоцитах активується
гормончувствітельной 117. Продукуючи ряд гормонів,
трігліцерідліпази. Який вторинний плацента грає роль тимчасової
посередник бере участь у цьому ендокринної залози. Який гормон
процесі? може бути визначений в крові жінки
A. * ЦАМФ. вже на третю-четверту добу після
B. ЦГМФ. початку імплантації, що
C. АМФ. використовується в медичній
D. Диацилглицерол. практиці для ранньої діагностики
E. Іони Са2 +. вагітності?
A. * Хоріонічний гонадотропін.
114. У хворої 50 років підвищена B. Прогестерон.
спрага. Добовий діурез 4-5літров. C. Соматостатин
Рівень глюкози в крові 4,6 ммоль / л, D. Вазопресин.
в сечі глюкоза не виявлена. В даному E. Окситоцин.
випадку доцільно перевірити
кількість в крові: 118.Лімонная кислота має високі
A. * вазопресин. комплексонобразующіе властивості і
B. естрогенів. бере участь в процесі мобілізації
C. Альдостерону. кальцію. У якої тканини з
D. Кортизола. перерахованих найбільший вміст
E. Тироксину. лимонної кислоти?
A. * Кісткова.
115.Інозітолтріфосфати в тканинах B.Паренхіматозная.
організму утворюються в результаті C.Нервная.
гідролізафосфатіділінозітолдіфосфато D.Мишечная.
в і грають роль вторинних E.Епітеліальная.
посередників (месенджерів) в
механізмі дії гормонів. Їх дія в клітці 119.Продуктамі гідролізу та
направлено на: модифікації деяких білків є
A. * Вивільнення іонів кальцію з біологічно активні речовини -
клітинних депо гормони. Вкажіть, з якого із
B .Актівація аденілатциклази зазначених білків в гіпофізі
C. Активація протеїнкінази А утворюється ліпотропін,
D. Гальмування фосфодіестерази кортикотропін, меланотропін і
E. Гальмування протеїнкінази С ендорфіни?
A. * проопиомеланокортина (ПОМК).
B.Нейроальбумін. крові. На якому фізико-хімічному
C.Нейростромін. властивості білків заснований цей
D.Нейроглобулін. метод?
E.Тіреоглобулін. A. * Наявності заряду.
B. В'язкості.
120.Больному з ревматоїдним C. Нездатності до денатурації.
артірітом тривалий час вводили D. гідрофільних і здатності до
гідрокортизон. У нього з'явилася набухання.
гіперглікемія, поліурія, спрага. Ці E. Оптичної активності.
ускладнення лікування є наслідком:
A. * глюконеогенезі. 4. Еритроцити для своєї
B.Глікогеноліза. життєдіяльності вимагає енергію у
C.Глікогенеза. вигляді АТФ. Який процес забезпечує
D.Гліколіза. цю клітину необхідною кількістю
E.Ліполіза. АТФ?
A. * Анаеробний гліколіз.
Кров B. Аеробне окислення глюкози.
C. пентозного цикл.
1. Вкажіть, з яким білком зв'язується D. Бета-окислення жирних кислот.
гемоглобін для переносу в E. Цикл трикарбонових кислот.
ретикулоендотеліальної системи
печінки? 5. В якості антикоагулянтів
A. * гаптоглобіну. використовують різні речовини, в
B. альбуміну. тому числі полісахарид природного
C. феритин. походження, а саме:
D. трансферина. A. * Гепарин.
E. церулоплазміну. B. Гіалуронову кислоту.
C. дерматансульфат.
2. У хворого з тяжкою формою D. Хондроідінсульфат.
гемолітичної анемії еритроцити E. Декстран.
мають форму серпа. У чому полягає
молекулярна причина виникнення 6. При підвищенні концентрації
даного захворювання? чадного газу в повітрі може
A. * Заміна глутаміну на валін в Δ наступити отруєння. При цьому
ланцюга гемоглобіну. порушується транспортування
B. Порушення синтезу порфіринів. гемоглобіном кисню від легенів до
C. Порушення синтезу Δ ланцюга тканин. Яке похідне гемоглобіну при
гемоглобіну. цьому утворюється?
D. Зупинка синтезу Δ ланцюга A. * Карбоксигемоглобин.
гемоглобіну. B. Оксигемоглобін.
E. Порушення синтезу гема. C. Метгемоглобін.
D. карбгемоглобін.
3. Лікар, перш ніж призначити E. Гемохромоген.
виснаженому хворому білкове
парентеральне харчування, призначив 7. Хворий поступив в реанімаційне
в лабораторії визначити відділення з підозрою на отруєння
електрофоретичний спектр білків чадним газом (монооксидом
вуглецю). Яке з'єднання гемоглобіну D. Цитохром.
буде виявлена при спектральному E. Цитохромоксидази.
аналізі?
A. * Карбоксигемоглобин. 12. Біохімічний аналіз сироватки
B. карбгемоглобін. крові пацієнта з гепатолентикулярной
C. Метгемоглобін. дегенерацією (хвороба Вільсона-
D. Оксигемоглобін. Коновалова) виявив зниження вмісту
E. дезоксигемоглобином. церулоплазміну. Концентрація яких
іонів буде підвищена в сироватці
8. Поряд з нормальними типами крові цього пацієнта?
гемоглобіну в організмі дорослої A. * Мідь.
людини можуть бути присутніми B. Кальцію.
патологічні. Вкажіть один з них. C. Фосфору.
A. * HbS. D. Калію.
B. HbF. E. Натрію.
C. HbA1.
D. HbA2. 13. У сироватці крові пацієнта
E. HbO2. встановили підвищення активності
гіалуронідази. Визначення якого
9. Гемоглобін дорослої людини (HbA) біохімічного показника сироватки
- білок-тетрамер, що складається з крові дозволить підтвердити
двох альфа- і двох бета-пептидних припущення про патологію сполучної
ланцюгів. Як називається така тканини?
структура цього білка? A. * сіалові кислоти
A. * Четвертичная. B. Білірубіну.
B. Третинна. C. Січовий кислоти.
C. Вторинна. D. Глюкози.
D. Первинна. E. галактоза.

10. При хворобі Коновалова-Вільсона 14. При гострих запальних процесах в


має місце зниження вмісту в плазмі плазмі крові з'являється "білок
крові білка, який транспортує іони гострої фази", визначення якого має
міді. Вкажіть цей білок. діагностичне значення. Що це за
A. * Церулоплазмін. білок?
B. трансферин. A. * С-реактивний білок.
C. Гаптоглобін. B. Альбумін.
D. Фібронектин. C. Миоглобин.
E. З-реактивний білок. D. Гемоглобін.
E. карбгемоглобін.
11. У легких вугільна кислота
(Н2СО3) за допомогою ферменту 15. У процесі катаболізму
розкладається до води і вуглекислого гемоглобіну звільняється залізо, яке в
газу, який виділяється в повітря. Який складі спеціального транспортного
фермент каталізує цю реакцію? білка надходить в кістковий мозок і
A. * Карбоангидраза. знову використовується для синтезу
B. Каталаза. гемоглобіну. Цим транспортним
C. Пероксидаза. білком є:
A. * трансферина (сидерофилина). білків згортання крові бере участь цей
B. транскобаламіном. вітамін?
C. Гаптоглобін. A. * протромбіну.
D. Церулоплазмін. B. Фібриногену.
E. Альбумін. C. Фактора Хагемана.
D. антигемофільний глобуліну A.
16. При дії окислювачів (перекису E. Фактора Розенталя.
водню, оксидів азоту та ін.),
Гемоглобін, до складу якого входить 20. У хворого швидко розвиваються
Fe2 +, перетворюється в з'єднання, що набряки. Зниження яких білків
містить Fe3 +. Це з'єднання здатне сироватки крові призводить до їх
переносити кисень і називається: виникнення?
A. * Метгемоглобін. A. * альбуміну.
B. Карбоксигемоглобин. B. Альфа-1-глобулінів.
C. карбгемоглобін. C. Альфа-2-глобулінів.
D. Оксигемоглобін. D. Бета-глобулінів.
E. Глікозильований гемоглобін. E. Фібриногену.

17. У процесі метаболізму в організмі 21. Ті організми, які в процесі


людини виникають активні форми еволюції не створили захисту від
кисню, в тому числі супероксидний Н2О2, можуть жити лише в
аніон-радикал О2. Цей аніон анаеробних умовах. Які з
інактивується за допомогою перерахованих ферментів можуть
ферменту: зруйнувати перекис водню?
A. * супероксиддисмутази. A. Пероксидаза і каталаза.
B. каталази. B. оксигенази і гідроксилази.
C. Пероксидази. C. Цитохромоксидази, цитохром В5.
D. глутатіонпероксидази. D. оксигенази і каталаза.
E. глутатіонредуктаза. E. Флавінзавісімие оксидази.

18. Знешкодження хвороботворних 22. При обтураційній жовтяниці і


бактерій і розщеплення чужорідних жовчних свищах часто
тіл в лейкоцитах здійснюється за спостерігається протромбінового
типом реакції окислення: недостатність. З дефіцитом якого
A. * пероксидазного. вітаміну це пов'язано?
B. оксидазний. A. * К.
C. оксігеназной. B. В6.
D. Перекисного. C. А.
E. Анаеробного. D. С.
E. Е.
19. У 6-ти місячної дитини
спостерігалися часті і сильні 23. Яке похідне гемоглобіну
підшкірні кровотечі. Призначення виявляється в крові при отруєнні
синтетичного аналога вітаміну K чадним газом (монооксидом
(вікасолу) дало позитивний ефект. У вуглецю)?
гамма-карбоксилювання глутамінової A. * Карбоксигемоглобин.
кислоти якого з перерахованих нижче B. Метгемоглобін.
C. Оксигемоглобін.
D. карбгемоглобін. 28. Молекулярний аналіз гемоглобіну
E. Вердогемоглобін. пацієнта, що страждає анемією,
виявив заміну 6Глу на 6Вал бета-
24. Електрофоретичне дослідження ланцюга. Який молекулярний
сироватки крові хворого на механізм патології?
пневмонію показало збільшення A. * Генна мутація.
однієї з білкових фракцій. Вкажіть її. B. Хромосомная мутація.
A. * Гамма-глобуліни. C. Геномна мутація.
B. Альбуміни. D. Ампліфікація генів.
C. Альфа1-глобуліни. E. Трансдукція генів.
D. альфа2 глобуліни.
E. Бета глобуліни. 29. У хворого на цукровий діабет
змінилося значення рН крові і стало
25. При аналізі крові у хворого рівним 7,3. Визначення компонентів
залишковий азот склав 48 ммоль / л, який буфферной системи
сечовина 15,3 ммоль / л. Про використовується для діагностики
захворювання якого органу свідчать розладів кислотно-лужної рівноваги?
результати аналізу? A. * бікарбонатному.
A. * Нирок. B. Фофатной.
B. Печінки. C. гемоглобінового.
C. Шлунка. D. Оксігемоглобіновой.
D. Кишечника. E. Білкової.
E. селезінки.
30. Мікроелемент мідь є складовим
26. При аналізі крові хворого компонентом білків
визначено залишковий азот і (Металопротеїни). При порушенні
сечовина. Частка сечовини в обміну міді виникає хвороба Вільсона
залишковому азоті істотно зменшена. (гепатоцеребральная дистрофія).
Для захворювання якого органу Концентрація якого білка
характерний даний аналіз? зменшується в крові?
A. * Печінка. A. * Церулоплазміну.
B. Нирки. B. трансферин.
C. Шлунок. C. феритин
D. Кишечник. D. колагену.
E. Серце. E. Глобуліну.

27. Пролонгована дія ряду 31. У немовляти внаслідок


антибіотиків і сульфаніламідів неправильного годування виникла
обумовлено тим, що вони виражена діарея. Одним з основних
циркулюють в крові тривалий час в наслідків діареї є екскреція великої
комплексі з: кількості бікарбонату натрію. Яка
A. * альбуміну. форма порушення кислотно-лужного
B. трансферин. балансу має місце в даному випадку?
C. Гемоглобін. A. * Метаболічний ацидоз.
D. гаптоглобіну. B. Метаболічний алкалоз.
E. гемопексин. C. Респіраторний ацидоз.
D. Респіраторний алкалоз. Виберіть причину розвитку набряків
E. Чи не буде порушень кислотно- у дитини.
лужного балансу. A. * Зниження синтезу альбумінів.
B. Зниження синтезу глобулінів.
32. Мати звернулася до лікаря: у C. Зниження синтезу гемоглобіну.
дитини 5 років під дією сонячних D. Зниження синтезу ліпопротеїнів.
променів на шкірі з'являються E. Зниження синтезу глікопротеїнів.
еритеми, везикулярна висип, дитина
скаржиться на свербіж. Дослідження 21. Ті організми, які в процесі
крові виявили зменшення заліза в еволюції не створили захисту від
сироватці крові, збільшення Н2О2, можуть жити лише в
виділення з сечею уропорфіріноген І. анаеробних умовах. Які з
Найбільш імовірною спадковою перерахованих ферментів можуть
патологією у дитини є: зруйнувати перекис водню?
A. * Ерітропоетіческая порфірія. A. Пероксидаза і каталаза.
B. Метгемоглобінемія. B. оксигенази і гідроксилази.
C. Печінкова порфірія. C. Цитохромоксидази, цитохром В5.
D. копропорфірія. D. оксигенази і каталаза.
E. Інтермітуюча порфірія. E. Флавінзавісімие оксидази.

33. У 3-річну дитину з підвищеною 22. При обтураційній жовтяниці і


температурою тіла після прийому жовчних свищах часто
аспірину спостерігається посилений спостерігається протромбінового
гемоліз еритроцитів. Вроджена недостатність. З дефіцитом якого
недостатність якого ферменту могла вітаміну це пов'язано?
викликати у дитини гемолітична A. * К.
анемія? B. В6.
A. * Глюкозо-6-фосфатдегідрогенази. C. А.
B. Глюкозо-6-фосфатази. D. С.
C. Глікогенфоссфорілази. E. Е.
D. Гліцеролфосфатдегідрогенази.
E. Гамма-глутамілтрансферази. 23. Яке похідне гемоглобіну
виявляється в крові при отруєнні
34. При визначенні залишкового чадним газом (монооксидом
азоту знайшли, що азот сечовини вуглецю)?
значно знижений. Для захворювання A. * Карбоксигемоглобин.
якого органу це характерно? B. Метгемоглобін.
A. * Печінки. C. Оксигемоглобін.
B. Мозку. D. карбгемоглобін.
C. Серця. E. Вердогемоглобін.
D. Кишечника.
E. Шлунка. 24. Електрофоретичне дослідження
сироватки крові хворого на
35. До лікарні потрапила дитина 4-х пневмонію показало збільшення
років з ознаками тривалого білкового однієї з білкових фракцій. Вкажіть її.
голодування: затримка росту, анемія, A. * Гамма-глобуліни.
набряки, розумова відсталість. B. Альбуміни.
C. Альфа1-глобуліни. E. Трансдукція генів.
D. альфа2 глобуліни.
E. Бета глобуліни. 29. У хворого на цукровий діабет
змінилося значення рН крові і стало
25. При аналізі крові у хворого рівним 7,3. Визначення компонентів
залишковий азот склав 48 ммоль / л, який буфферной системи
сечовина 15,3 ммоль / л. Про використовується для діагностики
захворювання якого органу свідчать розладів кислотно-лужної рівноваги?
результати аналізу? A. * бікарбонатному.
A. * Нирок. B. Фофатной.
B. Печінки. C. гемоглобінового.
C. Шлунка. D. Оксігемоглобіновой.
D. Кишечника. E. Білкової.
E. селезінки.
30. Мікроелемент мідь є складовим
26. При аналізі крові хворого компонентом білків
визначено залишковий азот і (Металопротеїни). При порушенні
сечовина. Частка сечовини в обміну міді виникає хвороба Вільсона
залишковому азоті істотно зменшена. (гепатоцеребральная дистрофія).
Для захворювання якого органу Концентрація якого білка
характерний даний аналіз? зменшується в крові?
A. * Печінка. A. * Церулоплазміну.
B. Нирки. B. трансферин.
C. Шлунок. C. феритин
D. Кишечник. D. колагену.
E. Серце. E. Глобуліну.

27. Пролонгована дія ряду 31. У немовляти внаслідок


антибіотиків і сульфаніламідів неправильного годування виникла
обумовлено тим, що вони виражена діарея. Одним з основних
циркулюють в крові тривалий час в наслідків діареї є екскреція великої
комплексі з: кількості бікарбонату натрію. Яка
A. * альбуміну. форма порушення кислотно-лужного
B. трансферин. балансу має місце в даному випадку?
C. Гемоглобін. A. * Метаболічний ацидоз.
D. гаптоглобіну. B. Метаболічний алкалоз.
E. гемопексин. C. Респіраторний ацидоз.
D. Респіраторний алкалоз.
28. Молекулярний аналіз гемоглобіну E. Чи не буде порушень кислотно-
пацієнта, що страждає анемією, лужного балансу.
виявив заміну 6Глу на 6Вал бета-
ланцюга. Який молекулярний 32. Мати звернулася до лікаря: у
механізм патології? дитини 5 років під дією сонячних
A. * Генна мутація. променів на шкірі з'являються
B. Хромосомная мутація. еритеми, везикулярна висип, дитина
C. Геномна мутація. скаржиться на свербіж. Дослідження
D. Ампліфікація генів. крові виявили зменшення заліза в
сироватці крові, збільшення швидку стомлюваність. При
виділення з сечею уропорфіріноген І. обстеженні виявлено: гемоглобін
Найбільш імовірною спадковою крові 80 г / л, мікроскопічно виявлено
патологією у дитини є: еритроцити зміненої форми.
A. * Ерітропоетіческая порфірія. Причиною може бути:
B. Метгемоглобінемія. A. * Серповидноклеточная анемія.
C. Печінкова порфірія. B. Паренхіматозна жовтяниця.
D. копропорфірія. C. Гостра переміжна порфірія.
E. Інтермітуюча порфірія. D. Обтураційна жовтяниця.
E. Хвороба Аддісона.
33. У 3-річну дитину з підвищеною
температурою тіла після прийому 37. Людина в стані спокою штучно
аспірину спостерігається посилений примушує себе дихати часто і
гемоліз еритроцитів. Вроджена глибоко протягом 3-4 хв. Як це
недостатність якого ферменту могла відбивається на кислотно-лужний
викликати у дитини гемолітична рівновазі організму?
анемія? A. * Виникає дихальний алкалоз.
A. * Глюкозо-6-фосфатдегідрогенази. B. Виникає дихальний ацидоз.
B. Глюкозо-6-фосфатази. C. Виникає метаболічний алкалоз.
C. Глікогенфоссфорілази. D. Виникає метаболічний ацидоз.
D. Гліцеролфосфатдегідрогенази. E. Кислотно-лужна рівновага не
E. Гамма-глутамілтрансферази. змінюється.

34. При визначенні залишкового 38. У хворої 38 років ревматизм в


азоту знайшли, що азот сечовини активній фазі. Визначення якого з
значно знижений. Для захворювання наступних лабораторних показників
якого органу це характерно? сироватки крові має діагностичне
A. * Печінки. значення при даній патології?
B. Мозку. A. * С-реактивного білка.
C. Серця. B. Січовий кислоти.
D. Кишечника. C. Сечовини.
E. Шлунка. D. Креатиніну.
E. трансферинами.
35. До лікарні потрапила дитина 4-х
років з ознаками тривалого білкового 39. У відділення травматології
голодування: затримка росту, анемія, поступив хворий з размозжением
набряки, розумова відсталість. м'язової тканини. Вкажіть, який
Виберіть причину розвитку набряків біохімічний показник сечі при цьому
у дитини. буде збільшений:
A. * Зниження синтезу альбумінів. A. * Креатинін.
B. Зниження синтезу глобулінів. B. Загальні ліпіди.
C. Зниження синтезу гемоглобіну. C. Глюкоза.
D. Зниження синтезу ліпопротеїнів. D. Мінеральні солі.
E. Зниження синтезу глікопротеїнів. E. Сечова кислота.

36. Хворий 20 років. Скаржиться на 40. У хворого 27-ми років виявлено


загальну слабкість, запаморочення, патологічні зміни печінки і головного
мозку. У плазмі крові виявлено різке B. Гіпоглікемічна кома.
зниження, а в сечі підвищення вмісту C. Гостра серцева недостатність.
міді. Поставлено діагноз - хвороба D. Гостре алкогольне сп'яніння.
Вільсона. Активність якого ферменту E. Анафілактичний шок.
в сироватці крові необхідно дослідити
для підтвердження діагнозу? 44. Жінка 62 років скаржиться на
A. * Церулоплазміну. часту біль в області грудної клітини
B. Карбоангідрази. та хребта, переломи ребер. Лікар
C. ксантиноксидазу. припустив мієломну хворобу
D. лейцінамінопептідази. (плазмоцити). Який з перерахованих
E. Алкогольдегідрогенази. нижче лабораторних показників буде
мати найбільш діагностичне
41. Пацієнт звернувся до лікаря зі значення?
скаргами на задишку, що виникає A. * парапротеінеміях.
після фізичного навантаження. B. Гіперальбумінемія.
Клінічне обстеження виявило анемію C. Протеїнурія.
та наявність парапротеина в зоні D. гіпоглобулінемія.
гамма-глобулінів. Який показник в E. Гипопротеинемия.
сечі необхідно визначити для
підтвердження діагнозу мієломи? 45. Хворий 50-ти років звернувся в
A. * Білок Бенс-Джонса. лікарню зі скаргами на загальну
B. Білірубін. слабкість, втрату апетиту, аритмію.
C. Гемоглобін. Спостерігається гіпотонія м'язів,
D. Церулоплазмін. мляві паралічі, ослаблення
E. антитрипсину. перистальтики кишечника. Причиною
такого стану може бути:
42. У результаті м'язової роботи у A. * Гіпокаліємія.
робочого значно зменшилася буферна B. Гипопротеинемия.
ємність крові. Надходженням якого C. Гіперкаліємія.
кислого речовини можна пояснити це D. Гипофосфатемия.
явище? E. Гипонатриемия.
A. * Лактату.
B. пірувату. 46. Споживання забруднених овочів і
C. 1,3-бісфосфогліцерата. фруктів протягом тривалого часу
D. Альфа-кетоглутарата. призвело до отруєння пацієнта
E. 3-фосфогліцерата. нітратами і утворення в крові
похідного гемоглобіну:
43. Хвору привезла швидка допомога. A. * Hb-OH.
Стан важкий, свідомість відсутня, B. Hb СО.
адинамія. Шкірні покриви сухі, запалі C. Hb O2.
очі. Ціаноз особи. Тахікардія. Запах D. Hb CN
ацетону з рота. Результати аналізів: E. Hb NHCOOH.
глюкоза крові 20,1 ммоль / л (N = 3,3-
5,5 ммоль / л), в сечі 3,5% (в N = 0). 47. Робочий цеху по виробництву
Який попередній діагноз можна нітросполук звернувся до лікаря зі
поставити? скаргами на задишку і швидку
A. * Гипергликемическая кома. стомлюваність. При обстеженні
хворого виявлено ціаноз нижніх D. Обтураційна жовтяниця.
кінцівок. Яка причина цього стану? E. Гемолітична жовтяниця.
A. * Посилене
метгемоглобинообразование. 51. У хворого виявлено зниження рН
B. Гіповітаміноз. крові і вмісту бікарбонатних іонів
C. Гіпервітаміноз. (падіння лужного резерву крові),
D. Жирова інфільтрація печінки. зростання вмісту молочної,
E. Авітаміноз. піровиноградної кислот в крові і сечі.
Який тип порушення кислотно-
48. У хворого 35 років, який часто лужної рівноваги спостерігається?
вживає алкоголь, на тлі лікування A. * Метаболічний ацидоз.
сечогінними засобами виникла сильна B. Респіраторний ацидоз.
м'язова і серцева слабкість, блювання, C. Метаболічний алкалоз.
діарея, АТ - 100/60 мм рт. ст. , D. Респіраторний алкалоз.
Депресія. Причиною такого стану є E. Дихальний алкалоз.
посилене виділення з сечею:
A. * Калію. 52. Після ремонту автомобіля в
B. Натрію. гаражному приміщенні водій
C. Хлора. потрапив до лікарні з симптомами
D. Кальцію. отруєння вихлопними газами.
E. Фосфатів. Концентрація якого гемоглобіну в
крові буде підвищена?
49. Жінка 33 років страждає A. * карбоксигемоглобін
гепатоцеребральной дистрофією B. метгемоглобіну
(хвороба Вільсона). У крові знижений C. карбгемоглобін
вміст церулоплазміну. У сечі різко D. оксигемоглобіном
підвищений вміст амінокислот. Ці E. Глікозильованого гемоглобіну
зміни в першу чергу обумовлені
посиленням процесу: 53. У хворого 37 років на тлі
A. * Комплексоутворення тривалого застосування антибіотиків
амінокислот з міддю. спостерігається підвищена
B. Синтезу сечовини. кровоточивість при невеликих
C. Ппереамінірованія амінокислот. пошкодженнях. У крові - зниження
D. разпад тканинних білків. активності факторів згортання крові
E. глюконеогенезі. II, VII, X, подовження часу згортання
крові. Недоліком якого вітаміну
50. Пацієнт звернувся в лікарню зі обумовлені зазначені зміни?
скаргами на загальну слабкість, ниючі A. * Вітаміну К.
болі в животі, поганий апетит, з B. Вітаміну А.
підозрою на жовтяницю. У сироватці C. Вітаміну С.
крові виявлено 77,3 мкмоль / л D. Вітаміну D.
загального білірубіну і 70,76 мкмоль / E. Вітаміну Е.
л кон'югованого білірубіну. Який
найбільш ймовірний вид жовтяниці? 54. Для лікування жовтяниці показано
A. * Механічна жовтяниця. призначення барбітуратів, які
B. Гострий гепатит. індукують синтез УДФ-
C. Цироз печінки. глюкуронілтрансферази. Лікувальний
ефект при цьому обумовлений D. Калію.
утворенням: E. Натрію.
A. * Прямого (кон'югованого)
білірубіну. 58. При укусі отруйної змії у людини
B. Непрямого (некон'югованій) може розвинутися гемолітична
білірубіну. жовтяниця. Вкажіть показник плазми
C. биливердина. крові, який підвищується у
D. протопорфирин. потерпілого в першу чергу.
E. Гема. A. * Білірубін непрямий
(неконьюгірованний).
55. У юнака 20 років діагностовано B. Білірубін прямий (кон'югований).
спадковий дефіцит УДФ- C. Сечова кислота.
глюкуронілтрансферази. Підвищення D. Сечовина.
якого показника крові підтверджує E. Вільні амінокислоти.
діагноз?
A. * непрямі (некон'югованого) 59. При обстеженні чоловіка 45 років,
білірубіну. що знаходиться довгий час на
B. Прямого (кон'югованого) вегетаріанської рослинній дієті,
білірубіну. виявлено негативний азотистий
C. Уробилин. баланс. Яка особливість раціону стала
D. Стеркобіліноген. причиною?
E. Тварину індікана. A. * Недостатня кількість білків.
B. Недостатня кількість жирів.
56. Для запобігання післяопераційної C. Надмірна кількість води.
кровотечі 6-річній дитині D. Надмірна кількість вуглеводів.
рекомендовано приймати вікасол, E. Недостатня кількість вітамінів.
який є синтетичним аналогом
вітаміну К. Вкажіть, які 60. Поряд з нормальними типами
Посттрансляційні зміни факторів гемоглобіну в організмі дорослої
згортання крові активується під людини можуть бути присутніми
впливом вікасолу. патологічні. Вкажіть один з них
A. * Карбоксилювання глутамінової A. * HbS.
кислоти. B. HbА1.
B. Фосфорилювання радикалів C. HbСО2.
серину. D. HbF.
C. Частковий протеолізу. E. HbA2.
D. Полімеризація.
E. Глікозилювання. 61. У медичній практиці для
профілактики алкоголізму широко
57. У хворого, який страждає використовують тетурам, який є
анемією, в еритроцитах підвищився інгібітором альдегіддегідрогенази.
вміст протопорфірину IX. Недолік Підвищення в крові якого метаболіту
якого мінерального елементу викликає відразу до алкоголю:
призвело до цієї патології? A. * ацетальдегід.
A. * Заліза. B. Етанолу.
B. Фосфору. C. малонового альдегіду.
C. Магнію. D. Пропіонова альдегіду.
E. метанол. B. Валін.
C. Серін.
62. Внаслідок посттрансляційних змін D. Фенілаланін.
деяких білків, які беруть участь в E. Аргінін.
згортанні крові, зокрема протромбіну,
вони набувають здатність зв'язувати 66. У немовляти внаслідок
кальцій. У цьому процесі бере участь неправильного годування виникла
вітамін: виражена діарея. Одним з основних
A. * К. наслідків діареї є екскреція великої
B. С. кількості бікарбонату натрію. Яка
C. А. форма порушення кислотно-лужного
D. В1. балансу має місце в цьому випадку?
E. В2. A. * Метаболічний ацидоз.
B. Метаболічний алкалоз.
63. У плазмі крові здорової людини C. Респіраторний ацидоз.
знаходиться декілька десятків білків. D. Респіраторний алкалоз.
При захворюванні організму E. Чи не буде порушень кислотно-
з'являються нові білки, зокрема "білок лужного балансу.
гострої фази". Таким білком є:
A. * С-реактивний білок. 67. До лікаря звернувся пацієнт зі
B. Протромбін. скаргами на задишку в стані спокою і
C. Фібриноген. при навантаженні. Лабораторне
D. імуноглобулінов G. дослідження крові виявило наявність
E. Імуноглобулін А. форми еритроцитів у вигляді серпа.
Як змінюється кількість
64. У хворого геморагічний інсульт. У оксигемоглобіну в крові і киснева
крові виявлено підвищена ємність?
концентрація кининов. Лікар A. * Зменшується кількість
призначив хворому контрикал. Для оксигемоглобіну і киснева ємність
гальмування який протеїнази було крові.
зроблено це призначення? B. Спостерігається збільшення
A. * КАЛІКРЕЇН. кількості гемоглобіну і кисневої
B. Пепсину. ємність крові.
C. трипсин. C. Не змінюється кількість
D. хімотрипсину. гемоглобіну і киснева ємність крові.
E. Колагенази. D. Кількість гемоглобіну не
65. Плазмові фактори згортання крові змінюється, а киснева ємність крові
відчувають посттрансляційної зростає.
модифікації з участю вітаміну К. Як E. Все невірно.
кофактор, він потрібен у ферментної
системі гамма-карбоксилювання 68. Мати звернулася до лікаря: у
білкових факторів згортання крові дитини 5 років під дією сонячних
завдяки збільшенню спорідненості їх променів на шкірі з'являються
молекул з іонами кальцію. Яка еритеми, везикулярна висип, дитина
амінокислота карбоксіліруется в цих скаржиться на свербіж. Аналіз крові
білках? виявив зменшення заліза в сироватці
A. * Глутаминовая. крові, збільшення виділення з сечею
уропорфіріноген. Найбільш E. Гема.
можливою спадковою патологією у
дитини є: 72. У хворого діагностовано бета-
A. * Ерітропоетіческая порфірія. таласемія. Вкажіть, які порушення
B. Метгемоглобінемія. спостерігаються в синтезі
C. Печінкова порфірія. гемоглобіну при цьому
D. копропорфірія. A. * Пригнічення синтезу бета-
E. Інтермітуюча порфірія. ланцюгів.
B. Пригнічення синтезу дельта-
69. У пацієнта візуально виявлено ланцюгів.
бульбашки і посилена пігментація C. Пригнічення синтезу гамма-
після дії УФ-променів. Сеча після ланцюгів.
відстоювання набуває червоного D. Пригнічення синтезу альфа-
кольору. Виявлення в сечі якого з ланцюгів.
перерахованих показників допоможе E. Пригнічення синтезу дельта- і бета-
діагностувати хворобу Гюнтера? ланцюгів.
A. * уропорфіріноген.
B. Гемоглобін. 73. Знешкодження хвороботворних
C. Білірубін. бактерій і розщеплення чужорідних
D. Креатинін. тіл в лейкоцитах здійснюється за
E. Ацетон. типом реакції окислення:
A. * пероксидазного.
70. 52-річну пацієнтку протягом B. оксидазний.
останніх декількох днів турбують C. оксігеназной.
напади болю в правому підребер'ї D. Перекисного.
після вживання жирної їжі. Візуально E. Анаеробного.
визначається пожовтіння склер і
шкіри, ахолічний кал, сеча "кольору 74. У 6-тимісячної дитини
пива". Присутність якої речовини в спостерігалися часті і сильні
сечі пацієнтки зумовило темне підшкірні кровотечі. Призначення
забарвлення сечі при обтурації, синтетичного аналога вітаміну K
A. * Білірубіндіглюкуроніда. (вікасолу) дало позитивний ефект. У
B. Кетонових тел. гамма-карбоксилювання глутамінової
C. Уробилин. кислоти якого з перерахованих нижче
D. стеркобіліна. білків системи згортання крові бере
E. Глюкози. участь цей вітамін?
A. * протромбіну.
71. У хворого, який страждає B. Фібриногену.
вродженою ерітропоетіческой C. Фактора Хагемана.
порфирией, відзначена D. антигемофільний глобуліну A.
світлочутливість шкіри. E. Фактора Розенталя.
Накопиченням якого з'єднання в
клітинах шкіри це можна пояснити? 75. У хворого виявлена
A. * уропорфіріноген I. серповидноклеточная анемія. Заміна
B. протопорфирин. якої амінокислоти в поліпептидному
C. уропорфіріноген Ш. ланцюгу Hb на валін призводить до
D. Копропорфіріногена Ш. цього захворювання?
A. * глутамінової кислоти. D. Кетоглутара
B. аспарагінова кислоти. E. аконітаза.
C. лейцин.
D. Аргініну. 80. Електрофоретичне дослідження
E. треоніну. сироватки крові хворого на
пневмонію показало збільшення
76. При обтураційній жовтяниці і однієї з білкових фракцій. Вкажіть її.
жовчних свищах часто A. * Гамма-глобуліни.
спостерігається протромбінового B. Альбуміни.
недостатність. З дефіцитом якого C. Альфа 1-глобуліни.
вітаміну це пов'язано? D. Альфа 2-глобуліни.
A. * К. E. Бета-глобуліни.
B. В6.
C. А ,. 81. При аналізі крові хворого
D. С. залишковий азот склав 48 ммоль / л,
E. Е. сечовина 15,3 ммоль / л. Про
захворювання якого органу свідчать
77. Хворому поставлений діагноз результати цього аналізу?
мегалобластична анемія. Вкажіть A. * Нирок.
з'єднання, недостатня кількість якого B. Печінки.
може призводити до розвитку цієї C. Шлунка.
хвороби. D. Кишечника.
A. * Ціанокобаламін. E. селезінки.
B. Гліцин.
C. Мідь. 82. Пролонгована дія ряду
D. Холекальциферол. антибіотиків і сульфаніламідів
E. Магній. обумовлено тим, що вони
циркулюють в крові тривалий час в
78. Яке похідне гемоглобіну комплексі з:
з'являється в крові при отруєнні A. * альбуміну.
чадним газом? B. трансферин.
A. * Карбоксигемоглобин. C. Гемоглобін.
B. Метгемоглобін. D. гаптоглобіну.
C. Оксигемоглобін. E. гемопексин.
D. карбгемоглобін.
E. Вердогемоглобін. 83. Пацієнт 33-річного віку хворіє 10
років. Періодично звертається до
79. При патологічних процесах, які лікаря зі скаргами на гострі болі в
супроводжуються гіпоксією, животі, судоми, порушення зору. У
відбувається неповне відновлення його родичів спостерігаються подібні
молекули кисню в дихальному симптоми. Сеча червоного кольору.
ланцюзі і накопичення пероксиду Госпіталізовано з діагнозом - гостра
водню. Вкажіть фермент, який переміжна порфірія. Причиною
забезпечує його руйнування захворювання можуть бути
A. * Каталаза. порушення біосинтезу:
B. Цитохромоксидази. A. * Гема.
C. Сукцинатдегідрогеназа. B. Інсуліну.
C. Жовчних кислот. концентрації якої речовини буде
D. простагландинів. спостерігатися в сироватці крові?
E. Коллагена. A. * Вільного білірубіну.
B. кон'югованих білірубін.
84. При хворобі Вільсона C. Мезобілірубіна.
(гепатоцеребральная дистрофія) в D. Вердоглобіна.
крові виявлено зниження вмісту E. биливердина.
церулоплазміну. Чим це обумовлено?
A. * Комплексоутворення 88. Хворому С. поставлений діагноз -
амінокислот з міддю. мієломна хвороба. Загальний білок
B. Розпадом тканинних білків. крові - 180 г / л. Такий рівень білка
C. декарбоксилюванню амінокислот. ймовірний за рахунок:
D. Синтезом сечовини. A. * Білка Бенс-Джонса.
E. Переамінування амінокислот. B. альбуміну.
C. гаптоглобіну.
85. Хворий 20 років скаржиться на D. імуноглобуліну.
загальну слабкість, швидку E. трансферина.
стомлюваність. При обстеженні
виявлено: гемоглобін крові 80 г / л, 89. Людина в стані спокою штучно
мікроскопічно виявлені еритроцити примушує себе дихати часто і
зміненої форми. Причиною може глибоко на протязі 3-4 хв. Як це
бути: відбивається на кислотно-лужній
A. * Серповидноклеточная анемія. рівновазі організму?
B. Паренхіматозна жовтяниця. A. * Виникає дихальний алкалоз.
C. Гостра переміжна порфірія. B. Виникає дихальний ацидоз.
D. Обтураційна жовтяниця. C. Виникає метаболічний алкалоз.
E. Хвороба Аддісона. D. Виникає метаболічний ацидоз.
E. Кислотно-лужна рівновага не
86. У хворого гострий напад змінюється.
жовчнокам'яної хвороби. Як це може
бути виявлено при лабораторному 90. У 70-ті роки вчені встановили, що
обстеженні? причиною важкої жовтяниці
A. * Негативна реакція на новонароджених є порушення
стеркобилин в калі. зв'язування білірубіну в гепатоцитах.
B. Позитивна реакція на стеркобилин Яка речовина використовується для
в калі. освіти кон'югату?
C. Наявність сполучної тканини в A. * Глюкуронова кислота.
калі. B. Сечова кислота.
D. Наявність неперетравленої C. Сірчана кислота.
клітковини в калі. D. Молочна кислота.
E. Наявність крохмальних зерен в E. Піровиноградна кислота.
калі.
91. У хворого цироз печінки з'явилися
87. * У пацієнта, який звернувся до набряки. Яка можлива причина їх
лікаря, спостерігається жовте появи?
забарвлення шкіри, сеча - темна, кал - A. * Зменшення вмісту альбумінів в
темно-жовтого кольору. Підвищення крові.
B. Зменшення вмісту в крові ланцюга. Який молекулярний
гаптоглобіну. механізм патології?
C. Збільшення вмісту в крові A. * Генна мутація.
трансферину. B. Хромосомная мутація.
D. Збільшення вмісту гамма- C. Дуплікація генів.
глобулінів в крові. D. Ампліфікація генів.
E. Зниження вмісту глюкози в крові. E. Трансдукція генів.

92. У хворого, який страждає 96. У хворого 27 років виявлені


стрептококової інфекцією, патологічні зміни печінки і головного
розвинувся геморагічний діатез. Яка мозку. У плазмі крові виявлено різке
причина підвищеної кровоточивості? зниження, а в сечі - підвищення
A. * Посилений фибринолиз. вмісту міді. Поставлено діагноз -
B. Недодостаток вітаміну А хвороба Вільсона. Активність якого
C. Збільшення кількості калликреина ферменту в сироватці крові необхідно
в плазмі крові. дослідити для підтвердження діагнозу
D. Збільшення кількості гепарину в ?:
плазмі крові. A. * Церулоплазміну.
E. Недодостаток вітаміну С. B. Карбоангідрази.
C. ксантиноксидазу.
93. У жінки 46 років, яка страждає на D. лейцінамінопептідази.
жовчнокам'яну хворобу, сеча стала E. алкогольдегідрогеназу.
темно-жовтого кольору, а кал -
знебарвлений. Вкажіть, концентрація 97. Пацієнт звернувся до лікаря зі
якої речовини в сироватці крові скаргами на задишку, яка виникала
зросте в найбільшій мірі після фізичного навантаження.
A. * коньюгированной білірубіну. Клінічне обстеження виявило анемію
B. Вільного білірубіну. та наявність парапротеина в зоні
C. биливердина. гамма-глобулінів. Який показник в
D. Мезобілірубіна. сечі необхідно визначити для
E. Уробіліноген. підтвердження діагнозу мієломи?
A. * Білок Бенс-Джонса.
94. У малюка, який народився 2 дні B. Білірубін.
тому недоношеним, спостерігається C. Гемоглобін.
жовте забарвлення шкіри і слизових D. Церулоплазмін.
оболонок. Вкажіть, тимчасовий E. антитрипсину.
недолік якого ферменту є причиною
цього стану малюка 98. У хворого встановлено
A. * УДФ- глюкуронілтрансферази. підвищення в плазмі крові кількості
B. Амінолевулінатсінтази. кон'югованого (прямого) білірубіну
C. Гем- оксигенази. при одночасному підвищенні
D. гемсінтетази. некон'югованого (непрямого) і
E. Білівердінредуктази. різкому зниженні в калі і сечі
кількості стеркобилиногена. Про який
95. Молекулярний аналіз гемоглобіну вид жовтяниці можна говорити?
пацієнта, який страждає анемією, A. * обтураційне.
показав заміну 6Глу на 6Вал в бета- B. Паренхіматозна (печінкової).
C. Гемолітичної. D. тромбопластином.
D. Жовтяниці немовлят. E. тромбін.
E. Хвороби Жильбера.
103. Жінка 62 років скаржиться на
99. У хворого на цукровий діабет часту біль в області грудної клітини
змінилося значення рН крові і стало та хребта, переломи ребер. Лікар
рівним 7,3. Визначення компонентів припустив мієломну хворобу
який буфферной системи (плазмоцити). Який з перерахованих
використовується для діагностики нижче лабораторних показників буде
розладів кислотно-лужної рівноваги? мати найбільше діагностичне
A. * бікарбонатному. значення? A. * парапротеінеміях.
B. фосфатних. B. Гіперальбумінемія.
C. гемоглобінового. C. Протеїнурія.
D. Оксігемоглобіновой. D. гіпоглобулінемія.
E. Білкової. E. Гипопротеинемия.

100. Які білки крові відповідають за 104. У чоловіка 70 років


підтримання колоїдно-осмотичного спостерігається підвищений рівень
тиску і постійний обсяг крові? згортання крові, обумовлений, в
A. * Альбуміни. першу чергу, зниженням в плазмі
B. Альфа1-глобуліни. крові:
C. Альфа2-глобуліни. A. * антитромбін.
D. Гамма-глобуліни. B. Альбуміну.
E. Бета-глобуліни. C. Імуноглобуліну А.
D. Калію.
101. У юнака 16 років діагностовано E. Вітаміну Д.
спадковий дефіцит УДФ-
глюкуронілтрансферази. Лабораторно 105. Мікроелемент мідь є складовим
визначається гіпербілірубінемія, компонентом білків металопротеінов.
обумовлена, переважно, підвищенням При порушенні обміну міді виникає
в крові концентрації: хвороба Вільсона -
A. * непрямого білірубіну. гепатоцеребральная дистрофія.
B. Прямого білірубіну. Концентрація якого білка
C. Уробіліноген. зменшується в крові?
D. Стеркобіліноген ,. A. * Церулоплазміну.
E. биливердина. B. трансферин.
C. феритин.
102. У жінки 32 років розвинувся D. Коллагена.
післяпологовий ДВЗ-синдром. При E. Глобуліну.
лабораторному обстеженні виявлено
підвищений рівень продуктів 106. Споживання забруднених овочів
деградації фібрин-фібриногену. Це, і фруктів протягом тривалого часу
перш за все, свідчить про підвищення призвело до отруєння пацієнта
активності: нітратами і утворення в крові
A. * плазміну. похідного гемоглобіну:
B. Aльфа- 2-макроглобуліну. A. * Hb-OH.
C. трансглутамінази. B. Hb СО.
C. Hb O2. D. Білка.
D. Hb CN. E. мевалонової кислоти.
E. Hb NHCOOH.
110. У хворого жовтяницею
107. Працівник цеху по виробництву встановлено: підвищення в плазмі
нітросполук звернувся до лікаря зі крові вмісту загального білірубіну за
скаргами на задишку і швидку рахунок непрямого (вільного), в калі і
стомлюваність. При обстеженні сечі - високий вміст стеркобилина,
хворого виявлено ціаноз нижніх рівень прямого (зв'язаного) білірубіну
кінцівок. Яка причина цього стану? в плазмі крові в межах норми. Про
A. * Посилення який вид жовтяниці можна
метгемоглобинообразование. стверджувати?
B. Гіповітаміноз. A. * Гемолітична.
C. Гіпервітаміноз. B. Паренхіматозна (печінкова).
D. Жирова інфільтрація печінки. C. Механічна.
E. Авітаміноз. D. Жовтяниця немовлят.
E. Хвороба Жильбера.
108. Хвора госпіталізована з
вираженою жовтяницею, скаргами на 111. Пацієнт звернувся в клініку зі
відчуття важкості в епігастральній скаргами на загальну слабкість, ниючі
ділянці та правому підребер'ї, свербіж болі в животі, поганий апетит, з
шкіри, стомлюваність, дратівливість. підозрою на жовтяницю. У сироватці
При лабораторному обстеженні крові знайдено 77,3 мкмоль / л
встановлено: гіпербілірубінемія з загального білірубіну і 70,76 мкмоль /
переважним підвищенням л коньюгированного білірубіну. Який
кон'югованого білірубіну. Який вид вид жовтяниці найбільш вірогідний?
жовтяниці найбільш вірогідний? A. * Механічна жовтяниця.
A. * Механіческоая жовтяниця, B. Гострий гепатит.
B. Гострий гепатит, C. Цироз печінки.
C. Гемолітична жовтяниця, D. Жовтяниця новонароджених.
D. Жовтяниця Крітлера-Кайяра, E. Гемолітична жовтяниця.
E. Цироз печінки.
112. У хворого, що приймає
109. Жінка 43 років, працівниця антикоагулянти непрямої дії,
лакофарбового підприємства, виявлено зниження рівня
скаржиться на загальну слабкість, протромбіну з 0,15г / л до 0,05 г / л,
зниження ваги, апатію, сонливість. який бере участь у другій фазі
Хронічна свинцева інтоксикація згортання крові - утворення тромбіну.
підтверджена лабораторно - виявлено Це сталося в результаті:
гіпохромна анемія. У крові A. * Нестачі вітаміну К.
підвищений рівень протопорфірину і B. Нестачі вітаміну В12.
знижений рівень дельта- C. Нестачі вітаміну С.
аминолевулиновой кислоти, які D. Зниження концентрації Са2 +.
свідчить про порушення синтезу: E. Зниження кількості глобуліну
A. * Гема. крові.
B. ДНК.
C. РНК.
113. У хворого А. після переливання знижується транспортування
крові спостерігається жовтушність гемоглобіном кисню від легенів до
шкіри і слизових оболонок, в крові тканин. Яке похідне гемоглобіну при
підвищені рівень загального і цьому утворюється?
непрямого білірубіну, в сечі A. * Карбоксигемоглобин.
підвищений рівень уробіліну, в калі - B. Оксигемоглобін.
рівень стеркобилина. Про який вид C. Метгемоглобін.
жовтяниці йдеться: D. карбгемоглобін.
A. * Гемолітичної жовтяниці. E. Гемохромоген.
B. Спадкової жовтяниці.
C. обтураційнійжовтяниці. 117. Після ремонту автомобіля в
D. паренхіматозноїжовтяниці. гаражному приміщенні водій
E. Жовтяниці новонародженого. потрапив до лікарні з симптомами
отруєння вихлопними газами.
114. У дитини протягом перших трьох Концентрація якого гемоглобіну в
місяців після народження крові буде підвищена?
розвинулася важка форма гіпоксії A. * карбоксигемоглобін.
(задуха, синюшність). Який з B. метгемоглобіну.
процесів гемоглобінообразованія C. карбгемоглобін.
порушений? D. оксигемоглобіном.
A. * Заміна Фет-гемоглобіну на E. Глікозильованого гемоглобіну.
гемоглобін А.
B. Заміна Фет-гемоглобіну на 118. Еритроцити людини не містять
гемоглобін S. мітохондрій. Який основний шлях
C. Заміна Фет-гемоглобіну на утворення АТФ в цих клітинах?
глікозильований гемоглобін. A. * гліколіз.
D. Заміна Фет-гемоглобіну на B. Аеробний гліколіз.
гемоглобін А2. C. Окислювальне фосфорилювання.
E. Заміна Фет-гемоглобіну на D. Креатінкіназная реакція.
гемоглобін М. E. Аденілаткіназная реакція.

115. У хворого виявлено зниження рН 119. У хворого 37 років на тлі


крові і вмісту бікарбонатних іонів тривалого застосування антибіотиків
(падіння лужного резерву крові), спостерігається підвищена
зростання вмісту молочної та кровоточивість при невеликих
піровиноградної кислот у крові та пошкодженнях. У крові - зниження
сечі. Який тип порушення кислотно- активності факторів згортання крові
лужної рівноваги спостерігається? II, VII, X, подовження часу згортання
A. * Метаболічний ацидоз. крові. Недостатність якого вітаміну
B. Респіраторний ацидоз. обумовлює зазначені зміни?
C. Метаболічний алкалоз. A. * Вітаміну К.
D. Респіраторний алкалоз ,. B. Вітаміну А.
E. Дихальний алкалоз. C. Вітаміну С.
D. Вітаміну D.
116. При підвищенні концентрації E. Вітаміну Е.
чадного газу в повітрі може
наступити отруєння. При цьому
120. У новонародженого фізіологічна D. Креатину.
жовтяниця. Рівень вільного білірубіну E. Церулоплазміну.
в крові значно перевищує норму.
Недоліком якого ферменту це 124. У хворого, який страждає
обумовлено? анемією, в еритроцитах збільшився
A. * Удф-глюкуронілтрансферази. вміст протопорфірину. Недодостаток
B. Трансамінази. якого мінерального елементу привів
C. ксантиноксидазу. до даної патології?
D. аденозіндезамінази. A. * Заліза.
E. Гем-оксигенази. B. Фосфору.
C. Магнію.
121. Для лікування жовтяниці D. Калію.
показано призначення барбітуратів, E. Натрію.
які індукують синтез УДФ-
глюкуронілтрансферази. Лікувальний 125.Ізвестно, що в слині курців
ефект при цьому обумовлений значно більше роданидов, ніж у
утворенням: некурящих. З надходженням який
A. * Прямого (коньюгированного) кислоти тютюнового диму це
білірубіну. пов'язано?
B. непрямі (неконьюгированного) A. * синильна.
білірубіну. B.Оцтовой.
C. биливердина. C.Азотной.
D. протопорфирин. D.Лімонной.
E. Гема. E.Нікотіновой.

122. У юнака 20 років діагностовано


спадковий дефіцит УДФ- Печінка
глюкуронілтрансфе-рази. Підвищення
якого показника крові підтверджує 1. У людини порушений процес
діагноз? синтезу сечовини. Про патології
A. * непрямі (неконьюгированного) якого органу це свідчить?
білірубіну. A. * Печінки.
B. Загального білірубіну. B. Нирок.
C. Уробилин. C. Мозку.
D. Стеркобіліноген. D. М'язів.
E. Тварину індікана. E. Сечового міхура.

123. Молодий чоловік після 2. Біологічне окислення і


імплантації серцевого клапана знешкодження ксенобіотиків
систематично отримує непрямі відбувається за рахунок
антикоагулянти. Його стан гемсодержащих ферментів. Який
ускладнилося кровотечею. Зі метал є обов'язковою складовою цих
зменшенням вмісту в крові якої ферментів?
речовини це пов'язано? A. * Fe.
A. * протромбіну. B. Zn.
B. гаптоглобіну. C. Co.
C. Гепарину. D. Mg.
E. Mn. в плазмі крові в межах норми. Про
який вид жовтяниці можна
3. Для визначення антитоксичної стверджувати?
функції печінки хворому призначений A. * Гемолітичної.
бензонат натрію, який в печінці B. Паренхіматозна (печінкової).
перетворюється в гиппуровую C. механічний.
кислоту. Яке з'єднання D. Жовтяниці немовлят.
використовується для цього процесу? E. Хвороби Жильбера.
A. * Гліцин.
B. Цистеин. 7. У недоношеного новонародженого
C. Метіонін. спостерігається жовте забарвлення
D. ФАФС. шкіри і слизових оболонок.
E. УДФ - Глюкуронова к-та. Ймовірною причиною такого стану
може бути тимчасовий недолік
4. У хворого цирозом печінки ферменту:
з'явилися набряки. Яка можлива A. * УДФ - глюкуронілтрансферази.
причина їх появи? B. Урідінтрансферази.
A. * Зменшення вмісту альбумінів в C. гемсінтетази.
крові. D. гемоксигенази.
B. Зменшення вмісту в крові E. Білівердінредуктази.
гаптоглобіну.
C. Збільшення вмісту в крові 8. У жінки 46 років, яка страждає
трансферину. жовчно - каменнуой хворобою,
D. Збільшення вмісту гамма- розвинулась жовтяниця. При цьому
глобулінів в крові. сеча стала темно жовтого кольору, а
E. Зниження вмісту глюкози в крові. кал - знебарвлений. Вкажіть,
концентрація якої речовини в
5. У хворого встановлено підвищення сироватці крові зросте в найбільшій
в плазмі крові вмісту кон'югованого мірі.
(прямого) білірубіну при A. * Коньюгованого білірубіну.
одночасному підвищенні B. Вільного білірубіну.
некон'югованого (непрямого) і C. биливердина.
різкому зниженні в калі і сечі вмісту D. Мезобілірубіна.
стеркобилиногена. Про який вид E. Уробіліноген.
жовтяниці можна стверджувати?
A. * обтураційне. 9. У чоловіка 32 років з ураженням
B. Паренхіматозна (печінкової). печінки при проведенні проби Квик
C. Гемолітичної. на детоксикаційну здатність
D. Жовтяниці немовлят. спостерігали низький рівень в сечі:
E. Хвороба Жильбера. A. * гиппуровую кислоти,
B. оксипроліну.
6. У хворого з жовтяницею C. Бензоату натрію,
встановлено: підвищення в плазмі D. Креатиніну,
крові вмісту загального білірубіну за E. Аминокислот.
рахунок непрямого (вільного), в калі і
сечі - високий вміст стеркобилина, 10. У новонародженого з'явилися
рівень прямого (зв'язаного) білірубіну ознаки жовтяниці. Введення
невеликих доз фенобарбіталу, який E. церулоплазміну.
індукує синтез УДФ-
глюкуронілтрансферази, призвело до 14. У хворого на цингу виявлено
поліпшення стану дитини. Який з порушення гідроксилювання проліну
перерахованих нижче процесів і лізину в складі колагену.
активується під впливом Гальмування якого біохімічного
індукованого фенобарбіталом процесу призводить до цього
ферменту? порушення?
A. * Кон'югація. A. * мікросомального окислення.
B. мікросомального окислення. B. Перекисного окислення ліпідів.
C. Тканинне дихання. C. Тканинного дихання.
D. Глюконеогенез. D. аеробне окислення глюкози.
E. Синтезу глікогену. E. Окисного фосфорилювання.

11. При аналізі крові у хворого 15. У пацієнта, який звернувся до


концентрація альбуміну становить 20 лікаря спостерігається жовте
г / л, підвищена активність забарвлення шкіри, сеча - темна, кал
ізоферменту лактатдегідрогенази 5 (темно - жовтого кольору.
[ЛДГ 5]. Про захворювання якого Підвищення концентрації якої
органу свідчить цей аналіз? речовини буде спостерігатися в
A. * Печінки. сироватці крові?
B. Нирок. A. * Вільного білірубіну
C. Серця. B .Кон'югірованного білірубіну
D. Легких. C .Мезобілірубіна
E. селезінки. D. Вердоглобіна
E .Білівердіна
12. Хворому, що страждає на
хронічний гепатит, для з'ясування 16.Женщіна 46-ти років, страждає
знешкоджуючих функції печінки була жовчно -камінь хвороб, розвинулася
проведена навантаження бензоатом желтуха.Прі цьому сеча стала темно -
натрію. За виділення якої речовини з жовтого кольору, а кал -
сечею судять про знешкоджуючих знебарвлений. Вкажіть, концентрація
функції печінки? якої речовини в сироватці крові
A. * гиппуровую кислоти. зросте в найбільшій мірі?
B. фенілоцтової кислоти. A. * Кон'югованих білірубін.
C. Лимонної кислоти. B .Свободного білірубіну.
D. валеріанових кислоти ,. C. биливердина.
E. Щавлевої кислоти. D. Мезобілірубіна.
E. Уробіліноген.
13. Вкажіть, з яким білком зв'язується
гемоглобін для переносу в 17. У пацієнта цироз печінки.
ретикулоендотеліальної системи Дослідження яких з перерахованих
печінки? речовин, що екскретуються з сечею,
A. * гаптоглобіну. може характеризувати стан
B. альбуміну. антитоксичної функції печінки?
C. феритин. A. * Гиппуровую кислоти.
D. трансферина. B .Аммонійних солей.
C. Креатиніну. B .УДФ -глюкозопірофосфорілази.
D. Січовий кислоти. C .УДФ - глікогентрансферази.
E. Аминокислот. D .Орнітінкарбамоілтрансферази.
E. фосфорібозілпірофосфат -
18. У хворого після переливання крові амідотрансферази
спостерігається жовтушність шкіри і
слизових оболонок, в крові 22. При ферментативних желтухах
підвищений рівень загального та має місце порушення активності
непрямого білірубіну, в сечі ферменту УДФ -
підвищено рівень уробіліну, і калі - глюкуронілтрасферази. Вкажіть, яке
стеркобилина. Який вид жовтяниці у з'єднання накопичується в сироватці
хворого? крові при цих патологіях.
A. * Гемолітична. A. * Непрямий білірубін.
B. Спадкова. B .Прямой білірубін.
C. Обтураційна. C. биливердина.
D. Паренхіматозна. D. Мезобілірубін.
E .Желтуха новонароджених. E. Вердоглобін.

19. Для визначення антитоксичної 23. У клініку поступив хворий з


функції печінки хворому призначений яскраво вираженою желтушностью
бензоат натрію, в печінці шкіри, склер, слизових оболонок.
перетворюється в гиппуровую Аналіз сечі показав наявність в ній
кислоту. Яке з'єднання прямого білірубіну. Сеча кольору
використовується для цього процесу? темного пива, кількість жовчних
A. * Гліцин. пігментів в калі знижено. Який тип
B. Цистеин. жовтяниці спостерігається у хворого?
C. Метіонін. A. * Обтурационная.
D .ФАФС. B. Паренхіматозна.
E .УДФ - Глюкуронова к-та. C. Гемолітична.
D. Кон'югаційна.
20.У дитини, яка народилася 2 дні E. Абсорбційна.
тому, спостерігається жовте
забарвлення шкіри і слизових 24. 52- річну хвору вже кілька днів
оболонок. Причиною такого стану є турбують напади болю в правому
тимчасова нестача ферменту: підребер'ї після вживання жирної їжі.
A. * УДФ- глюкуронілтрансферази. Візуально відзначається жовтушність,
B .Сульфотрансферази. склер і шкіри. Ахолічний кал, сеча
C .Гемсінтетази. "кольору пива". Присутність якої
D .Гемоксігенази. речовини в сечі хворого призводить в
E .Білівердінредуктази. темний колір, сечу при обтураційній
жовтяниці?
21. У пацієнта з генетичної A. * Прямий білірубін.
ензимопатіях (хвороба Жильбера) B. Кетонові тіла.
збуджена кон'югації білірубіну в C. Непрямий білірубін.
печінці. Синтез якого ферменту D. Стеркобілін.
заблокований у цього пацієнта? E. білірубінглюкуроніди.
A. * УДФ- глюкуронілтрансферази.
25. У хворого з яскраво вираженою A. * Дефіцит вітаміну К.
желтушностью шкіри, склер, B. Тромбоцитопенія.
слизових оболонок сеча має колір C. Еритропенія.
темного пива, кал світлий. У крові D. Лейкопенія.
підвищений вміст прямого білірубіну, E. Дефіцит заліза.
в сечі визначається білірубін. Тип
жовтяниці у хворого? 29.В 70-ті роки вчені встановили, що
A. * Обтурационная. причиною важкої жовтяниці
B. Паренхіматозна. новонароджених є порушення
C. Гемолітична. зв'язування білірубіну в гепатоцитах.
D. Кон'югаційна. Яка речовина використовується для
E. Екскреційна. освіти коньюгата?
A. *. Глюкуронова кислота.
26. У хворого з жовтяницею в крові B.Мочевая кислота.
виявлено збільшення загального C.Серная кислота.
білірубіну за рахунок непрямої його D.Молочная кислота.
фракції. Сеча і кал інтенсивно E.Піровіноградная кислота.
пофарбовані. Який найбільш
ймовірний механізм зазначених 30.У хворого, виснаженого
порушень? голодуванням, в печінці та нирках
A. * Підвищений гемоліз еритроцитів. посилюється процес?
B. Порушення перетворення A. * глюконеогенезі.
уробіліногену в печінці. B.Сінтеза сечовини.
C. Складне Становище відпливу C.Сінтеза білірубіну.
жовчі з печінки. D.Образованія гіпуровой кислоти.
D. Порушення освіти прямого E.Сінтеза сечової кислоти.
білірубіну.
E. Пошкодження паренхіми печінки.
Травлення
27. При ферментативних желтухах
має місце порушення активності 1. У хворого поганий апетит,
ферменту УДФ- відрижка. Загальна кислотність
глюкуронілтрансферази. Яке шлункового соку становить 10
з'єднання накопичується в сироватці одиниць. Такий стан може свідчити
крові при цих патологіях? про:
A. * Прямий білірубін. A. * гіпоацидний гастрит.
B. биливердина. B. гіперацидний гастрит.
C. Вердоглобін. C. Остром панкреатиті.
D. Мезобілірубін. D. антацидний гастрит.
E. Непрямий білірубін. E. Виразкова хвороба шлунка.

28. При захворюваннях печінки, що 2. У хворого в порції шлункового


супроводжуються недостатнім соку виявлено лактат. Поясніть його
надходженням жовчі в кишечник, появи.
спостерігається погіршення A. * Брак HCl.
гемокоагуляції. Чим можна пояснити B. Надлишок HCl.
це явище? C. Нестача пепсину.
D. Нестача гастріксіна. соку загальна кислотність 20 од.
E. Нестача реніну. Травлення яких компонентів їжі
порушено в першу чергу?
3. У хворого в крові і сечі виявлено A. * Білків.
високий вміст індікана - показника B. Фосфоліпідів.
активації процесів гниття білків в C. Нейтральних жирів.
кишечнику. Яка амінокислота є D. олігосахариди.
джерелом індікана? E. Крохмалю.
A. * Триптофан.
B. Тирозин. 8. Після прийому жирної їжі хворий
C. Пролин. відчуває дискомфорт, а в калі
D. Фенілаланін. неперетравлені краплі жиру. Реакція
E. Гістидин. сечі на жовчні кислоти позитивна.
Причиною такого становища є
4. У пацієнта 40 років недолік:
непереносимість вуглеводів молока. A. * Жовчних кислот.
Недостатністю якого ферменту B. Жирних кислот.
травлення можна пояснити це явище? C. хиломикрон.
A. * лактази. D. Тригліцеридів.
B. Лактатдегідрогенази. E. Фосфоліпідів.
C. мальтази.
D. Ліпази. 9. У новонародженої дитини після
E. Амілази. годування молоком спостерігалися
диспепсичні розлади (диспепсія,
5. Яка речовина надає слині в'язкий, блювота). При годуванні розчином
слизовий характер, виконує захисну глюкози ці явища зникали. Вкажіть
роль, попереджають слизові ротової фермент, який бере участь у
порожнини від механічного перетравлюванні вуглеводів,
пошкодження? недостатня активність якого
A. * Муцин. призводить до вказаних розладів.
B. Глюкоза. A. * Лактаза.
C. калікреїн. B. Амілаза.
D. Амілаза. C. сахараза.
E. Лізоцим. D. ізомальтаза.
6. У слині міститься фермент, який E. Мальтаза.
здатний руйнувати альфа-1, 4 -
гликозидні зв'язку в молекулі 10. Перетравлення білків в шлунку є
крохмалю. Вкажіть цей фермент. початковою стадією розщеплення
A. * Альфа-амілаза. білків в травному каналі людини.
B. Фосфатаза. Назвіть ферменти, що беруть участь в
C. фруктофуранозидази. перевареваніі білків в шлунку:
D. Бета-галактозидаза. A. * Пепсин і гастриксин.
E. Лізоцим. B. Трипсин і катепсини.
C. Хімотрипсин і лізоцим.
7. Хворий скаржиться на зниження D. ентеропептідази і еластаза.
ваги, болі в області шлунка після E. Карбоксипептидаза і
прийому їжі, при аналізі шлункового амінопептідазу.
A. * Про інтенсивність гниття білків в
11. В організмі людини хімотрипсин кишечнику.
секретується підшлунковою залозою і B. Про зниження активності
в порожнині кишечника піддається ферментних систем орнітінового
обмеженому протеолізу з циклу.
перетворенням в активний C. Про швидкості окисного
хімотрипсин під дією: дезамінування ароматичних
A. * трипсину. аммінокіслот.
B. Ентерокінази. D. Про інтенсивність знешкодження
C. Пепсину. аміаку.
D. амінопептидази. E. Про стан клубочковоїфільтрації
E. Карбоксипептидаза. нирок.

12. У добовому раціоні дорослої 15. До лікарні поступив хворий зі


здорової людини повинні бути жири, скаргами на здуття живота, діарею,
білки, вуглеводи, вітаміни, мінеральні метеоризм після вживання білкової
солі і вода. Вкажіть кількість білка, їжі, що свідчить про порушення
яке забезпечує нормальну травлення білків і їх посиленого
життєдіяльність організму. гниття. Вкажіть, яка з перерахованих
A. * 100 - 120. речовин є продуктом цього процесу в
B. 50 - 60. кишечнику.
C. 10 - 20. A. * Індол.
D. 70 -80. B. Білірубін.
E. 40 - 50. C. Кадаверин.
D. агматину.
13. Хворого з явищами енцефалопатії E. Путрецін.
госпіталізували в неврологічний
стаціонар і виявили кореляцію між 16.При споживанні їжі, що містить
наростанням енцефалопатії і білок глютен, розвивається целіакія
речовинами, які надходять з (глютенова хвороба), яка
кишечника в загальний кровотік. Які характеризується дегенерацією
сполуки, що утворюються в кишкових ворсинок з втратою їх
кишечнику, можуть викликати абсорбтівноі функції, діареєю і
ендотоксімію? стеаторея, здуттям живота, втратою
A. * Індол. ваги і іншими позакішечніми
B. Бутират. проявами. Глютен є білком:
C. Ацетоацетат. A. * Пшениці.
D. Біотин. B .Кукурузи.
E. Орнитин. C Рис.
D. Суниці.
14. У хворого, прооперованого з E. Яєць.
приводу "гострого живота", сеча
набула коричневий колір, кількість 17.У новонародженої дитини в
індікана в сечі перевищило 90 мкмоль шлунку відбувається "створожіваніе"
/ добу. За кількістю індікана в сечі молока, тобто перетворення
людини можна судити: розчинних білків козеін в нерозчинні
- параказеіна за участю іонів кальцію
і ферменту. Який фермент приймає B.Тріпсіноген.
участь в цьому процесі? C.Проеластаза.
A. * Ренін. D.Прокарбоксіпептідаза.
B.Пепсін. E.Хімотріпсіноген.
C.Гастрін.
D.Секретін. 22.У чоловіка 60 років, який страждає
E.Ліпаза. на хронічну непрохідністю
кишечника, посилюється гниття
18.Недостаточная секреція якого білків в товстому кишечнику.
ферменту обумовлює неповне Підтвердженням цього процесу є:
перетравлювання жирів в шлунково-
кишковому тракті і поява великої A. * Індіканурія.
кількості нейтральних жирів в B. білірубінурія.
калових масах? C. Гіперурикурія.
A. * панкреатичної ліпази. D. креатинурія.
B.Фосфоліази. E .Глюкозурія.
C.Етерокінази.
D.Амілази. 23.Прі обстеженні чоловіка 45-ти
E.Пепсіна. років, який знаходиться довгий час на
вегетаріанської рослинній дієті,
19.У хворого посилено гниття білків виявлено негативний азотистий
в кишечнику. Як знешкоджуються баланс. Яка особливість раціону стала
токсичні продукти цього процесу? причиною цього?
A. * Реакцією кон'югації. A. * Недостатня кількість білків.
B.Частічним протеолізом. B. Недостатня кількість жирів.
C.Гідролізом. C. Надмірна кількість води.
D.Ізомерізаціей. D. Надмірна кількість вуглеводів.
E.Сольватаціей. E. Недостатня кількість вітамінів.

20.У хворого на хронічний гепатит


виявлено значне зниження синтезу і Сполучна тканина
секреції жовчних кислот. Який
процес в кишечнику порушений у 1. У сечі хворого виявлено
цього хворого? оксипролін і пролін в підвищених
A. * Емульгування жирів. концентраціях. Порушення
B. Тралення білків. метаболізму якого білка можна
C. Травлення вуглеводів. припустити у даного хворого?
D. Всмоктування гліцерину. A. * Коллагена.
E. Всмоктування амінокислот. B. Гемоглобіну.
C. міозину.
21.Прі механічної жовтяниці D. Фібриногену.
порушуються процеси перетравлення E. протромбіну.
в кишечнику через відсутність
жовчних кислот. Який панкреатичний 2. У хворого на цингу виявлено
фермент активується цими порушення гідроксилювання проліну
кислотами? і лізину в складі колагену.
A. * Ліпаза. Гальмування якого біохімічного
процесу приводити до цього Який етап модифікації проколагену
порушення? порушений при цьому авітамінозі?
A. * мікросомального окислення. A. * Гідроксилювання пролина.
B. Перекисного окислення ліпідів. B. Освіта поліпептидних ланцюгів.
C. Тканинного дихання. C. Глікозилювання гідроксілізінових
D. пероксидазного окислення. залишків.
E. Окисного фосфорилювання. D. Видалення з проколагену С-
кінцевого пептиду.
3. До косметолога звернувся пацієнт з E. Відщеплення N- кінцевого
проханням позбавити його від пептиду.
татуювання на плечі. Яка речовина,
що міститься в сполучній тканині, 7. Гіповітаміноз С призводить до
обмежує поширення барвника і зменшення утворення органічного
робить можливим такий вид матриксу та порушення синтезу
"живопису"? колагену, так як цей вітамін бере
A. * Гіалуронова кислота. участь в процесах:
B. Гамма-глобулін. A. * гідроксилювання проліну.
C. Фібронектин. B. карбоксилирование пролина.
D. Гепарин. C. карбоксилирование лізину.
E. еластин. D. гідроксилюванні аргініну.
E. гідроксилювання триптофану.
4. При парадонтозі відбувається
деструкція білкових і полісахаридних 8. Гідроксипролін є важливою
компонентів сполучної тканини. Який амінокислотою у складі колагену. За
з наведених білків входить до складу участю якого вітаміну відбувається
сполучної тканини? утворення цієї амінокислоти шляхом
A. * Колаген. гідроксилювання проліну?
B. Альбумін. A. * C.
C. трансферина. B. D.
D. церрулоплазміна. C. B1.
E. антитрипсину. D. B2.
E. B6.
5. Відомо, що синовіальна рідина
зменшує тертя суглобових поверхонь. 9. Після загоєння рани на її місці
При ревматизмі або артриті її утворився рубець. Яка речовина є
в'язкість знижується внаслідок основним компонентом цього
деполімеризації (руйнування): різновиду сполучної тканини?
A. * гіалуроновоі кислоти. A. * Колаген.
B. Глікогену. B. еластин.
C. колагену. C. Гіалуронова кислота.
D. Гепарину. D. Хондроітінсульфат.
E. Альбуміну. E. кератансульфатів.

6. Підвищена крихкість судин, 10. У хворих колагенозом має місце


руйнування емалі та дентину зубів процес деструкції сполучної тканини.
при цинзі багато в чому обумовлені Збільшення вмісту яких сполук в
порушенням дозрівання колагену. крові це підтверджує?
A. * Змісту оксипролина і оксилізину B. индикан.
в крові. C. Креатиніну.
B. Змісту креатину і креатиніну. D. Сечовини.
C. Активності ізоферментів ЛДГ. E. Уробіліноген.
D. Активності трансаміназ.
E. Рівня уратів в крові. 15. У дитини спостерігається
затримка фізичного і розумового
11. У чоловіка 53 років діагностовано розвитку, глибокі порушення з боку
хворобу Педжета. У сечі різко сполучної тканини внутрішніх
підвищений рівень оксипроліну, що органів, в сечі виявлено
свідчить перш за все про посилення кератансульфатів. Обмін яких
розпаду: речовин порушений:
A. * Коллагена. A. * глікозаміногліканами.
B. кератину. B. Коллагена.
C. Альбуміну. C. еластину.
D. Гемоглобіну.
E. Фібриногену. 16. У сироватці крові пацієнта
виявлено підвищення концентрації
12. При обстеженні хворого виявлено оксипроліну, сіалових кислот, С-
характерна клініка коллагенозов. реактивного білка. Загострення якої
Вкажіть, збільшення якого показника патології найімовірніше у даного
сечі характерне для цієї патології. пацієнта?
A. * гідроксипролін. A. * Ревматизму.
B. Аргініну. B. ентероколіти.
C. Глюкози. C. Гепатиту.
D. Мінеральних солей. D. Бронхіт.
E. Солей аммоніяю E. панкреатиту.

13. Жінка 30 років хворіла близько 17. У сироватці крові пацієнта


року, коли вперше з'явилися болі в встановлено підвищення активності
області суглобів, їх припухлість, гіалуронідази. Визначення якого
почервоніння шкіри над ними. біохімічного показника сироватки
Попередній діагноз ревматоїдний крові дозволить підтвердити
артрит. Однією з ймовірних причин припущення про патологію сполучної
цього захворювання є зміна в тканини?
структурі білка сполучної тканини: A. * сіалові кислоти.
A. * Коллагена. B. Білірубіну.
B. муцини. C. Січовий кислоти.
C. міозину. D. Глюкози.
D. Овоальбуміна. E. галактоза.
E. тропонин.
18. У процесі старіння організму
14. Хвора 36-ти років страждає зменшується зв'язування води
колагенозом. Збільшення вмісту сполучною тканиною. Це пов'язано зі
якого метаболіту найімовірніше буде зменшенням концентрації:
встановлено в сечі? A. * глікозаміногліканами.
A. * оксипроліну. B. Коллагена.
C. Фосфоліпідів. E. протромбіну.
D. Гиалуроновой кислоти.
E. хондроїтинсірчаної кислоти. 23. У хворого з частими кровотечами
з внутрішніх органів і слизових
19. При обстеженні хворого виявлено оболонок виявлені пролін і лізин у
характерна клініка колагенозу. складі колагенових волокон. Через
Вкажіть, збільшення якого показника відсутність якого вітаміну порушено
сечі характерно для цієї патології їх гідроксилювання?
A. * гідроксипролін. A. * Вітаміну С.
B. Аргініну. B. Вітаміну К.
C. Глюкози. C. Вітаміну А.
D. Мінеральних солей. D. Тіаміну.
E. Солей амонію. E. Вітаміну Е.

20. У хворого з частими кровотечами 24. Хвора 36 років страждає


у внутрішні органи і слизові оболонки колагенозом. Збільшення вмісту якого
в складі колагенових волокон метаболіту найбільш вірогідно буде
виявили пролін і лізин. Відсутність встановлено в сечі?
якого вітаміну призводить до A. * оксипроліну.
порушення їх гідроксилювання? B. индикан.
A. * Вітаміну С. C. Креатиніну.
B. Вітаміну Е. D. Сечовини.
C. Вітаміну К. E. Уробіліноген.
D. Вітаміну А.
E. Вітаміну Д. 25. У жінки 63 років є ознаки
ревматоїдного артриту. Підвищення
21. Відомо, що молекула колагену рівня якого з перерахованих нижче
містить амінокислоти оксипроліну, показників крові буде найбільш
оксілізін. Які з перерахованих значущим для підтвердження
речовин беруть участь в діагнозу?
гидроксилировании пролина і лізину A. * Сумарних гликозаминогликанов.
під час синтезу колагену? B. ліпопротеїнів.
A. * Аскорбінова кислота. C. Кислого фосфатази.
B. Фолієва кислота. D. Загального холестерину.
C. Пантотеновакислота. E. R-глікозідази.
D. Глутаминовая кислота.
E. Аспарагінова кислота. 26. У хворого спостерігаються часті
кровотечі з внутрішніх органів,
22. У сечі хворого виявлені слизових оболонок. Аналіз виявив
оксипроліну і пролін в підвищених недостатність гідрооксіпроліна і
концентраціях. Порушення гідроксилізин в складі колагенових
метаболізму якого білка можна волокон. При нестачі якого вітаміну в
припустити у даного хворого? організмі порушуються процеси
A. * Коллагена. гідроксилювання названих
B. Гемоглобіну. амінокислот?
C. міозину. A. * Вітаміну С.
D. Фібриногену. B. Вітаміну А.
C. Вітаміну Н. в'язкість сніжаеться внаслідок
D. Вітаміну К. деполімеризації такого речовини:
E. Вітаміну РР. A. * Гиалуроновой кислоти.
B.Глікогена.
27. До фибрилярную елементів C.Колагена.
сполучної тканини належать колаген, D.Гепаріна.
еластин і ретикулін. Вкажіть E.Альбуміна.
амінокислоту, яка входить тільки до
складу колагену і визначення якої в 31.У хворого з нирковою
біологічних рідинах недостоточностью виявлено різке
використовується для діагностики зниження вмісту натрію в сироватці
захворювань сполучної тканини крові. Відзначаються бліді пухкі
A. * Гідроксипролін. набряки обличчя, з'являються в
B. Пролін. ранковий час. Яка речовина, що
C. Гліцин. входить в міжклітинний матрикс
D. Лізин. сполучної тканини, зв'язує іони
E. Фенілаланін. натрію, що надходять з кров'яного
русла?
28. У вагітної в віці 28 років, A. * Гіалуронова кислота.
досліджували ферменти в клітинах B.Коллаген.
амніотичної рідини. При цьому C.Еластін.
виявилася недостатня активність D.Проколлаген.
бета-глюкуронідази. Який E.Фібронектін.
патологічний процес спостерігається
A. * Мукополісахарідоз. 32. У хворого, який страждає на
B. Глікогеноз. цингу, порушені процеси утворення
C. аглікогенози. сполучної тканини, що призводить до
D. колагеноза. розхитування і випадання зубів.
E. ліпідозах Порушення активності якого
ферменту викликає ці симптоми?
29.Наследственние захворювання - A. * Лізілгідроксілаза. .
мукополісахаридози проявляються B. глікозілтрансферази.
порушенням обміну в сполучній C. Прокаллагенпептідаза С-кінцевого
тканині, потологии суглобів і кісток. пептиду
Який показник аналізу сечі D. Еластаза.
свідетельствет про наявність такої E. Проколлагенпептідаза N-кінцевого
патології? пептиду.
A. * Надмірна секреція
гликозаминогликанов. 33.Прі остеолатерізме зменшується
B.Чрезмерная екскреція амінокислот. міцність колагену, обумовлена
C.Чрезмерная екскреція ліпідів. помітним зменшенням освіти
D.Чрезмерная екскреція глюкози. поперекових зшивок в колагенових
E.Чрезмерная екскреція альбуміну. фібрила. Причиною цього явища є
зниження активність:
30.Ізвестно, що синовіальна рідина A. * Лізілоксідази.
зменшує тертя суглобових поверхонь. B. моноаміноксидази.
При ревматизмі або артриті її C. Пролілгідроксілази.
D .Лізілгідроксілази. формуванні хрящової і кісткової
E. Колагенази. тканини:
A. * Хондроітінсульфат.
34.К лікаря звернувся юнак зі B.Гіалуроновая кислота.
скаргами, які можна розцінювати як C.Дерматансульфат.
симптоми активної форми D .Кератансульфат.
ревматизму - пошкодження сполучної E.Гепарін.
тканини, що супроводжується
руйнуванням гетеро полісахаридів в 38. У хворого з системною
складі глікопротеїнів. Які біохімічні склеродермією посилений розпад
показники крові та сечі необхідно колагену. Посилення екскреції з
досліджувати для уточнення сечею якої амінокислоти
діагнозу? відображатиме процеси деструкції
A. * сіалові кислоти. колагену?
B. Глюкозу. A. оксипроліну.
C. Кетонові тіла. B. Серін.
D .Індікан. C. Аланин.
E. Білкові фракції. D. Триптофан.
E. Фенілаланін.
35.У вагітної у віці 28 років,
досліджували ферменти в клітинах Нуклеотиди
амніотичної рідини, при цьому
виявилася недостатня активність бета 1. Пацієнт, 46 років, звернувся до
- глюкуронідази. Який патологічний лікаря зі скаргами на болі в малих
процес спостерігається? суглобах ніг і рук. Суглоби збільшені,
A. * Мукополісахародози. мають вигляд потовщених вузлів. У
B .Глікогенози. сироватці встановлено підвищений
C. аглікогенози. вміст уратів. Причиною цього може
D. Колагенози. бути:
E. ліпідозах. A. * Порушення обміну пуринів.
B. Порушення обміну вуглеводів.
36.Агрегати муцина затримують воду, C. Порушення обміну ліпідів.
що забезпечу¹ добрі їх в'язкість і D. Порушення обміну піримідинів.
захисну дію. Це забезпечу¹ добрі їх E. Порушення обміну амінокіслот.
в'язкість і захисне дейстівія. Це
можливо тому, що до складу 2. Хворому на подагру лікар
структури муцину входять: призначив алопуринол, що призвело
A. * Глікозаміноглікани. до зменшення концентрації сечової
B.Гомополісахаріди. кислоти. Яка властивість
C.Дісахаріди. алопуринолу забезпечує
D.Олігосахаріди. терапевтичний ефект в даному
E.Глюкоза. випадку?
A. * Конкурентне інгібування
37.Какие глікозаміноглікан є ксантиноксидази.
найбільш типовим для кісткової B. Збільшення швидкості виведення
тканини і виконує головну функцію в азотовмісних речовин.
C. Прискорення катаболізму сечі. Вкажіть, при розпаді яких
піримідинових нуклеотидів. речовин утворюється сечова кислота?
D. Уповільнення реутилізацію A. * пуринових нуклеотидів.
піримідинових нуклеотидів. B. піримідинових нуклеотидів.
E. Прискорення синтезу нуклеїнових C. Аминокислот.
кислот. D. Білків.
E. хромопротеїни.
3. Для лікування злоякісних пухлин
призначають метотрексат - 8. Чоловік 55 років, що страждає
структурний аналог фолієвої кислоти, болями в області нирок, поступив до
який є конкурентним інгібітором лікарні. При ультразвуковому
дигідрофолатредуктази і тому обстеженні пацієнта виявлено
пригнічує синтез: наявність ниркових каменів.
A. * нуклеотидів ДНК. Наявність якої речовини в сечі є
B. Моносахаридів. найбільш імовірною причиною
C. Жирних кислот. утворення каменів у даного пацієнта?
D. гліцерофосфатіди. A. * Січовий кислоти.
E. Глікогену. B. Білірубіну.
C. биливердина.
4. Для лікування подагри хворому D. уробіліну.
призначили алопуринол, структурний E. Креатиніну.
аналог гіпоксантину, що призвело до
зростання екскреції останнього з 9. Чоловік 46 років звернувся до
сечею. Який процес блокується при лікаря зі скаргами на болі в дрібних
цьому лікуванні? суглобах, які загострилися після
A. * Освіта сечової кислоти. вживання м'ясної їжі. У хворого
B. Запасний шлях синтезу пуринових діагностовано сечокам'яна хвороба з
нуклеотидів. накопиченням сечової кислоти.
C. Основний шлях синтезу пуринових Цьому пацієнту призначено
нуклеотидів. алопуринол, який є конкурентним
D. Синтез сечовини. інгібітором ферменту:
E. Розпад піримідинових нуклеотидів. A. * ксантиноксидазу.
B. Уреаза.
6. У хворого збільшені і болючі C. аргінази.
суглоби, а в сироватці крові D. Дігідроурацілдегідрогенази.
підвищений вміст уратів. Обмін яких E. Карбамоілсінтетази.
речовин порушений?
A. * Пуринів. 10. Чоловік 52 років звернувся до
B. піримідиніл. лікаря зі скаргами на ураження
C. Холестерину. суглобів. Був поставлений діагноз -
D. фенілаланіну. подагра. Порушення обміну якого
E. Гліцерину. ферменту призводить до розвитку цієї
патології?
7. Чоловік 58 років звернувся до A. * ксантиноксидазу.
лікаря зі скаргою на біль в суглобах. B. сукцинатдегідрогенази.
При обстеженні виявлено підвищення C. піруватдегідрогенази.
концентрації сечової кислоти в крові і D. епімерази.
E. Трансамінази. B. Розпаду піримідинових
нуклеотидів.
11. Механізм дії поширеного C. Синтезу пуринових нуклеотидів.
протипухлинного препарату D. Розпаду пуринових нуклеотидів.
метотрексату полягає в тому, що він є E. Перетворення рибонуклеотидов в
структурним аналогом: дезоксирибонуклеотидів.
A. * Фолієвої кислоти.
B. Віт. РР (нікотинової кислоти). 15. Людина 52 років звернувся до
C. параамінобензойноїкислоти. лікаря зі скаргами на ураження
D. Ціанкобаламін. суглобів. Був поставлений діагноз
E. ретиноевую кислоти. подагра. Порушення обміну якого
ферменту призводить до розвитку цієї
12. Похідні птерину - аміноптерин і патології?
метотрексат - є конкурентними A. * ксантиноксидазу.
інгібіторами дигідрофолатредуктази, B. сукцинатдегідрогенази.
в результаті чого вони пригнічують C. піруватдегідрогенази.
регенерацію тетрагідрофолієвої D. епімерази.
кислоти з дигідрофолат. Ці лікарські E. Трансамінази.
засоби призводять до гальмування 16. У клініку поступив хворий з
міжмолекулярної транспорту підозрою на подагру. Який
одноуглеродних груп. Біосинтез якого біохімічний аналіз слід призначити
полімеру при цьому пригнічується? для уточнення діагнозу?
A. * ДНК. A. * Визначення сечової кислоти в
B. Білок. крові і сечі.
C. Гомополісахариди. B. Визначення сечовини в крові і сечі.
D. Гангліозид. C. Визначення креатину в крові.
E. Глікозаміноглікани. D. Визначення активності уриказа в
крові.
13. У хворої суглоби збільшені, E. Визначення амінокислот в крові.
болять. У крові пацієнтки підвищений
рівень уратів. Як називається така 17. На основі лабораторного аналізу у
патологія? хворого підтверджено діагноз -
A. * Подагра. подагра. Який аналіз був проведений
B. Рахіт. для постановки діагнозу?
C. Скорбут. A. * Визначення сечової кислоти в
D. Пелагра. крові і сечі.
E. Карієс. B. Визначення креатиніну в сечі.
C. Визначення залишкового азоту в
14. У дитини спостерігається крові.
затримка росту і розумового D. Визначення сечовини в крові і сечі.
розвитку, з сечею виділяється велика E. Визначення аміаку в сечі.
кількість оротової кислоти. Ця
спадкова хвороба розвивається 18. У чоловіка 42 років, який
внаслідок порушення: страждає на подагру, в крові
A. * Синтезу піримідинових підвищена концентрація сечової
нуклеотидів. кислоти. Для зниження рівня сечової
кислоти йому необхідний
аллопуринол. Вкажіть, конкурентним D. піримідиніл.
інгібітором якого ферменту є E. Вуглеводів.
алопуринол.
A. * ксантиноксидазу. 22. Хворий 46 років звернувся до
B. аденозіндезамінази. лікаря зі скаргою на біль в суглобах,
C. Аденінфосфорібозілтрансферази яка посилюється напередодні зміни
D. погоди. У крові виявлено підвищення
Гіпоксантінфосфорібозілтрансферази. концентрації сечової кислоти.
E. Гуаніндезамінази. Посилений розпад якої речовини є
найімовірнішою причиною
19. У крові 12-річного хлопчика захворювання?
виявлено зниження концентрації A. * АМФ.
сечової кислоти і накопичення B. ЦМФ.
ксантину та гіпоксантину. C. УТФ.
Генетичний дефект якого ферменту D. УМФ.
має місце у дитини? E. ТМФ.
A. * ксантиноксидазу.
B. аргінази. 23. Чоловік 65 років, який БОЛЕТ
C. Уреаза. подагрою, скаржиться на болі в
D. Орнітінкарбамоілтрансферази. області нирок. При ультразвуковому
E. Гліцеролкінази. обстеженні встановлена наявність
ниркових каменів. Підвищення
20. Під час експерименту було концентрації якої речовини є
показано, що опромінені найбільш імовірною причиною
ультрафіолетом клітини шкіри хворих утворення каменів в даному випадку?
пігментними ксеродермою
повільніше відновлюють структуру A. * Січовий кислоти.
ДНК, ніж клітини нормальних людей B. Холестерину.
внаслідок дефекту ферменту C. Білірубіну.
репарації. Виберіть фермент цього D. Сечовини.
процесу. E. цистину.
A. * Ендонуклеаза.
B. РНК-лігаза. 24. У хворого в крові підвищений
C. праймаза. рівень сечової кислоти, яка клінічно
D. ДНК-полімераза III. проявляється больовим синдромом в
E. ДНК-гіраза. результаті відкладення уратів в
суглобах. В результаті якого процесу
21. Хворий 48 років звернувся до утворюється ця кислота?
лікаря зі скаргами на сильні болі, A. * Розпаду пуринових нуклеотидів.
припухлість, почервоніння в ділянках B. Розпаду пуринових нуклеотидів.
суглобів, підвищення температури, до C. катаболізм гема.
38ºС. У крові виявлено високий вміст D. Розщеплення білків.
уратів. Ймовірною причиною такого E. реутилізацію пуринових основ.
стану може бути порушення обміну:
A. * Пуринів. 25. При спадкової оратацідуріі
B. Коллагена. виділення оротової кислоти в багато
C. Холестерину. разів перевищує норму. Синтез яких
речовин буде порушений при цій D. ОМФ.
патології? E. АМФ.
A. * піримідинових нуклеотидів.
B. пуринових нуклеотидів. 29. З нітратів, нітритів і нітрозамінів в
C. біогенні аміни. організмі утворюється азотистая
D. Січовий кислоти. кислота, яка викликає окисне
E. Сечовини. дезамінування азотистих основ
нуклеотидів. Це може привести до
26. Встановлено, що деякі сполуки, точкової мутації - заміни цитозину на:
наприклад, токсини грибів та деякі A. * Урацил.
антибіотики, можуть пригнічувати B. Гуанін.
активність РНК-полімерази. C. Тимин.
Порушення якого процесу D. Аденін.
відбувається в клітці в разі утиску E. Інозин.
даного ферменту?
A. * Транскрипції. 30. На прийом до терапевта прийшов
B. процесинг. чоловік 37 років зі скаргами на
C. Реплікації. періодичні інтенсивні больові напади
D. Трансляції. в суглобах великого пальця стопи і їх
E. репарації. припухлість. При аналізі сечі
встановлено її різко кислий характер і
27. Онкологічному рожеве забарвлення. З наявністю яких
шебольному призначили препарат речовин це може бути пов'язано?
метотрексат, до якого згодом клітини- A. * Солей сечової кислоти.
мішені пухлини втратили чутливість. B. Хлоридів.
Експресія гена якого ферменту при C. амонійного солей.
цьому змінюється? D. Фосфату кальцію.
A. * дегідрофолатредуктази. E. Сульфату магнію.
B. Тімінази.
C. дезамінази. 31. У 12-річного хлопчика виявлено
D. Фолатоксідази. зниження концентрації сечової
E. Фолатдекарбоксілази. кислоти і накопичення ксантину та
гіпоксантину. Генетичний дефект
28. Похідні птерину - аміноптерин і якого ферменту має місце в організмі
метатрексат - є конкурентними дитини?
інгібіторами дигідрофолатредуктази, A. * ксантиноксидазу.
внаслідок чого вони інгібують B. аргінази.
відновлення дигідрофолієвої кислоти C. Уреаза.
в тетрагідрофолієву кислоту. Ці D. Орнітінкарбамоілтрансферази.
лікувальні засоби призводять до E. Гліцеролкінази.
гальмування міжмолекулярної
транспорту одноуглеродних груп. 32. У хворого болі в дрібних
Біосинтез якого нуклеотиду при суглобах, суглоби збільшені. У
цьому відзначено зниження? сироватці крові повинное вміст
A. * дТМФ. уратів. Обмін яких речовин
B. ІМФ. порушено?
C. УМФ. A. * Пуринів.
B. Аминокислот. C. уратоксідазиі.
C. дисахариди. D. Дігідроурацілдегідрогенази.
D. піримідиніл. E. Уріділілтрансферази.
E. Гліцерину.
37. У хлопчика 8 років хвороба Леш-
33. Хворому в курсі хіміотерапії Нихана. У крові збільшена
пухлини призначено структурний концентрація сечової кислоти.
аналог глутаміну - антибіотик Вкажіть, порушення якого процесу є
азасерін - потужний інгібітор синтезу причиною цього спадкового
пуринових нуклеотидів. Який тип захворювання:
інгібування характерний для цього A. * Розпаду пуринових нуклеотидів.
препарату? B. Синтезу пуринових нуклеотидів.
A. * Конкурентний. C. Синтезу піримідинових
B. Необоротний. нуклеотидів.
C. неконкурентний. D. Розпаду піримідинових
D. безконкурентному. нуклеотидів.
E. аллостеріческого. E. Освіта дезоксирибонуклеотидов.

34. Пацієнту для лікування подагри 38. Підвищений виділення з сечею


був призначений аллопуринол - яких речовин може привести до
інгібітор ферменту ксантиноксидази. виникнення сечокам'яної хвороби?
Який продукт накопичується в A. * уратів і фосфатів.
тканинах і виводиться з сечею при B. Сульфатів і хлоридів.
цьому стані? C. Сульфатів і сечовини.
A. * Гіпоксантин. D. Аминокислот і індікана.
B. Ксантин. E. карбонат, бікарбонатів і цитрату.
C. Сечова кислота.
D. Аденозин. 39. Після обстеження хворому з
E. Інозин. сечокам'яною хворобою призначили
алопуринол - конкурентний інгібітор
35. У хворого 50 років діагностовано ксантиноксидази. Підставою для
подагра, а в крові виявлено цього був хімічний аналіз ниркових
гіперурикемія. Обмін яких речовин каменів, які складалися переважно з:
порушено? A. * уратів натрію.
A. * Пуринів, B. оксалатів кальцію.
B. Ліпідів. C. Холестерину.
C. Аминокислот. D. Фосфату кальцію.
D. Вуглеводів. E. Сульфату кальцію.
E. піримідиніл.
40. Людина 65 років, який страждає
36. У чоловіка 53 років діагностовано на подагру, скаржиться на болі в
сечокам'яна хвороба з утворенням області нирок. При ультразвуковому
уратів. Цьому пацієнту призначено обстеженні установленно наявність
алопуринол, який є конкурентним ниркових каменів. Підвищення
інгібітором ферменту: концентрації якої речовини є
A. * ксантиноксидазу. найбільш можливою причиною
B. Уреаза. утворення каменів в даному випадку?
A. * Січовий кислоти. реакцію утворення карбомоілфосфата
B. Холестерину. не з вільного аміаку, а з глутаміну.
C. Білірубіну. Цей фермент постачає
D. Сечовини. карбомоілфосфат для синтезу:
E. цистину. A. * піримідиніл.
B. Пуринів.
41. Людина 65 років, який страждає C. Сечовини.
на подагру, скаржиться на болі в D. Ліпідів.
області нирок. При ультразвуковому E. Аминокислот.
обстеженні установленно наявність
ниркових каменів. В результаті якого 45. Людина 58 років звернувся до
процесу утворюються ниркові лікаря зі скаргою на біль в суглобах.
камені? При обстеженні виявлено підвищення
A. * Розпаду пуринових нуклеотидів. концентрації сечової кислоти в крові і
B. катаболізмбілка. сечі. Вкажіть, при розпаді яких
C. орнітінового циклу. речовин утворюється сечова кислота?
D. Розпаду гема. A. * пуринових нуклеотидів.
E. Відновлення цистеїну. B. піримідинових нуклеотидів.
C. Аминокислот.
42. Жінка 40 років звернулася до D. Білків.
лікаря зі скаргами на болі в дрібних E. хромопротеїни.
суглобах ніг і рук. Суглоби збільшені,
мають вигляд потовщених вузлів. У 46. У хворого в крові підвищений
сироватці крові виявлено підвищений вміст сечової кислоти, що клінічно
вміст уратів. Причиною є порушення проявляється больовим синдромом
обміну: вследвтвіе відкладення уратів в
A. * Пуринів. суглобах. В результаті якого процесу
B. Аминокислот. вони утворюються?
C. Вуглеводів. A. мРаспада пуринових нуклеотидів.
D. Ліпідів. B. Розпаду піримідинових
E. піримідиніл. нуклеотидів.
C. катаболізм гема.
43. Для лікування злоякісних пухлин D. Розщеплення білків.
призначають метотрексат - E. реутилізацію пуринових підстав.
структурний аналог фолієвої кислоти,
який є конкурентним інгібітором 47. Людина 46 років звернувся до
дегідрофолатредуктази і тому лікаря зі скаргами на болі в дрібних
пригнічує синтез нуклеїнових кислот суглобах, які загострилися у хворого
на рівні: після вживання м'ясної їжі. У хворого
A. * Синтезу мононуклеотидів. діагностовано сечокам'яна хвороба з
B. Реплікації. накопиченням сечової кислоти.
C. Транскрипції. Цьому пацієнту призначено
D. Репарації. алопуринол, який є конкурентним
E. процесингу. інгібітором ферменту:
A. * ксантиноксидазу.
44. Локалізована в цитоплазмі B. Уреаза.
карбомоілфосфатсінтетаза II каталізує C. аргінази.
D. Дігідроурацілдегідрогенази. Ампліфікація гена якого ферменту
E. Карбамоілсінтетази. при цьому спостерігається?
A. * Дегідрофолатредуктази
48..Женщіна 42 років звернулася до B. Тімінази
лікаря зі скаргами на біль в суглобах, C. дезамінази
збільшення і зміна їх форми. В D. тіоредоксінредуктази
аналізах крові виявлено збільшення
концентрації уратів. Встановлено 52. Для нормального протікання
діагноз - подагра. Причиною цього процесу реплікації потрібні
захворювання є порушення обміну: тіміділових нуклеотиди, синтез яких
A. * Пуринів. відбувається за участю
B.Порфірінов. тимідилатсинтетази, як кофермент
C.Пірімідінов. використовується:
D.Амінокіслот. A. * Метилентетрагідрофолат
E.Вітамінов. B .Карбоксібіотін
C .Тіаміндіфосфат
49.Мужчіна 55 років, що страждає D. Піридоксальфосфат
болем в області нирок, поступив до E. Никотинамид.
лікарні. При ультразвуковому
дослідженні пацієнта виявлені камені 53. Новонароджена дитина погано
в нирках. Наявність якої речовини в набирає вагу, в сечі виявлено
сечі є найбільш імовірною причиною підвищений вміст оротової кислоти,
утворення каменів у даного пацієнта? що свідчить про порушення синтезу
A. * Січовий кислоти. піримідинових нуклеотидів. Який
B.Білірубіна. метаболіт необхідно використовувати
C.Білівердіна. для нормалізації метаболізму?
D.Уробіліна. A. * Уридин
E.Креатініна. B .Аденозін
C .Гуанозін
50.Біосінтез пуринового кільця D. Тимидин
відбувається на рибозо-5-фосфаті E. Гістидин
шляхом поступового нарощування 54. У 9-ти місячної дитини з
атомів азоту і вуглецю і замикання затримкою розвитку та проявами
кілець. Джерелом рібозофосфата самоагресіі, вміст сечової кислоти в
служити процес: крові - 1,96 ммоль / л. При якому
A. * пентозофосфатний цикл. метаболічному порушенні це
B.гліколіз. спостерігається?
C.глікогенез. A. * Синдром Леша - Нихана
D.глюконеогенез. B. Подагра
E.глікогеноліз. C. Синдром набутого імунодефіциту
D .Болезнь Гірке
51. При тривалому введенні в E .Болезнь Іценко – Кушинга
організм онкологічних хворих
препарату Молекулярна біологія
"Метотрексату" спостерігається з
часом втрата чутливості до нього
клітин - мішенейопухолі.
1. Для утворення транспортної форми B. глюконеогенез.
амінокислот для синтезу білка на C. фібринолізі.
рибосомах необхідна: D. β-окислення жирних кислот.
A. * тРНК. E. Синтезу біогенних амінів.
B. Ревертаза.
C. ГТФ. 6. Уотсон і Крик встановили, що
D. мРНК. подвійна спіраль ДНК стабілізується
E. Рибосома. за рахунок зв'язків між
комплементарними азотистими
2. При отруєнні аманітіна - отрутою підставами. Які це зв'язку?
блідої поганки - блокується РНК- A. * Водневі.
полімераза В (II). При цьому B. N- гликозидні.
припиняється: C. фосфодіефірнимі.
A. * Синтез мРНК. D. Пептидні.
B. Синтез тРНК. E. Складно-ефірні.
C. Зворотній транскрипція.
D. Синтез праймерів. 7. У всіх живих організмів одні і ті ж
E. Дозрівання мРНК. триплети кодують одні й ті ж
амінокислоти, що дозволяє
3. Виродженість генетичного коду - пересадити E. Coli ген інсуліну
здатність декількох триплетів людини. Як називається ця
кодувати 1амінокислоту. Яка властивість генетичного коду?
амінокислота кодується 1 триплетом? A. * Універсальністю.
A. * Метіонін. B. виродження.
B. Серін. C. Надмірністю.
C. Аланин. D. тріплетном.
D. Лейцин. E. безперервного.
E. Лізин.
8.Больному призначили антибіотик
4. Генний апарат людини містить хлорамфенікол (левомецітін), який
близько 30 тисяч генів, а кількість порушує у мікроорганізмів синтез
варіантів антитіл досягає мільйонів. білка шляхом гальмування процесу:
Який механізм використовується для A. * Елонгації трансляції.
утворення нових генів, що B .Образованіе полірібосом.
відповідають за синтез такої кількості C. Транскрипції.
антитіл? D. процесингу.
A. * Рекомбінація генів. E .Ампліфікаціі генів.
B. Ампліфікація генів.
C. Реплікація ДНК. 9. Відомо, що інформація про
D. Репарація ДНК. послідовність амінокислот в молекулі
E. Освіта фрагментів Окадзакі. білка записана у вигляді
послідовності чотирьох видів
5. У дитини, що страждає на нуклеотидів в молекулі ДНК,
дифтерію, виявлені фібринозні причому різні амінокислоти
нальоти на мигдалинах. Який процес кодуються різною кількістю
пригнічує дифтерійного токсину? триплетів - від одного до шести. Як
A. * Синтезу білка.
називається така особливість D. м-РНК
генетичного коду? E .мя - РНК
A. * Виродженість.
B. Неперекриваемость. 13. Антибіотик ріфоміцін, який
C. Специфічність. використовується при лікуванні
D. Триплетність. туберкульозу, впливає на певні
E. Універсальність. біохімічні процеси. Назвіть їх
A. * Пригнічує РНК - полімерази на
10. У хворих з пігментного стадії ініціації.
ксеродермою шкіра надзвичайно B. Пригнічує ДНК - полімерази на
чутлива до сонячного світла, може стадії ініціації.
розвиватися рак шкіри. Причиною C. Пригнічує ДНК - лігази.
цього є спадкова недостатність D. Пригнічує аміноацил-РНК-
ферменту УФ-ендонуклеази. синтетази.
Внаслідок чого порушується процес: E. Пригнічує дію білкових факторів у
A. * репарації ДНК. синтезі білка.
B. Зворотною транскрипції.
C. Реплікації ДНК. 14. Для лікування урогенітальних
D. Транскрипції ,. інфекцій використовують хінолони -
E. Трансляції. інгібітори ферменту ДНК-гірази.
Вкажіть, який процес порушується
11.Больние пігментна ксеродерма під дією хінолонів в першу чергу
характеризуються аномально A. * реплікація.
високою чутливістю до B. Репарація.
ультрафіолетового світла, C. Ампліфікація генів.
результатом чого є рак шкіри, D. Рекомбінація генів.
внаслідок нездатності ферментних E. Зворотній транскрипція.
систем відновлювати пошкодження
спадкового апарату клітин. З 15. У експериментальних
порушенням якого процесу пов'язана дослідженнях було встановлено, що
ця патологія? стероїдні гормони впливають на
A. * репарації ДНК. протеосінтез. Вкажіть, на який етап
B .Генной конверсії. цього процесу вони впливають
C. Рекомбінації ДНК. A. * Синтез специфічних м-РНК.
D. генної комплементации. B. Синтез АТФ.
E .Редуплікаціі ДНК. C. Синтез специфічних Т-РНК.
D. Синтез ГТФ.
12.На судово-медичну експертизу E. Синтез специфічних р-РНК.
надійшла кров дитини і
передбачуваного батька для 16. Людина 58 років переніс операцію
встановлення батьківства .Укажіте з приводу раку простати. Через 3
ідентифікацію яких хімічних місяці йому провели курс променевої
компонентів необхідно здійснити в і хіміотерапії. У комплекс лікарських
дослідній крові. препаратів входив 5-
A. * ДНК фтордезоксіурідін - інгібітор
B .т- РНК тіміділатсінтази. Синтез якої
C. р- РНК
речовини в першу чергу блокується 20. Жінка 40 років потрапила в
під дією цього препарату? лікарню у важкому стані з
A. * ДНК. симптомами отруєння Amanita
B. і-РНК. phalloides (блідою поганкою). Відомо,
C. р-РНК. що один з токсинів цих грибів блокує
D. т-РНК. синтез попередників мРНК. Цим
E. Білка. токсином є:
A. * аманітіна.
17. Для утворення транспортної B. Актіноміцини.
форми амінокислот для синтезу білка C. Таурин.
на рибосомах необхідна: D. Рібофорін.
A. * Аминоацил-тРНК-синтетаза. E. Бікукулін.
B. ГТФ.
C. м-РНК.
D. Рибосома. Матричні синтези
E. Ревертаза.
1. У хворих з пігментного
18. Генетичний апарат людини ксеродермою шкіра надзвичайно
містить близько 30 тисяч генів, а чутлива до сонячного світла, може
кількість варіантів антитіл досягає розвиватися рак шкіри. Причиною
мільйонів. Який механізм цього є спадкова недостатність
використовується для утворення ферменту УФ-ендонуклеази.
нових генів, які відповідають за Внаслідок чого порушується процес:
синтез такої кількості антитіл? A. * репарації ДНК.
A. * Рекомбінація генів. B. Зворотною транскрипції.
B. Ампліфікація генів. C. Реплікації ДНК.
C. Реплікація ДНК. D. Транскрипції ,.
D. Репарація ДНК. E. Трансляції.
E. Освіта фрагментів Окадзакі.
2. Антибіотик ріфоміцін, який
19. Хворі пігментного ксеродермою використовується при лікуванні
характеризуються аномально туберкульозу, впливає на певні
високою чутливістю до біохімічні процеси. Назвіть їх
ультрафіолетового світла, A. * Пригнічує РНК - полімерази на
результатом чого є рак шкіри стадії ініціації.
внаслідок нездатності ферментних B. Пригнічує ДНК - полімерази на
систем відновлювати пошкодження стадії ініціації.
спадкового апарату клітин. З C. Пригнічує ДНК - лігази.
порушенням якого процесу пов'язана D. Пригнічує аміноацил-РНК-
ця патологія? синтетази.
A. * репарації ДНК. E. Пригнічує дію білкових факторів у
B. генної конверсії. синтезі білка.
C. Рекомбінації ДНК.
D. Точкової мутації ,. 3. Для лікування урогенітальних
E. редуплікацію ДНК. інфекцій використовують хінолони -
інгібітори ферменту ДНК-гірази.
Вкажіть, який процес порушується утворення нових генів, які
під дією хінолонів в першу чергу відповідають за синтез такої кількості
A. * реплікація. антитіл?
B. Репарація. A. * Рекомбінація генів.
C. Ампліфікація генів. B. Ампліфікація генів.
D. Рекомбінація генів. C. Реплікація ДНК.
E. Зворотній транскрипція. D. Репарація ДНК.
E. Освіта фрагментів Окадзакі.
4. В експериментальних
дослідженнях було встановлено, що 8. Хворі пігментного Ксерокс-дермой
стероїдні гормони впливають на характеризуються аномально
протеосінтез. Вкажіть, на який етап високою чутливістю до
цього процесу вони впливають ультрафіолетового світла,
A. * Синтез специфічних м-РНК. результатом чого є рак шкіри
B. Синтез АТФ. внаслідок нездатності ферментних
C. Синтез специфічних Т-РНК. систем відновлювати пошкодження
D. Синтез ГТФ. спадкового апарату клітин. З
E. Синтез специфічних р-РНК. порушенням якого процесу пов'язана
ця патологія?
5. Людина 58 років переніс операцію A. * репарації ДНК.
з приводу раку простати. Через 3 B. генної конверсії.
місяці йому провели курс променевої C. Рекомбінації ДНК.
і хіміотерапії. У комплекс лікарських D. Точкової мутації ,.
препаратів входив 5- E. редуплікацію ДНК.
фтордезоксіурідін - інгібітор
тіміділатсінтази. Синтез якої 9. Жінка 40 років потрапила в
речовини в першу чергу блокується лікарню у важкому стані з
під дією цього препарату? симптомами отруєння Amanita
A. * ДНК. phalloides (блідою поганкою). Відомо,
B. і-РНК. що один з токсинів цих грибів блокує
C. р-РНК. синтез попередників мРНК. Цим
D. т-РНК. токсином є:
E. Білка. A. * аманітіна.
B. Актіноміцини.
6. Для утворення транспортної форми C. Таурин.
амінокислот для синтезу білка на D. Рібофорін.
рибосомах необхідна: E. Бікукулін.
A. * Аминоацил-тРНК-синтетаза. біохімія нирки
B. ГТФ.
C. м-РНК. 1. У хворого нефритом виявлено
D. Рибосома. глюкозурія і аминоацидурия.
E. Ревертаза. Можливою причиною цього є
порушення механізму реабсорбції
7. Генетичний апарат людини містить глюкози і амінокислот шляхом:
близько 30 тисяч генів, а кількість A. * Вторинного Nа + залежного
варіантів антитіл досягає мільйонів. транспорту.
Який механізм використовується для B. Піноцитоз.
C. Простий дифузії. 3. У 18-річного юнака діагностована
D. Первинного активного транспорту. м'язова дистрофія. Підвищення якої
E. фагоцитозу. речовини в сироватці крові найбільш
імовірно при цій патології?
2. При аналізі крові хворого A. * Креатину.
залишковий азот склав 48 ммоль / л, B. міоглобіну.
сечовина 15,3 ммоль / л. Про C. міозину.
захворювання якого органу свідчать D. Лактату.
результати цього аналізу? E. Аланін.
A. * Нирок.
B. Печінки. 4. У відділення травматології
C. Шлунка. поступив хворий після роздроблення
D. Кишечника. м'язової тканини. Вкажіть, який
E. селезінки. біохімічний показник сечі при цьому
буде збільшений
3. У хворого виявили глюкозурию, A. * Креатинін.
вміст глюкози в крові в межах норми. B. Загальні ліпіди.
Результатом яких порушень може C. Глюкоза.
бути викликано це стан? D. Мінеральні солі.
A. * Функції ниркових канальців. E. Сечова кислота.
B. Розпаду глікогену нирки.
C. Функції підшлункової залози. 5. Хворому з прогресуючою м'язовою
D. глюконеогенезі. дистрофією було проведено
E. гліколізу. біохімічне дослідження сечі. Поява
якої речовини у великій кількості в
Біохімія м'язів сечі може підтвердити захворювання
м'язів у даного хворого?
1. У хворого явна прогресуюча A. * Креатину.
м'язова дистрофія. Назвіть показник B. Порфірини.
обміну азоту сечі, характерний для C. Сечовини.
такого стану D. гиппуровую кислоти,
A. * Креатин. E. Креатиніну.
B. Амонійні солі.
C. Креатинін. Біохімія слюни
D. Сечова кислота.
E. Сечовина. 1. Захисна функція слини зумовлена
декількома механізмами, в тому числі
2. У хворого спостерігається атонія наявністю ферменту, який має
м'язів. Назвіть фермент м'язової бактерицидну дію, викликає лізис
тканини, активність якого може бути полісахаридного комплексу оболонки
знижена при цьому? стафілококів, стрептококів. Вкажіть
A. * Креатинфосфокиназа. цей фермент.
B. Амілаза. A. * Лізоцим.
C. транскетолази. B. Альфа-амілаза.
D. глутамінтрансферази. C. Олиго-1, 6-глюкозидази.
E. Каталаза. D. Коллагеназа.
E. Бета-глюкуронідаза.
C. Гипонатриемия.
2. У слині міститься фермент, який D. Гіперфосфатемія.
здатний руйнувати альфа-1,4- E. Гіпернатріємія.
гликозидні зв'язку в молекулі
крохмалю. Вкажіть цей фермент 4. У хворого 35 років, який часто
A. * Альфа-амілаза. вживає алкоголь, на тлі лікування
B. Фосфатаза. сечогінними засобами виникли
C. фруктофуранозидази. сильна м'язова і серцева слабкість,
D. Бета-галактозидаза. блювання, діарея, АТ - 100/60 мм рт.
E. Лізоцим. ст. , Депресія. Причиною такого стану
є посилене виділення з сечею:
Мінеральний обмін A. * Калію.
B. Натрію.
1. Відомо, що в деяких біогеохімічних C. Хлора.
зонах поширене захворювання D. Кальцію.
ендемічний зоб. Недолік якого E. Фосфатів.
біоелемента викликає це 5. Відомо, що в деяких біогеохімічних
захворювання? зонах расспространено захворювання
A. * Йода. ендемічний зоб. Недолік якого
B. Заліза. біоелемента викликає це
C. Цинку. захворювання?
D. Міді. A. * Йода.
E. кобальту. B. Заліза.
C. Цинку.
2. Хворому 50 років. Звернувся в D. Міді.
клініку зі скаргами на загальну E. кобальту.
слабкість, втрату апетиту, аритмію.
Спостерігається гіпотонія м'язів, Енергетичний обмін
мляві паралічі, ослаблення
перистальтики кишечника. Причиною 1. Ціанід калію, який є отрутою,
такого стану може бути: потрапив в організм пацієнта Б. і
A. * Гіпокаліємія. викликав смерть через кілька хвилин.
B. Гипопротеинемия. Найбільш імовірною причиною його
C. Гіперкаліємія. токсичної дії було порушення
D. Гипофосфатемия. активності:
E. Гипонатриемия. A. * цитохромоксидазу.
B. каталази.
3. В клініку доставлений хворий з C. АТФ-синтетази.
такими характерними симптомами: D. НАДФН-дегідрогенази.
слабкість, гіпотонія м'язів, слабкі E. Порушення синтезу гемоглобіну.
паралічі, втрата апетиту, ослаблена
перистальтика, тахікардія, 2. Центральним проміжним Подукти
розширення серця. Яке порушення всіх обмінів (білків, ліпідів,
електролітного обміну можна вуглеводів) є:
припустити у цього хворого? A. * Ацетил-КоА.
A. * Гіпокаліємія. B. сукцініл-КоА.
B. Гіперкаліємія. C. щавлевої-оцтова кислота.
D. Лактат. B. Розпаду глікогену.
E. Цитрат. C. гліколіз.
D. Циклу трикарбонових кислот.
3. У дитини 6 років знижена E. глюконеогенезі.
активність, є ознаки порушення
координації рухів. При обстеженні 7. При розрізі трупа 40 річної жінки
встановлено генетичний дефект судовий експерт встановив, що
одного з ферментів ПВК- смерть настала в результаті отруєння
дегідрогеназну комплексу. Вкажіть, ціанідами. Гальмування якого з
який з перерахованих нижче перерахованих нижче ферментів в
лабораторних показників був першу чергу призвело до смерті в
вирішальним в обґрунтуванні даному випадку?
діагнозу? A. * цитохромоксидазу.
A. * Піруват вище норми. B. глікогенфосфорилази.
B. Аланін нижче норми. C. піруваткарбоксілази.
C. Піруват нижче норми. D. сукцинатдегідрогенази.
D. Аланин вище норми. E. Глюкозо6-фосфатдегідрогенази.
E. Лактат нижче норми.
8. Повне окислення молекули
4. При ентеробіозі призначають глюкози і сполучення його з
акрихін - структурний аналог вітаміну фосфорилюванням еквівалентно
В2. Порушення синтезу яких утворенню наступного сумарної
ферментів у мікроорганізмів викликає кількості молекул АТФ:
цей препарат? A. * 38.
A. * ФАД-залежних дегідрогеназ. B. 8.
B. цитохромоксидазу. C. 12.
C. пептідази. D. 52.
D. НАД залежних дегідрогеназ ,. E. 58.
E. амінотрансфераз.
9. Процес синтезу АТФ, що йде
5. У печінці хворого порушена зв'язано з реакціями окислення за
детоксикація природних метаболітів і участю системи дихальних ферментів
ксенобіотиків. Назвіть цитохром, мітохондрій, називається:
активність якого може бути знижена. A. * окисного фосфорилювання.
A. * Цитохром Р-450. B. Cубстратним фосфорилюванням.
B. Цитохромоксидази ,. C. Вільний окисленням.
C. Гемоглобін. D. фотосинтетичної
D. Цитохром в. фосфорилюванням.
E. Цитохром з1. E. Перекисне окислення.

6. При розрізі трупа 40-річної жінки 10. Для нормального метаболізму


судовий експерт встановив, що клітин необхідні макроергічні
смерть настала в результаті отруєння сполуки. Що належить до макроергів?
ціанідами. Блокування якого процесу A. * Креатинфосфат.
є найбільш можливою причиною B. Креатин.
смерті? C. Креатинін.
A. * Тканинного дихання. D. Глюкозо-6-фосфат.
E. АМФ. D. Пригнічує активність Фад-
дегідрогеназ.
11. Пацієнт звернувся зі скаргами на E. Пригнічує активність Над-
напади утрудненого дихання, втрати дегідрогеназ.
свідомості. З'ясувалося, що він
працює на хімічному підприємстві по 15. Вкажіть, яке з'єднання є
виробництва синильної кислоти. З акцептором аміногруп в реакціях
порушенням функції якого ферменту трансамінування амінокислот:
можуть бути пов'язані зазначені A. Альфа-кетоглутарат.
симптоми? B. аргініносукцінат.
A. * цитохромоксидазу. C. Лактат.
B. Лактатдегідрогенази. D. цітрулін.
C. сукцинатдегідрогеназу. E. Орнитин.
D. каталази.
E. піруватдегідрогенази. 16. Цикл Кребса відіграє важливу
роль в реалізації глюкопластіческого
12. Встановлено, що до складу ефекту амінокислот. Це обумовлено
пестициду входить арсенат натрію, обов'язковим перетворенням
який блокує ліпоєвої кислоти. Безазотисті залишку в
Вкажіть, активність яких ферментів A. * оксалоацетата.
збільшується: B. Малат.
A. * ПВК-дегідрогеназну комплексу. C. Сукцинат.
B. мікросомального окислення. D. Фумарат ,.
C. метгемоглобінредуктази. E. Цитрат.
D. глутатіонпероксидази.
E. глутатіонредуктаза. 17. Чоловік 40 років пробіг 10 км за
60 хв. Як зміниться енергетичний
13. Яка кількість молекул АТФ може обмін в м'язовій тканині?
синтезуватися при повному окисленні A. * Збільшиться швидкість
ацетил КоА в циклі трикарбонових окислення жирних кислот.
кислот? B. Посилиться гліколіз.
A. * 12. C. Посилиться глюконеогенез.
B. 1. D. Посилиться глікогеноліз.
C. 5. E. Посилиться протеоліз.
D. 8.
E. 3. 18. Як називається процес синтезу
АТФ, що йде зв'язано з реакціями
14. Як тироксин впливає на процеси окислення за участю системи
тканинного дихання і окисного дихальних ферментів мітохондрій?
фосфорилювання у хворої A. * Окислювальне фосфорилування.
тиреотоксикозом? B. Cубстратное фосфорилирование.
A. * Роз'єднує процес тканинного C. Cвободное окислення.
дихання і окисного фосфорилювання. D. Фотосинтетичне
B. Блокує транспорт електронів по фосфорилирование.
ланцюгу цитохромів. E. Відновне фосфорилирование.
C. Викликає гідроліз АТФ.
19. У хворих на алкоголізм часто 22. Біологічне окислення і
спостерігається гіповітаміноз B1, знешкодження ксенобіотиків
який є наслідком порушення відбувається за рахунок
харчування. Симптомами гемосодержащіхся ферментів. Який
гіповітамінозу В1 є порушення метал є обов'язковою складовою цих
нервової системи, психози, втрата ферментів?
пам'яті. Чому до дефіциту вітаміну В1 A. * Fe.
особливо чутливі клітини нервової B. Zn.
тканини? C. Co.
A. * Посилюється аеробний розпад D. Mg.
глюкози. E. Mn.
B. Посилюється ліполіз жирової
тканини. 23. Біоенергетика мозку
C. Посилюється окислення жирних характеризується значною
кислот. залежністю від постачання киснем.
D. Підвищується інтенсивність Який субстрат окислення має
гліколізу. найбільше значення для забезпечення
E. Знижується інтенсивність енергією мозку?
гліколізу. A. * Глюкоза.
B. Жирні кислоти.
20. При тиреотоксикозі підвищується C. Кетонові тіла.
продукція тире-ОІД-них гормонів Т3 і D. Гліцерол-3-фосфат.
Т4, розвивається схуднення, E. фосфоенолпіруват.
тахікардія, психічна збудженість та
інше. Як саме впливають тиреоїдні 24. У експериментальних тварин з
гормони на енергетичний обмін в раціону харчування виключили
мітохондріях клітин? ліпоєвої кислоти, при цьому у них
A. * роз'єднує окислювання й спостерігалося пригнічення
фосфорилювання. піруватдегідрогеназного комплексу.
B. Активують субстратне Чим є ліпоєва кислота для цього
фосфорилювання ,. ферменту?
C. Блокують субстратне A. * Коферментом.
фосфорилювання. B. Субстратом.
D. Блокують дихальний ланцюг. C. Інгібітором.
E. Активують окисне D. аллостеріческого регулятором.
фосфорилювання. E. Продуктом реакції.

21. При отруєнні ціанідами настає Біохімія різних тканин


миттєва смерть. В чому полягає
механізм дії ціанідів на 1. У хворого нефритом виявлено
молекулярному рівні? глюкозурія і аминоацидурия.
A. * Інгібують цитохромоксидазу. Можливою причиною цього є
B. Пов'язують субстрати ЦТК. порушення механізму реабсорбції
C. Блокують сукцинатдегідрогеназу. глюкози і амінокислот шляхом:
D. інактивує кисень. A. * Вторинного Nа + залежного
E. Інгібують цитохром В. транспорту.
B. Піноцитоз.
C. Простий дифузії. речовини в сироватці крові найбільш
D. Первинного активного транспорту. імовірно при цій патології?
E. фагоцитозу. A. * Креатину.
B. міоглобіну.
2. При аналізі крові хворого C. міозину.
залишковий азот склав 48 ммоль / л, D. Лактату.
сечовина 15,3 ммоль / л. Про E. Аланін.
захворювання якого органу свідчать
результати цього аналізу? 7. У відділення травматології
A. * Нирок. поступив хворий після роздроблення
B. Печінки. м'язової тканини. Вкажіть, який
C. Шлунка. біохімічний показник сечі при цьому
D. Кишечника. буде збільшений
E. селезінки. A. * Креатинін.
B. Загальні ліпіди.
3. У хворого виявили глюкозурию, C. Глюкоза.
вміст глюкози в крові в межах норми. D. Мінеральні солі.
Результатом яких порушень може E. Сечова кислота.
бути викликано це стан?
A. * Функції ниркових канальців. 8. Хворому з прогресуючою м'язовою
B. Розпаду глікогену нирки. дистрофією було проведено
C. Функції підшлункової залози. біохімічне дослідження сечі. Поява
D. глюконеогенезі. якої речовини у великій кількості в
E. гліколізу. сечі може підтвердити захворювання
м'язів у даного хворого?
4. У хворого явна прогресуюча A. * Креатину.
м'язова дистрофія. Назвіть показник B. Порфірини.
обміну азоту сечі, характерний для C. Сечовини.
такого стану D. гиппуровую кислоти,
A. * Креатин. E. Креатиніну.
B. Амонійні солі.
C. Креатинін. 9. Захисна функція слини зумовлена
D. Сечова кислота. декількома механізмами, в тому числі
E. Сечовина. наявністю ферменту, який має
бактерицидну дію, викликає лізис
5. У хворого спостерігається атонія полісахаридного комплексу оболонки
м'язів. Назвіть фермент м'язової стафілококів, стрептококів. Вкажіть
тканини, активність якого може бути цей фермент.
знижена при цьому? A. * Лізоцим.
A. * Креатинфосфокиназа. B. Альфа-амілаза.
B. Амілаза. C. Олиго-1, 6-глюкозидази.
C. транскетолази. D. Коллагеназа.
D. глутамінтрансферази. E. Бета-глюкуронідаза.
E. Каталаза.
10. У слині міститься фермент, який
6. У 18-річного юнака діагностована здатний руйнувати альфа-1,4-
м'язова дистрофія. Підвищення якої
гликозидні зв'язку в молекулі 14. У хворого 35 років, який часто
крохмалю. Вкажіть цей фермент вживає алкоголь, на тлі лікування
A. * Альфа-амілаза. сечогінними засобами виникли
B. Фосфатаза. сильна м'язова і серцева слабкість,
C. фруктофуранозидази. блювання, діарея, АТ - 100/60 мм рт.
D. Бета-галактозидаза. ст. , Депресія. Причиною такого стану
E. Лізоцим. є посилене виділення з сечею:
A. * Калію.
11. Відомо, що в деяких B. Натрію.
біогеохімічних зонах поширене C. Хлора.
захворювання ендемічний зоб. D. Кальцію.
Недолік якого біоелемента викликає E. Фосфатів.
це захворювання?
A. * Йода. 15.После розпаду комплексу
B. Заліза. глюкагон- рецептор, дія гормону на
C. Цинку. обмін речовин клітини анулюється за
D. Міді. допомогою:
E. кобальту. А. * Фофодіестерази і
фосфопротеінфосфосфотази
12. Хворому 50 років. Звернувся в В. Кінази і фосфорілази
клініку зі скаргами на загальну С. фосфодіестерази і АТФ-ази
слабкість, втрату апетиту, аритмію. D. фосфорилазу і
Спостерігається гіпотонія м'язів, фосфопротеінфосфотази
мляві паралічі, ослаблення Е. глікогенсінтази і кінази
перистальтики кишечника. Причиною фосфорілази
такого стану може бути:
A. * Гіпокаліємія. 16.Убольного підвищена
B. Гипопротеинемия. концентрація сечовини і залишкового
C. Гіперкаліємія. азоту в крові. Про патології якого
D. Гипофосфатемия. органу це свідчить?
E. Гипонатриемия. А. * Нирок
В. Печінки
13. У клініку доставлений хворий з С.Скелетних м'язів
такими характерними симптомами: Д.Сердца
слабкість, гіпотонія м'язів, слабкі Е.Надпочечніков
паралічі, втрата апетиту, ослаблена
перистальтика, тахікардія, 17.Прі ацидозі в нирках збільшується
розширення серця. Яке порушення кількість аміаку, а в сечі збільшується
електролітного обміну можна кількість хлориду-амонію. З
припустити у цього хворого? підвищенням активності якого
A. * Гіпокаліємія. ферменту в нирках це пов'язано?
B. Гіперкаліємія. А. * глутамінази.
C. Гипонатриемия. В.Глутамінсінтази
D. Гіперфосфатемія. С.Глутаматдегідрогенази
E. Гіпернатріємія. Д.Карбомоілфосфатсінтази
Е. α-кетоглутаратдегидрогенази
18.Убольной сеча постійно 22. У нирках чоловіки посилився
коричневого кольору, кількість синтез ферменту реніну. До яких
індікана в сечі збільшилася .Про що наслідків це призведе?
це свідчить? А. * Підвищиться артеріальний тиск
А. * Про посилення гниття білків в крові
кишечнику. В. Зменшиться артеріальний тиск
В.О зниженні інтенсивності крові
орнітінового циклу С. Чи не зміниться артеріальний тиск
С. Про зниження клубочкової крові
фільтрації нирок Д. Зменшиться кількість
Д. Про активацію процесів альдостерону в крові
дезамінування Е. Не зміниться кількість
Є.О гальмуванні глюконеогенезу альдостерону в крові

19.Как з'єднання може з'явитися в сечі 23.У хворого 34 років має місце
при порушенні антитоксичної функції знижена витривалість до фізичних
печінки? навантажень в той час, як в скелетних
А. * Индикан м'язах вміст глікогену підвищено. Зі
В. Гліцин зниженням активності якого
С.Фенілглюкуроновая кислота ферменту це пов'язано?
Д.Бензойная кислота A. * глікогенфосфорилази
Е.Мочевіна B.Глюкозо-6-фосфатдегідрогенази
C. фосфофруктокинази
20.Паціент звернувся до лікаря зі D. глікогенсінтази
скаргами на задишку, яка виникала E.Глюкозо-6-фосфатази
після фізичного навантаження.
Клінічне обстеження виявило анемію 24.У людей, після тривалого
та наявність парапротеина в зоні фізичного навантаження виникають
гамма-глобулінів. Який показник в інтенсивні болі в м'язах. Що може
сечі необхідно визначити для бути найбільш можливою причиною
підтвердження діагнозу мієломи? цього?
A. * Білок Бенс-Джонса A. * Накопичення в м'язах молочної
B. Білірубін кислоти
C. Гемоглобін B. Посилений розпад м'язових білків.
D. Церулоплазмін C. Накопичення креатиніну в м'язах.
E. анти трипсину D .. Підвищена збудливість в м'язах.
E. Підвищення вмісту АДФ в м'язах.
21.Прі захворюванні якого органу
порушується утворення активної 25.У хворого спостерігається атонія
форми вітаміну Д3 тому, що не м'язів. Назвіть фермент м'язової
відбувається його гідроксилювання? тканини, активність якого може бути
А. * Нирок знижена?
В. Наднирників A. * Креатинфосфокиназа.
С. Скелетних м'язів B. Амілаза.
Д.Сердца C. транскетолази
Е.Щітовідной залози D.Глутамінтрансфераза.
E. Каталаза.
А. * Глутаминовая кислота
26.Как запасне макроергіческое В. Гіалуронова кислота
з'єднання утворюється в скелетних С.Янтарная кислота
м'язах за участю АТФ? Д. Яблучна кислота
А. * Креатинфосфат Е. Глюкуронова кислота
В.1,3-дифосфоглицерата
С. фосфоенолпіруват 31.Какие опіатні (морфіноподібні)
Д. ЦТФ пептиди, які проявляють
Е.ТТФ знеболювальну дію, синтезуються в
гіпофізі?
27.Как білок бере участь в А. * Енкефаліни
депонування кисню в скелетних В. Глікозаміноглікани
м'язах? С. Кініни
А. * Миоглобин Д. сіалові кислоти
В. кальмодуліном Е. Поліаміни
С. Цитохром
Д. Креатинфосфат 32.Амміак особливо токсичний для
Е. Гемоглобін ЦНС людини. Вкажіть головний шлях
його знешкодження в нервової
28.К фибрилярную елементів тканини:
сполучної тканини належить колаген, А. * Синтез глутаміну
еластин і ретикулін. Вкажіть В. Синтез солей амонію
амінокислоту, яка входить тільки до С. Синтез сечовини
складу колагену і визначення якої в Д. Трансамінування
біологічних рідинах Е. Утворення парних з'єднань,
використовується для діагностики
захворювань сполучної тканини:
A. * Гідроксипролін
B. Пролін
C. Гліцин
D. Фенілаланін
E. Лізин

29.Какие нейромедіатор, який


гальмує ЦНС, синтезується в тканини
мозку?
А. * ГАМК
В. триптаміни
С. Дофамин
Д. Серотонін
Е. Норадреналін

30.Возбужденіе нервової системи


супроводжується підвищенням вмісту
амонію в нервової ткані.Какое
речовина має особливу роль у
знешкодженні аміаку?

You might also like